You are on page 1of 119

Neurology

Jacquelyn L. Bainbridge, Pharm.D., FCCP


Department of Clinical Pharmacy and Department of Neurology
Skaggs School of Pharmacy and Pharmaceutical Sciences
University of Colorado Denver,
Anschutz Medical Campus
Aurora, Colorado

J. Mark Ruscin, Pharm.D., FCCP, BCPS


Southern Illinois University Edwardsville
Edwardsville, Illinois
Neurology

Neurology
Jacquelyn L. Bainbridge, Pharm.D., FCCP
Department of Clinical Pharmacy and Department of Neurology
Skaggs School of Pharmacy and Pharmaceutical Sciences
University of Colorado Denver,
Anschutz Medical Campus
Aurora, Colorado

J. Mark Ruscin, Pharm.D., FCCP, BCPS


Southern Illinois University Edwardsville
Edwardsville, Illinois

ACCP Updates in Therapeutics® 2016: Ambulatory Care Pharmacy Preparatory Review and Recertification Course

1-403
Neurology

Learning Objectives 21. Discuss the potential role of pharmacists in spe-


cialty pharmacies in improving the care of patients
1. Identify the seizure type(s), and devise a treatment with MS.
plan for a patient with new-onset or refractory 22. Discuss the common long-term complications asso-
epilepsy. ciated with spinal cord injuries.
2. Describe the mechanisms of action of recommended 23. Identify signs or symptoms associated with
antiepileptic drugs (AEDs). Alzheimer disease (AD) or Parkinson disease (PD)
3. Select an appropriate AED regimen for a patient that may be drug induced.
with epilepsy. 24. Describe reasonable expectations and limitations of
4. Identify common adverse effects and drug inter- available therapies for the treatment of patients with
actions for first- and second-generation AEDs, AD, traumatic brain injury (TBI), PD, and essential
focusing on the cytochrome P450 system. tremor (ET).
5. Formulate a monitoring plan for a given patient 25. Differentiate parkinsonian tremor from ET accord-
receiving AED therapy. ing to the patient’s response to drug therapy.
6. Discuss pertinent patient education counseling 26. Recommend an appropriate plan for the ini-
points, together with patient assistance programs. tiation, titration, monitoring, and adjustment of
7. Choose an appropriate AED regimen for a special pharmacotherapy for cognitive/functional symp-
population patient (e.g., pregnant, status epilepticus). toms in patients with AD, TBI, PD, or ET.
8. Distinguish between the signs and symptoms of 27. Recommend appropriate strategies for the medical
headache types. management of patients with psychiatric or behav-
9. Recommend an appropriate pharmacologic therapy ioral symptoms related to AD or PD.
for a patient with an acute migraine headache.
10. Choose an appropriate prophylactic regimen for a
patient with a migraine headache. Self-Assessment Questions
11. Identify agents that have been implicated in causing Answers and explanations to these questions can be
medication-overuse headache. found at the end of the chapter.
12. List common migraine triggers.
13. Provide patient education regarding pharmacologic 1. Dr. K. would like assistance choosing an antiepileptic
and lifestyle interventions for migraine headache. drug (AED) that will not interfere with his transplant
14. Describe the rationale for using urine drug testing, patient’s cyclosporine therapy. Which AED is most
drug monitoring programs, and medication con- appropriate to use in this patient (assuming all will
tracts to ensure patient adherence to and prevention provide adequate seizure control)?
of diversion of controlled substances. A. Carbamazepine.
15. Recommend an appropriate pharmacologic therapy B. Lacosamide.
for a patient with a chronic pain condition. C. Oxcarbazepine.
16. Apply current guidelines on the use of chronic D. Phenytoin.
opioid therapy for a patient with chronic pain.
17. Identify the role of a pharmacist within an inter- 2. Your patient will be admitted for a cholecystectomy.
disciplinary team providing care to patients with She is taking carbamazepine 400 mg orally three
chronic pain in an ambulatory care setting. times daily. She will be unable to take anything by
18. Provide patient education regarding the pharma- mouth for 3 days after surgery and requires an AED
cologic agents used in the treatment of myasthenia available as an injectable formulation. Which is the
gravis. best AED treatment to recommend during this time
19. List currently approved treatment options for the (assuming all will provide adequate seizure control)?
treatment of multiple sclerosis (MS).
A. Carbamazepine.
20. Discuss the common adverse effects and monitor-
B. Levetiracetam.
ing parameters associated with the pharmacologic
C. Topiramate.
agents used in MS.
D. Lamotrigine.

ACCP Updates in Therapeutics® 2016: Ambulatory Care Pharmacy Preparatory Review and Recertification Course

1-404
Neurology

3. Typically, carbamazepine is initiated at a low dose. A. Continue nortriptyline 75 mg/day.


Which option provides the best reason for starting at B. Increase nortriptyline to 150 mg/day.
a low dose? C. Take propranolol 80 mg/day.
A. Precipitation of absence seizures. D. Take candesartan 16 mg/day.
B. Dizziness caused by initial dose.
7. On the basis of duration of action, which is the best
C. Reduce risk of rash.
option for abortive treatment of L.P.’s migraines?
D. Reduce risk of hyponatremia.
A. Sumatriptan 25 mg.
4. You are called by the emergency department (ED) B. Frovatriptan 2.5 mg.
physician and asked about your patient from the C. Rizatriptan 5 mg.
ambulatory clinic. The patient is unconscious with D. Almotriptan 6.25 mg.
continuous seizures. He has been having seizures
for 12 minutes. Which is the most appropriate initial 8. M.J. is a 50-year-old man who comes to the clinic
therapy for this patient? seeking abortive treatment for his migraine head-
aches. The patient has a history of benign prostatic
A. Lorazepam.
hyperplasia, Parkinson disease, and hypercholes-
B. Phenobarbital.
terolemia. The patient’s home medications include
C. Propofol.
tamsulosin 0.4 mg/day, selegiline 5 mg twice daily,
D. Levetiracetam.
and simvastatin 20 mg/day. Given the potential drug
interactions, which is the best abortive treatment
Questions 5–7 pertain to the following case.
option for his migraine headache?
L.P. is a 46-year-old overweight woman who presents to
the clinic with a severe migraine attack. She was given a A. Eletriptan.
diagnosis of migraine headaches when she was 16 years B. Rizatriptan.
old. The patient says her headaches were mostly controlled C. Frovatriptan.
until about 6 months ago, when they began occurring D. Naratriptan.
more often. She says her migraines usually last 24 hours
9. A 35-year-old patient, A.A., is seen in the clinic for the
and usually occur around the start of her menstrual cycle.
first time and undergoes a physical examination and
She is currently experiencing severe pain, nausea, and
pain examination. She is deemed to be at high risk of
vomiting. Her home drugs include nortriptyline 75 mg by
misusing chronic opioid therapy on the basis of the
mouth daily, an oral contraceptive, and one multivitamin
Diagnosis, Intractability, Risk, Efficacy (DIRE) tool.
by mouth daily. The patient works a very stressful job
Her risk is increased by her comorbid conditions (bipolar
and often misses meals. The patient hydrates herself with
disorder; currently controlled) and history of substance
three 32-oz caffeinated soft drinks throughout the day to
use. Her primary care physician wishes to treat her
“keep her going” and help with her dry mouth. She also
current musculoskeletal pain, which is moderate—6
reports consuming red wine regularly in the evening to
on a 10-point scale. He has given her a diagnosis
calm down after a difficult day. Because of her stressful
of fibromyalgia (FM). She currently is uninsured.
job, her sleep schedule is sporadic.
Which would be the most appropriate first step in her
management, assuming she has received no therapy for
5. Which is the most appropriate nonpharmacologic
her FM and her only medication is divalproex?
therapy to recommend to L.P.?
A. Acetaminophen would be the most appropriate
A. Switch to diet soda.
choice for her moderate pain.
B. Limit red wine consumption.
B. Tramadol would be most appropriate for her
C. Begin taking naps during the day.
moderate pain.
D. Subscribe to a weight management program.
C. Duloxetine would be a good first agent to
address her pain and depression.
6. Given the information provided above, which is the
D. Gabapentin would be a good choice for this
best option for migraine prevention in L.P.?
patient with a musculoskeletal pain disorder.

ACCP Updates in Therapeutics® 2016: Ambulatory Care Pharmacy Preparatory Review and Recertification Course

1-405
Neurology

10. T.Z. is a patient whose low back pain has been 12. F.M.’s physician would like to initiate low-dose
very difficult to control. You initiated a therapeutic chronic opioid therapy for her neuropathic pain. He
trial with several adjuvant analgesics that resulted would like to start therapy with methadone 2.5 mg
in only a 1-point reduction on the pain scale. The every 12 hours. Which is the best recommendation
pain covers one whole side of his low back and hip regarding this therapy?
and shoots down his leg. You subsequently tried the A. She requires an electrocardiogram (ECG)
combination opioid, hydrocodone/acetaminophen before starting therapy.
5/325 mg, on a scheduled basis, which resulted in B. She is a poor methadone candidate because
some relief. You have followed your clinic policies, of her age.
and T.Z. is on a treatment agreement, which includes C. She is a poor methadone candidate because
treatment goals. However, he comes to the clinic and she is opioid naive.
reports that he tried some of his neighbor’s extended- D. An appropriate methadone initial dose is
release morphine. T.Z. states that for the first time in 5 mg every 12 hours.
more than a year, he was able to get some real sleep.
You are very uncomfortable with this behavior. You 13. D.D. is receiving chronic opioid therapy for a back
are aware that he has also made several trips to the injury. Currently, D.D. is prescribed morphine
ED for intramuscular injections of opioids. Which is extended release 90 mg three times daily. D.D. has
the best next step in his management, according to constant itching. He stopped taking diphenhydramine
the guidelines for managing chronic opioid therapy? in the past month, and the itching has become
A. Increase his opioid dose of the weak opioid intolerable. The attending physician wants to change
by 20%. the prescription to fentanyl transdermal patches.
B. Convert his medication to extended-release Which is the most appropriate starting dose?
morphine by using an equianalgesic dose to his A. 25-mcg/hour patch applied every 72 hours.
current dose of acetaminophen/hydrocodone. B. 50-mcg/hour patch applied every 72 hours.
C. Refer him to an interventional pain specialist C. 75-mcg/hour patch applied every 72 hours.
for further spinal injections. D. 150-mcg/hour patch applied every 72 hours.
D. Refer him to a pain/addiction
management clinic.

11. You are asked to see F.M., a 70-year-old patient with


widespread pain that is chronic (greater than 5 years’
duration). The patient is new to your clinic. The
patient currently takes the following medications:
duloxetine 60 mg daily for arthritis (knee and hip);
gabapentin 600 mg three times daily for diabetic
neuropathy; and acetaminophen 500 mg three or four
times daily as needed for arthritis pain. Currently,
her diabetes and hypertension (HTN) are controlled;
however, she expresses the desire for better pain
control. She is very adherent to her regimen and
is well educated about her disease states. She is a
college graduate. The medical resident says her pain
is an 8/10. Which tool would be best to assess her
pain currently?
A. Visual Analog Scale (VAS).
B. Brief Pain Inventory.
C. McGill Pain Questionnaire.
D. Wong-Baker FACES scale.

ACCP Updates in Therapeutics® 2016: Ambulatory Care Pharmacy Preparatory Review and Recertification Course

1-406
Neurology

14. P.W. arrives at the clinic for his 8:00 a.m. 16. Your clinic is designing a protocol for the care of
appointment and, as part of his patient agreement, patients receiving chronic opioid therapy. The
completes his patient evaluation forms regarding physicians have decided they will treat patients
his pain and response to his medication. At his with a remote history of substance abuse disorders.
original visit 3 months ago, he was deemed at high Which would be the most appropriate clinical tool
risk of misusing chronic opioid therapy. Before to evaluate individual patient risk associated with
he is seen by his primary care physician, he has a chronic opioid therapy for these patients during
urine drug screen. He indicates in his questionnaires their opioid therapy?
that his pain is slightly improved and that he has A. CAGE substance abuse screening tool.
been taking his medication. His list of prescribed B. Opioid risk tool (ORT).
medications is as follows: acetaminophen 325 mg/ C. Screener and Opioid Assessment for Patients
hydrocodone 10 mg: take 1 tablet every 12 hours with Pain (SOAPP).
for pain (most recent dose before bed the previous D. Current opioid misuse measure (COMM).
night; patient is not taking it every 12 hours, but
only when necessary for severe pain); gabapentin 17. P.V. requires appropriate therapy for his mild to
600 mg three times daily; sertraline 25 mg daily; moderate pain. He currently takes no medications for
hydrochlorothiazide 25 mg; and an over-the-counter chronic pain. Which would be the most appropriate
cough syrup (the nighttime version), the name of first step in therapy?
which the patient cannot remember. The results
A. Ibuprofen 400 mg three times daily for 5 days.
of the urine drug screen are as follows: opioids,
B. Acetaminophen/hydrocodone 500 mg/10 mg:
negative; amphetamines, positive; benzodiazepines,
Take 1 tablet every 4–6 hours as needed for
negative; cannabinoids, negative; and barbiturates,
5 days.
negative. From the urine drug screen results, which
C. Tramadol 50 mg three times daily for 5 days.
is the next best step in this patient’s management?
D. Celecoxib 200 mg daily for 5 days.
A. This patient’s chronic opioid therapy should
be discontinued because of his using inappro- 18. S.S., a 35-year-old patient with chronic low back
priate medications. pain with sciatica, has been treated with opioids for
B. This patient should be rescreened on upcoming more than 10 years. She currently takes the following
visits by using the immunoassay and receive medications: oxymorphone extended release 20 mg
patient education regarding his prescribed every 12 hours and ramipril 10 mg daily. Her only
pain therapy. other conditions are HTN and obesity. She weighs
C. This patient should be rescreened at this visit 150 kg, and her creatinine clearance is normal
by using a more sensitive test, namely, gas for her age. Her current pain level is a 3/10 in the
chromatography–mass spectrometry (GC-MS). mornings, which shifts to a 9/10 in the evening. Her
D. This patient should be rescreened at this visit pain medication slightly reduces her pain level in the
by using serum quantitative screening tests. evening. She describes her pain with words such as
shooting and stabbing, running down the outside of
15. Which drug is most likely to be associated with her leg to her ankle. Which is the best medication to
a positive result for a urine drug screen (enzyme recommend as an initial adjuvant medication for her?
multiplied immunoassay technique [EMIT] point of
A. Pregabalin.
care – immunoassay form) for opioids?
B. Gabapentin.
A. Methadone. C. Duloxetine.
B. Verapamil. D. Venlafaxine.
C. Tramadol.
D. Fentanyl.

ACCP Updates in Therapeutics® 2016: Ambulatory Care Pharmacy Preparatory Review and Recertification Course

1-407
Neurology

19. Despite several months of adjuvant therapy, S.S. is still 22. L.L. is experiencing more frequent multiple
in severe pain. Her medication is going to be converted sclerosis (MS) relapses and has been treated with
to fentanyl patches. You explain to the physician that interferon β-1a for 3 years. L.L., who is 28 years
this medication is covered by the U.S. Food and Drug old, is thinking of starting a family in 5 years. She
Administration (FDA) Risk Evaluation and Mitigation indicates a preference for an oral agent. Which is the
Strategies (REMS) program. Which course of action best DMT to recommend currently?
does this program most encourage for prescribers? A. Mitoxantrone.
A. Complete prescriber training to obtain a B. Teriflunomide.
specialized prescribing registry number for C. Dimethyl fumarate.
fentanyl patches to place on her prescription. D. Glatiramer acetate.
B. Complete prescriber training related to fen-
tanyl patches, and provide patient education. 23. P.W. is a 44-year-old man who had a road traffic
C. Complete prescriber training for fentanyl accident 3 years ago, resulting in complete spinal
patches to obtain approved education materials. injury at the level of T4. After the acute episode, P.W.
D. Complete prescriber training, provide patient suffered the associated complications from the loss of
education, and provide a medication guide. motor and sensory function from below the level of
injury, which include bladder dysfunction (requiring
20. A.J. is a 40-year-old man with newly diagnosed an indwelling catheter) and chronic constipation.
myasthenia gravis (MG). He experiences bilateral Today, he presents to the clinic with blood pressure
upper extremity muscle weakness, sagging of the (BP) of 200/110 mm Hg, headache, and nasal
right side of face, drooping of the right eyelid, congestion, indicative of autonomic dysreflexia.
and fatigue. He is given a new prescription for Which is the most appropriate action now?
pyridostigmine today at his neurology office visit. A. He should be placed in a supine position
Which is the most appropriate information to immediately.
provide A.J. regarding his medication? B. He should be given a long-acting antihyper-
A. Pyridostigmine is indicated to help prevent tensive agent and continue with this therapy
the further progression of MG. thereafter.
B. Pyridostigmine should be taken only on an as- C. His indwelling catheter should be checked for
needed basis to prevent the development obstruction and corrected.
of tolerance. D. He should be given an oral dose of lactulose
C. The most common adverse effects include immediately.
abdominal cramping, diarrhea, nausea,
and vomiting.
D. The effect of the drug can be seen only
1–2 weeks after treatment initiation.

21. K.D. is a 24-year-old woman recently given a diagnosis


of relapsing-remitting multiple sclerosis (RRMS).
She reports minimal disability currently. Magnetic
resonance imaging (MRI) reveals one white matter
brain lesion. She is severely depressed and not taking
an antidepressant. Which disease-modifying therapy
(DMT) is most appropriate to use for this patient
(assuming all will provide a reduction in relapses)?
A. Interferon β-1a.
B. Interferon β-1b.
C. Glatiramer acetate.
D. Fingolimod.

ACCP Updates in Therapeutics® 2016: Ambulatory Care Pharmacy Preparatory Review and Recertification Course

1-408
Neurology

24. A 71-year-old female patient is being seen in the clinic A. Liver function tests will be required during
for a routine annual visit. As part of the evaluation, the first 12 weeks.
a Mini-Mental State Examination (MMSE) is B. The drug will significantly improve the
performed, on which she scores 23/30. One year symptoms of the disease.
ago, she scored 26/30. Her medical problems include C. Adding vitamin E to rivastigmine will improve
HTN, osteoporosis, hypothyroidism, and overactive the efficacy of the drug.
bladder. Her current medication list includes D. Avoid taking medications such as diphen-
hydrochlorothiazide 12.5 mg/day, lisinopril 10 hydramine or chlorpheniramine while
mg/day, alendronate 70 mg once weekly, calcium/ taking this drug.
vitamin D 500 mg/400 international units twice
daily, levothyroxine 100 mcg/day, and tolterodine 4 27. A 76-year-old woman (height 67 inches, weight
mg/day. Her examination is unremarkable, and BP is 51 kg) has been taking donepezil 10 mg/day for 11
controlled at 132/84 mm Hg. A thyroid-stimulating months and has tolerated it well, except for mild to
hormone (TSH) measurement 2 months ago was 2.2 moderate nausea on rare occasions. She is in the
mIU/mL. Which factor is most likely contributing to clinic today with her daughter, who states that she
this patient’s cognitive changes? is concerned about her mother’s worsening memory
A. Hypothyroidism. and daily functioning. Her mother’s MMSE score
B. Alzheimer disease (AD). today is 15/30. One year ago, it was 19/30. No
C. Tolterodine. evidence of acute medical problems is found during
D. Levothyroxine. the examination, and a depression screen is negative.
Which is the most appropriate recommendation
25. A 77-year-old man with recently diagnosed probable currently to address the daughter’s concerns?
AD (MMSE 22/30) began treatment with galantamine A. Increase the donepezil dose to 23 mg/day.
extended release (ER) 8 mg/day 3 months ago. After B. Decrease the donepezil dose to 5 mg/day.
taking this dose for 1 month, his dose was titrated C. Continue donepezil and initiate
to galantamine ER 16 mg/day, but he was unable to memantine therapy.
tolerate this dose because of nausea; therefore, the D. Discontinue donepezil and initiate
dose was decreased to 8 mg/day. He received the memantine therapy.
8-mg dose for 6 additional weeks without problems,
and his dose was again titrated to galantamine ER 28. A 48-year-old man is receiving care in an inpatient
16 mg/day about 1 week ago. The patient’s wife is rehabilitation setting. He was involved in a motor
calling the clinic today to report that she has not seen vehicle accident 4 weeks earlier and suffered a
any improvement in her husband’s symptoms and severe nonpenetrating brain injury. He has been
he has been experiencing nausea and has not been in a minimally conscious state since entering the
eating well since the dose increase. Which is the best facility. The physician caring for this patient has
management strategy for this patient? prescribed amantadine 100 mg twice daily for this
A. Discontinue galantamine and initiate donepezil patient. Which statement is most appropriate with
5 mg/day. respect to amantadine use in this patient?
B. Decrease the galantamine ER dose to 8 mg/day. A. Amantadine may improve functional recovery
C. Discontinue galantamine and initiate meman- in the postacute treatment phase.
tine 5 mg/day. B. Amantadine has no effect on functional recov-
D. Discontinue galantamine and initiate rivastig- ery in the postacute treatment phase.
mine 6 mg twice daily. C. Amantadine may improve functional recovery,
but at a dose of 400–800 mg twice daily.
26. A 72-year-old woman is going to begin treatment D. Amantadine may improve functional recovery
with the rivastigmine patch for moderate AD. after mild to moderate, but not severe,
Which is the best information to provide the patient brain injuries.
and caregiver?

ACCP Updates in Therapeutics® 2016: Ambulatory Care Pharmacy Preparatory Review and Recertification Course

1-409
Neurology

29. A 64-year-old man with HTN, osteoarthritis, type 31. A 68-year-old woman with recently diagnosed
2 diabetes mellitus, renal insufficiency (estimated PD began treatment with levodopa/carbidopa
creatinine clearance [CrCl] 25 mL/minute), and 100/10 mg three times daily 5 days ago. She is
gastroesophageal reflux disease presents to the clinic calling the clinic to report symptoms, including
with symptoms of rigidity in the upper extremities, nausea and light-headedness. She states that her
mild hand tremors, and some changes in his gait. PD symptoms are improved and that her ability to
He takes hydrochlorothiazide 25 mg once daily and get around and function is better, but the adverse
amlodipine 5 mg once daily for HTN, ibuprofen 400 effects are quite bothersome. Which is the best
mg twice daily as needed for osteoarthritis, glipizide recommendation for this woman?
5 mg twice daily for type 2 diabetes mellitus, A. Add rasagiline 0.5 mg/day to levodopa/
and metoclopramide 10 mg four times daily and carbidopa.
omeprazole 20 mg once daily for esophageal reflux. B. Decrease the levodopa/carbidopa dose to
He states that the symptoms are quite bothersome 100/10 mg twice daily.
and that they are affecting his functioning. Which is C. Discontinue levodopa/carbidopa and initiate
the best initial recommendation for addressing this ropinirole 0.25 mg three times daily.
man’s symptoms? D. Change levodopa/carbidopa to 100/25 mg
A. Initiate levodopa/carbidopa 100/25 mg three times daily.
three times daily.
B. Initiate ropinirole 0.25 mg twice daily. 32. A 72-year-old woman with PD has been taking
C. Discontinue metoclopramide 10 mg levodopa/carbidopa for more than 6 years. Her
four times daily. current dose is 100/25 mg, two tablets in the morning
D. Discontinue amlodipine 5 mg once daily. and one tablet at noon, 4 p.m., and 8 p.m. She has
been experiencing motor complications (on-off
30. A 72-year-old man presents to the clinic with a symptoms, dyskinesias) related to chronic levodopa
6-month history of intermittent tremor in his hands and therapy, so her physician added rasagiline 0.5 mg
problems with his gait. He states that the symptoms once daily in the morning to her regimen 1 week ago.
have worsened since his past visit 3 months earlier. She is in the clinic today stating that she is having
Laboratory test results and a computed tomography adverse effects since the new medication was added,
(CT) scan performed at that time were normal. including nausea and involuntary movements, which
Physical examination reveals a resting hand tremor, are identified on examination as dyskinesias. Which
left greater than right, which ceases with purposeful is the best recommendation for this woman?
movement, as well as mild cogwheel rigidity in A. Discontinue rasagiline and change to selegiline
both elbows, left greater than right. Postural reflexes 5 mg twice daily.
and balance assessments are mildly abnormal. A B. Decrease the levodopa/carbidopa dose to
gait assessment reveals reduced arm swing while 100/25 mg one tablet in the morning and at
walking. He states that the symptoms are affecting noon, 4 p.m., and 8 p.m.
his daily life and that he is concerned about his ability C. Discontinue rasagiline and add ropinirole
to continue to work. From his history and physical 0.25 mg twice daily.
examination, he is given a diagnosis of Parkinson D. Add amantadine to levodopa/carbidopa
disease (PD). Which is the most appropriate initial and rasagiline.
therapy for this patient?
A. Benztropine 0.5 mg twice daily.
B. Coenzyme Q10 900 mg daily.
C. Levodopa/carbidopa 100/25 mg three times
daily plus entacapone 200 mg three times daily.
D. Pramipexole 0.125 mg three times daily.

ACCP Updates in Therapeutics® 2016: Ambulatory Care Pharmacy Preparatory Review and Recertification Course

1-410
Neurology

I.  EPILEPSY

A. Epidemiology
1. Third most common neurologic disorder
2. About 10% of the general population will experience a seizure at some point in their lifetime.
3. Around 1%–2% of the population has a diagnosis of epilepsy.
4. 40 million individuals worldwide
5. 2.3 million individuals in the United States
6. One in 100 adults has a formal diagnosis of epilepsy.
7. One in 50 children has a formal diagnosis of epilepsy.
8. Bimodal distribution of the incidence of first seizure
a. Before 1 year of age (from genetic and perinatal causes)
b. After 65 years of age (from stroke and other central nervous system [CNS] insults)

B. Etiology – Known Causes of an Isolated Seizure; this is inconsistent with a diagnosis of epilepsy
1. Alcohol ingestion
2. Withdrawal of illicit drugs or alcohol
3. Drugs that can lower the seizure threshold or precipitate a seizure
4. Metabolic disturbances (i.e., hypoglycemia)
5. CNS infections, febrile illness

Table 1. Drugs Reported to Precipitate Seizures


Agents or Class of Drugs
Acetylcholinesterase inhibitors Cyclosporine Narcotics
Amphetamines Dalfampridine Penicillins
Anticholinergics Estrogen (can worsen epilepsy) Pyrimethamine
Antidepressants Imipenem Quinolones
Antiemetics Iodinated contrast dyes (angiography) Sympathomimetics
Antihistamines Isoniazid Tacrolimus
Antipsychotics Lithium Theophylline
Baclofen (when withdrawn) Local anesthetics Tramadol
β-Blockers Methotrexate
Cephalosporin Methylphenidate Metronidazole
Cocaine

6. Epilepsy is the propensity to have unprovoked seizures repeatedly; the diagnosis can be made after
one episode.
a. Idiopathic: 68%
b. Cerebrovascular disease: 8%–12%
c. Developmental disabilities: 6%
d. CNS trauma: 4%
e. CNS tumors: 4%
f. CNS infections: 3%
g. Degenerative diseases: 2%
h. Other: 1%

ACCP Updates in Therapeutics® 2016: Ambulatory Care Pharmacy Preparatory Review and Recertification Course

1-411
Neurology

C. Pathophysiology
1. Epileptic seizures are the result of excessive excitation of neurons or hypersynchronization.
2. In an epileptic seizure, normal inhibitory synaptic currents and neural networks break down,
allowing neuronal excitability to spread rapidly. This occurs focally in a partial-onset seizure
or globally in a generalized seizure.
3. Clinical symptoms of a seizure depend on the site of seizure onset, amount of brain tissue irritability
and spread, and degree of the impulse.
4. There are several mechanisms by which seizures form; however, most drug therapy is targeted toward
transitory imbalances between inhibitory neurotransmitters (γ-aminobutyric-acid [GABA]), excitatory
neurotransmitters (glutamate), sodium and calcium ion channels, and neuromodulators such as
norepinephrine, serotonin, and acetylcholine (ACh).
5. Prolonged seizure activity, defined as more than 20 minutes, contributes to neuronal injury in
susceptible individuals, typically manifesting in problems with memory.

D. Clinical Presentation
1. International Classification of Epileptic Seizures – Provides a focus on a detailed description of
the individual patient

Table 2. International Classification of Epileptic Seizures


A. Partial seizures (begin focally in one hemisphere of the brain)
a. Simple (awareness or consciousness is not impaired)
i. Motor signs
ii. Somatosensory symptoms
iii. Psychiatric symptoms
iv. Autonomic symptoms
b. Complex (awareness or consciousness is impaired)
i. May begin with a simple partial seizure and then evolve
ii. May begin with awareness impaired
c. Secondary generalized (simple and complex partial seizures can evolve to a bilateral, convulsive seizure)
B. Generalized seizures (bilateral hemispheres)
a. Absence
b. Atonic
c. Clonic
d. Myoclonic
e. Tonic
f. Tonic-clonic
g. Infantile spasms
C. Seizures with unknown classifications
D. Status epilepticus
Information from: Berg AT, Berkovic SF, Brodie MJ, et al. Revised Terminology and Concepts for Organization of the Epilepsies: Report of the Commission
on Classification and Terminology. July 2009.

2. International Classification of Epilepsies and Epileptic Syndromes


a. Considers seizure type and etiologic classifications and is most important for prognosis; the syndrome
classification system requires more information but may provide better guidance in the clinic.
b. Revised terminology has been proposed but has not yet been widely adopted by practitioners.

ACCP Updates in Therapeutics® 2016: Ambulatory Care Pharmacy Preparatory Review and Recertification Course

1-412
Neurology

Table 3. The International Classification of Epilepsies and Epilepsy Syndromes


Idiopathic
a. Age-related onset
b. Clinical and/or electrographic characteristics of seizures
c. Genetic etiology
d. Underlying etiology not suspected
e. Neurologic function is normal
f. Positive family history of seizures

Symptomatic
a. Evidence of brain damage
b. Known underlying cause

Unknown or undetermined
a. No cause can be identified
Information from: Berg AT, Berkovic SF, Brodie MJ, et al. Revised Terminology and Concepts for Organization of the Epilepsies: Report of the Commission
on Classification and Terminology. July 2009.

Table 4. Revised Classification and Terminology of Seizures and Epilepsies


A. Genetic
a. Epilepsy is the direct result of a genetic disorder
b. Seizures are the core component of the symptoms of the disorder

B. Structural/metabolic
a. A structural or metabolic problem is identified in which seizures are associated
with the condition
b. Examples of conditions are stroke or infection

C. Unknown cause – The underlying cause is unknown


Special report from: Berg AT, Berkovic SF, Brodie MJ, et al. Revised terminology and concepts for organization of seizures and epilepsies: report of the ILAE
Commission on Classification and Terminology, 2005-2009. Epilepsia 2010;51:676-85.

E. Prognosis
1. Seizure freedom with AED trials
a. Thirty percent of patients will continue to have seizures with all treatment attempts.
b. Of the 70% who become seizure free, 70% will attain this status with the first AED exposure
(monotherapy) throughout 1 year.
c. Of the remaining 30% of patients who do not have response to the first AED, 13% will become
seizure free when the second AED is tried in monotherapy.
d. One percent of patients will become seizure free when the third AED is tried in monotherapy.
e. If adjunctive therapy is initiated after the first monotherapy is tried, around 26% of patients
will be seizure free.
2. Mortality – Life expectancy shortened for patients with epilepsy
a. From underlying cause of the epilepsy
i. Cerebral tumors
ii. Cerebrovascular disease

ACCP Updates in Therapeutics® 2016: Ambulatory Care Pharmacy Preparatory Review and Recertification Course

1-413
Neurology

b. Unrelated to epilepsy
i. Infection
ii. Heart disease
c. Epilepsy related
i. Suicide
ii. Seizure-related trauma – Drowning, falling from ladders, burns
iii. Status epilepticus – 20% mortality rate for each incidence
iv. Sudden unexplained or unexpected death in epilepsy (SUDEP) – Accounts for 2%–18% of
all deaths in patients with epilepsy

F. Pharmacologic Therapy
1. Choice of AED should be based on the seizure type, patient age, concurrent medical conditions,
organ function, concomitant drugs, and presumed mechanism of action of the AED.
2. First-generation or traditional AEDs (year of U.S. Food and Drug Administration [FDA] label approval)
a. Phenobarbital (1912)
b. Primidone (1938)
c. Phenytoin (1938)
d. Ethosuximide (1960)
e. Carbamazepine (1974)
f. Valproate (1978)
3. Second-generation AEDs (year of FDA label approval)
a. Felbamate (1993)
b. Lamotrigine (1993)
c. Gabapentin (1994)
d. Topiramate (1996)
e. Tiagabine (1997)
f. Oxcarbazepine (1999)
g. Levetiracetam (2000)
h. Zonisamide (2000)
i. Pregabalin (2006)
j. Lacosamide (2009)
k. Rufinamide (2009)
l. Vigabatrin (2009)
m. Ezogabine (2011)
n. Clobazam (2011)
o. Perampanel (2012)
p. Eslicarbazepine (2013)
q. Brivaracetam (2016)
4. Monitoring
a. Bone disorders
i. Recommend DXA (dual-energy x-ray absorptiometry) every 5 years
ii. Calcium (1200 mg) and vitamin D (800 international units) supplementation
b. Serum concentrations
i. Use the concentration as a guide to therapy; treat the patient, not the concentration.
ii. Trough concentration is most useful because it negates the variation in absorption.
iii. Measure concentrations when trying to determine a drug-drug interaction (DDI), if seizures
are not well controlled, if the patient is experiencing adverse effects, or if nonadherence is
suspected. It is also useful to obtain a concentration measurement when the patient is doing
well and to use this concentration as a historical control.

ACCP Updates in Therapeutics® 2016: Ambulatory Care Pharmacy Preparatory Review and Recertification Course

1-414
Neurology

iv. When obtaining a serum concentration measurement of a highly bound drug such as
phenytoin or valproic acid, it is important to order the testing of a free drug concentration or
unbound serum concentration.
v. If the patient has altered plasma protein binding (e.g., with malnutrition, pregnancy, chronic
hepatic or renal failure, burn survivors), testing of an unbound serum concentration of the
highly bound AED should be ordered.
c. Driving restrictions
i. Every state has different driving restrictions.
ii. Most states require patients to be seizure free for a specific period, confirmed by their physician.
d. Depression and suicide
i. Individuals with epilepsy have a higher risk of depression and suicide.
ii. Class labels state that AEDs may increase the risk of suicide and/or suicidal ideations.
e. Monitor for life-threatening rash
i. Stevens-Johnson syndrome
ii. Toxic epidermal necrolysis
f. Potential cross-sensitivity between agents with an aromatic ring
i. Phenytoin
ii. Carbamazepine
iii. Phenobarbital
iv. Primidone
v. Oxcarbazepine
vi. Lamotrigine
vii. Zonisamide (theoretical based on sulfonamide moiety)
viii. Lacosamide (theoretical based on aromatic ring)
ix. Rufinamide (theoretical based on aromatic ring)
x. Perampanel (theoretical based on aromatic ring)
g. Discontinuing AEDs in the seizure-free patient
i. Consider when seizure free for 2–5 years with a normal electroencephalogram (EEG)
finding while receiving treatment
ii. Single seizure type, normal IQ, normal neurologic examination findings

Table 5. Antiepileptic Drug Properties


Non-epileptic
Drug, Formulations Indications Mechanism of Action
Indications
Brivaracetam (Briviact) PS (adjunctive) Synaptic vessel protein
Tablet 10-, 25-, 50-, 75-, 100 mg; with or with- SV2A binding in the brain—
No current data
oral solution 10 mg/mL; out secondary Mechanism for seizures is
IV solution 50 mg/5 mL generalization unknown
Carbamazepine (Tegretol) CPS
Trigeminal
Chewable tablet 100 mg; tablet 200 mg; GTCS
neuralgia, Fast sodium channel blockade
oral suspension 100 mg/5 mL; XR: 100, Mixed seizure
bipolar I
200, and 300 mg patterns
Anxiety, alcohol
Clobazam (Onfi) Improves GABA’s attraction to
LGS withdrawal
5-, 10-, 20-mg tablets its receptor site
syndrome
Eslicarbazepine acetate (Aptiom) PS (adjunctive or Enhances sodium channel slow
No current data
200-, 400-, 600-, 800-mg tablets monotherapy) inactivation

ACCP Updates in Therapeutics® 2016: Ambulatory Care Pharmacy Preparatory Review and Recertification Course

1-415
Neurology

Table 5. Antiepileptic Drug Properties (continued)


Non-epileptic
Drug, Formulations Indications Mechanism of Action
Indications
Ethosuximide (Zarontin) Absence Inhibits T-type calcium
No current data
250-mg capsule; 250-mg/5-mL solution epilepsy channels
Ezogabine (Potiga) Opens voltage-gated potassium
PS (adjunctive) No current data
50-, 200-, 300-, 400-mg tablets channels
Felbamate (Felbatol) GTCS Fast sodium channel blockade;
600-mg tablets; 650-mg/5-mL PS No current data interacts with glutamate;
suspension LGS (adjunctive) enhances GABA
GTCS
Fosphenytoin (Cerebyx)
PS No current data Fast sodium channel blockade
Phenytoin equivalents 50 mg/mL
SE
Interacts with the presynaptic
voltage-gated calcium channels
Gabapentin (Neurontin) Pain, of the α-2-δ-1 subunit,
100-, 300-, 400-, 600-, and 800-mg PS (adjunctive) postherpetic modulates the release of
tablets; 250-mg/5-mL solution neuralgia excitatory neurotransmitters
(glutamate, norepinephrine,
substance P)
Lacosamide (Vimpat)
50-, 100-, 150-, 200-mg tablets; PS (adjunctive Enhances sodium channel slow
No current data
200-mg/20-mL IV solution; or monotherapy) inactivation
10-mg/mL oral solution
Lamotrigine (Lamictal)
GTCS
25-, 100-, 150-, and 200-mg Trigeminal
PS (adjunctive Fast sodium channel blockade;
tablets; 2-, 5-, 25-mg chewable; neuralgia,
or monotherapy) inhibits glutamate
25-, 50-, 100-, 200-mg ODT; bipolar I
LGS (adjunctive)
25-, 50-, 100-, and 200-mg XR
Levetiracetam (Keppra) PS (adjunc- Synaptic vessel protein
250-, 500-, 750-, and 1000-mg tive) Myoclonic SV2A binding in the brain
No current data
tablets; 100-mg/mL IV solution; 100- GTCS; primary – Mechanism for seizures is
mg/mL oral solution; 500-mg XR GTCS unknown
Oxcarbazepine (Trileptal/
Bipolar I,
Oxtellar XR) PS (adjunctive or
trigeminal Fast sodium channel blockade
150-, 300-, 600-mg tablets; 300-mg/5- monotherapy)
neuralgia
mL suspension
PS (adjunctive
for
Perampanel (Fycompa) 2-, 4-, AMPA glutamate receptor
≥12 years of age) No current data
6-, 8-, 10-, 12-mg tablets antagonist
GTCS
(adjunctive)
Phenobarbital (Luminal) GTCS
15-, 16.2-, 30-, 32.4-, 60-, PS
Insomnia Enhances GABA
64.8-, 97.2-, 100-mg tablets; 20-mg/5- SE
mL elixir Myoclonic

ACCP Updates in Therapeutics® 2016: Ambulatory Care Pharmacy Preparatory Review and Recertification Course

1-416
Neurology

Table 5. Antiepileptic Drug Properties (continued)


Non-epileptic
Drug, Formulations Indications Mechanism of Action
Indications
Phenytoin (Dilantin) GTCS
Trigeminal
30-, 100-, 200-, 300-mg XR; PS Fast sodium channel blockade
neuralgia
125-mg/5-mL suspension; 50-mg chewable SE
Diabetic Interacts with the presynaptic voltage-
peripheral gated calcium channels of the α-2-δ-1
Pregabalin (Lyrica)
neuropathic pain, subunit, modulates the release
25-, 50-, 75-, 100-, 150-, 200-, PS (adjunctive)
postherpetic of excitatory neurotransmitters
225-, 300-mg capsules
neuralgia, (glutamate, norepinephrine,
fibromyalgia substance P)
Primidone (Mysoline) GTCS
Essential tremor Enhances GABA
50-, 250-mg tablets PS
Rufinamide (Banzel) LGS (adjunctive) Prolongs sodium channel
No current data
200-, 400-mg tablets PSa inactive state
Tiagabine (Gabitril)
PS (adjunctive) No current data Enhances GABA
2-, 4-, 12-, 16-mg tablets
2 years and
older: LGS Migraine
Topiramate (Topamax, Trokendi XR,
(adjunctive) PS prophylaxis Fast sodium channel blockade;
Qudexy XR)
(adjunctive or (monotherapy), attenuates glutamate; enhances
15-, 25-mg sprinkle; 25-, 50-, 100-,
monotherapy), neuropathic pain, GABA; weak carbonic
200-mg tablets; 25-, 50-, 100-, 150-,
and GTCS essential tremor, anhydrase inhibitor
200-mg extended-release tablets
(adjunctive or weight loss
monotherapy)
Valproate (Depakene, Depakote,
Migraine
Depacon) GTCS Increases GABA; fast sodium
prophylaxis,
250-mg capsules; 250-mg/5-mL syrup; PS channel blockade; inhibits
trigeminal neu-
125-, 250-, 500- Absence T-type calcium channels
ralgia, bipolar
mg delayed release
Refractory CPS
Vigabatrin (Sabril)
(adjunctive) No current data GABA-transaminase inhibitor
500-mg powder/solution; 500-mg tablet
Infantile spasm
Fast sodium channel blockade;
Zonisamide (Zonegran)
PS (adjunctive) No current data inhibits T-type calcium channels;
25-, 50-, 100-mg capsules
carbonic anhydrase inhibitor
Non-FDA indications.
a

AMPA =α-amino-3-hydroxy-5-methyl-4-isoxazolepropionic acid; CPS = complex partial seizure; FDA = U.S. Food and Drug Administration; GABA =
γ-aminobutyric acid; GTCS = generalized tonic-clonic seizure; IV = intravenous; LGS = Lennox- Gastaut syndrome; ODT = orally disintegrating tablet; PS
= partial seizure; SE = status epilepticus; XR = extended- release.
Comparison of antiepileptic drugs. Pharmacist’s Letter/Prescriber’s Letter 2009;25:250707.
Lacy CF, Armstrong LL, Goldman MP, et al. Drug Information Handbook, 19th ed. Hudson, OH: Lexi-Comp, 2010.
Murphy JE. Clinical Pharmacokinetics, 5th ed. Bethesda, MD: American Society of Health-System Pharmacists, 2012.
Potiga [package insert]. Greenville, NC: GlaxoSmithKline, 2011.
Aptiom [package insert]. Marlborough, MA: Sunovion, 2013.
Trokendi XR [package insert]. Rockville, MD: Supernus Pharmaceuticals, 2013.
Qudexy XR [package insert]. Maple Grove, MN: Upsher-Smith Laboratories, 2014.

ACCP Updates in Therapeutics® 2016: Ambulatory Care Pharmacy Preparatory Review and Recertification Course

1-417
Neurology

Table 6. Antiepileptic Drug Dosing and Metabolism/Elimination


Typical
Maintenance Maximal Dose/ Metabolism/
Drug Initial Dose, mg Therapeutic
Dose, mg Day Elimination
Range

Hydroxylation
30% metabolism
Hydrolysis 60%
metabolism
50 to 200 mg
50 mg twice Renal elimina-
Brivaracetam divided two 200 mg Not established
daily tion (95%)
times daily
Hepatic
CYP2C19-S
<10% excreted
unchanged

Hepatic (inducer/
autoinducer)
CYP3A4-S
IR: 800–1200
CYP2C8
mg divided three
200 mg twice (minor)-S
or four times
daily (suspension CYP1A2-↑
Carbamazepine daily 1200 mg 4–12 mcg/mL
100 mg four CYP2B6-↑
XR: 800–1200
times daily) CYP2C8-↑
mg divided twice
CYP2C9-↑
daily
CYP2C19-↑
CYP3A4-↑
P-glycoprotein-↑
≤30 kg: 2.5 ≤30 kg: 10 mg Hepatic CYP3A4
mg twice daily twice daily ≤ 30 kg: 20 mg 100–300 (weak inducer)
Clobazam
>30 kg: 5 mg >30 kg: 20 mg > 30 kg: 40 mg mcg/L Highly protein
twice daily twice daily bound
Hydrolysis to
eslicarbazepine
Eslicarbazepine 400 mg once 800 mg once 1200 mg once (major active
Not established
acetate daily daily daily metabolite)
Glucuronidation
Renal (90%)
3–6 years: 250 Children: 20 mg/
Hepatic
mg once daily; kg/day
CYP3A4-S
Ethosuximide 6 years and Adults: 500– 1.5 g Not established
CYP2E1
older: 500 mg 1000 mg divided
(minor)-S
once daily twice daily
100 mg three 200–400 three Glucuronidation
Ezogabine 1200 mg Not established
times daily times daily and acetylation

ACCP Updates in Therapeutics® 2016: Ambulatory Care Pharmacy Preparatory Review and Recertification Course

1-418
Neurology

Table 6. Antiepileptic Drug Dosing and Metabolism/Elimination (continued)


Typical
Maintenance Maximal Dose/ Metabolism/
Drug Initial Dose, mg Therapeutic
Dose, mg Day Elimination
Range
50% hepatic
(inducer/
inhibitor) 50%
1200 mg divided 1200–3600 mg 3600 mg divided renal
Felbamate three or four divided three or three or four 30–60 mcg/ mL CYP3A4-S
times daily four times daily times daily CYP2E1
(minor)-S
CYP2C19-↓
CYP3A4-↑
Hepatic (inducer)
CYP2C9-S
15–20 mg
CYP2C19-S
PE/kg IV loading
CYP3A4 (minor)-S
dose, infusion up
CYP2B6-↑
Fosphenytoin to a maximal 4–6 mg PE/kg/ Individualize
10–20 mcg/mL CYP2C8-↑
(prodrug) rate of 150 mg day IV/IM dosing
CYP2C9-↑
PE/minute; this
CYP2C19-↑
can also be given
CYP3A4-↑
IM
Highly protein
bound
12 years and
older: 300 mg 900–3600 mg
12 years and
three times daily; divided three or
older: 3600 mg;
Gabapentin 3–12 years: four times daily, 2–12 mcg/mL Renal > 95%
3–12 years: 15
10–15 mg/ kg/ 3–12 years:
mg/kg/day
day divided three 10–15 mg/kg/day
times daily

17 years and
Renal (40%)
Lacosamide older: 50 mg 200–400 mg/day 400 mg Not established
CYP2C19 (30%)
twice daily
225–375 mg/day
in two divided
doses (dose –
Hepatic UDPGT
12 years and Dependent on Individualize 2.5–15
Lamotrigine (weak inducer)
older: 25 mg/day presence of other dosing mcg/mL
glucuronidation
enzyme inducers
or inhibitors
– Valproate)

IR 16 years
1000–3000 Renal (66%)
and older: 500
Levetiracetam mg divided twice 3000 mg 8–26 mcg/mL Extrahepatic
mg twice daily
daily hydrolysis (24%)
XR: 1000 mg/day

ACCP Updates in Therapeutics® 2016: Ambulatory Care Pharmacy Preparatory Review and Recertification Course

1-419
Neurology

Table 6. Antiepileptic Drug Dosing and Metabolism/Elimination (continued)


Typical
Maintenance Maximal Dose/ Metabolism/
Drug Initial Dose, mg Therapeutic
Dose, mg Day Elimination
Range
Hepatic
(moderate
inducer/inhibitor
300 mg 1200 mg divided 12.6–35 mcg/ doses greater
Oxcarbazepine 2400 mg
twice daily twice daily mL (MHD) than 1200 mg)
glucuronidation
CYP2C19-↓
CYP3A4/5-↑
CYP3A4-S
CYP3A5-S
8–12 mg once Sequential
Perampanel 2 mg once daily 12 mg once daily Not established
daily glucuronidation
Highly protein
bound
Hepatic (inducer)
CYP2C19-S

CYP2E1
(minor)-S
0.25–0.5 mg/
1–3 mg/kg/day CYP2C9
kg/day divided
Phenobarbital divided once or 200 mg 20–40 mcg/mL (minor)-S
two or three
twice daily CYP1A2-↑
times daily CYP2A6-↑
CYP2B6-↑
CYP2C8-↑
CYP2C9-↑
CYP3A4-↑
Hepatic (inducer)
CYP2C9-S
CYP2C19-S

CYP3A4
100 mg three
(minor)-S
times daily; 5–7 mg/kg/day
Individualize CYP2B6-↑
Phenytoin phenytoin can divided one to 10–20 mcg/mL
dosing CYP2C8-↑
be given as a three times daily
CYP2C9-↑
loading dose
CYP2C19-↑
CYP3A4-↑

Highly protein
bound

ACCP Updates in Therapeutics® 2016: Ambulatory Care Pharmacy Preparatory Review and Recertification Course

1-420
Neurology

Table 6. Antiepileptic Drug Dosing and Metabolism/Elimination (continued)


Typical
Maintenance Maximal Dose/ Metabolism/
Drug Initial Dose, mg Therapeutic
Dose, mg Day Elimination
Range
150 mg divided
Individualize
Pregabalin two or three 600 mg Not established Renal 90%
dosing
times daily
Hepatic (inducer)
– Phenobarbital
component)
CYP2C19-S
CYP2E1
(minor)-S
100–125 mg 250 mg three or
Primidone 2000 mg Not established CYP2C9
at night four times daily
(minor)-S
CYP1A2-↑
CYP2B6-↑
CYP2C8-↑
CYP2C9-↑
CYP3A4-↑
Enzymatic
hydrolysis of
400–800 its carboxamide
3200 mg divided 3200 mg divided
Rufinamide mg divided twice Not established group CYP3A4
twice daily twice daily
daily (weak inducer)
CYP2E1 (weak
inhibitor)
Hepatic
Individualize
56 mg divided CYP3A4-S
Tiagabine 4 mg once daily dosing up to 56 Not established
four times daily Highly protein
mg/day
bound
Hepatic (inducer/
IR: 25–50 IR: 100–400
IR: 1600 mg inhibitor in
mg divided twice mg divided twice
doses greater
daily daily
Topiramate 5–20 mcg/mL than 200 mg)
Glucuronidation
XR: 25–50 mg XR: 200–400
XR: 400 mg CYP2C19-↓
once daily mg once daily
CYP3A4-↑
↑ = inducer; ↓ = inhibitor.
CYP = cytochrome P450; IM = intramuscularly; IR = immediate release; IV = intravenous(ly); MHD = monohydroxy derivative; PE = phenytoin equivalents;
S = substrate; UDPGT = uridine diphosphate glucuronyl transferase; XR = extended release.
Comparison of antiepileptic drugs. Pharmacist’s Letter/Prescriber’s Letter 2009;25:250707.
Lacy CF, Armstrong LL, Goldman MP, et al. Drug Information Handbook, 19th ed. Hudson, OH: Lexi-Comp, 2010.
Murphy JE. Clinical Pharmacokinetics, 5th ed. Bethesda, MD: American Society of Health-System Pharmacists, 2012.
Potiga [package insert]. Greenville, NC: GlaxoSmithKline, 2011.
Aptiom [package insert]. Marlborough, MA: Sunovion, 2013.
Qudexy XR [package insert]. Maple Grove, MN: Upsher-Smith Laboratories, 2014.
Trokendi XR [package insert]. Rockville, MD: Supernus Pharmaceuticals, 2013.

ACCP Updates in Therapeutics® 2016: Ambulatory Care Pharmacy Preparatory Review and Recertification Course

1-421
Neurology

Table 7. AED Interactions, Pregnancy Category, and Drug-Specific Adverse Eventsa


Drug-Specific Adverse Events
Interaction with Pregnancy
Drug (can be concentration-dependent or idiosyncratic/
HBC Categoryb
less common*)
Potential to decrease
Brivaracetam efficacy of HBC but C Dizziness, sedation, somnolence, fatigue, psychosis
unknown at this time
Tremor, myoclonus, cardiotoxicity, sexual dysfunction,
visual distortion, diplopia, risk of osteopenia/
Decreases efficacy of
Carbamazepine D osteoporosis, life-threatening rash,*c hyponatremia,*
HBC
leukopenia, and aplastic anemia,* dizziness (can be seen
if dose is too high)
Decreases efficacy of
Clobazam C Somnolence, lethargy, fever, aggressive behavior
HBC
Dizziness, somnolence, nausea, diplopia, headache,
Eslicarbazepine Decreases efficacy of
C vomiting, abnormal coordination, blurred vision,
acetate HBC
vertigo, fatigue
Ethosuximide None known C Nightmares, sedation
Dizziness, somnolence, fatigue, confusion, vertigo,
tremor, diplopia, attention/memory impairment, urinary
Ezogabine None known C
retention,* QT prolongation,* skin discoloration,*
retinal abnormalities
Decreases efficacy of Insomnia, weight loss, HA, nausea, aplastic anemia,*
Felbamate C
HBC hepatotoxicity,* hepatic failure*
Decreases efficacy of
Fosphenytoin D Same as phenytoin
HBC
Gabapentin None known C Myoclonus, pedal edema, weight gain,* irritability
Dizziness, HA, nausea, diplopia, PR-interval
Lacosamide None known C
increase (minimal),* life-threatening rash*
HBC decreases the
efficacy of lamotrigine
and lamotrigine Dose- and titration-dependent rash and life-threatening
Lamotrigine C
decreases the efficacy rash,* visual distortion, dizziness, headache
of progesterone-only
OC
Levetiracetam None known C Sedation, behavioral changes, depression,* aggression*
Decreases efficacy of Hyponatremia,* dizziness, visual distortion, risk of
Oxcarbazepine C
HBC at higher doses osteopenia/osteoporosis, diplopia, life-threatening rash*
Abnormal gait, dizziness, headache,
Decreases efficacy of
Perampanel C somnolence, irritability, fatigue, mood disorder,*
HBC at higher doses
aggression, anger, homicidal ideation*
Connective tissue disorder, erectile dysfunction,
Decreases efficacy of
Phenobarbital D sedation, risk of osteopenia/osteoporosis, cognitive
HBC
impairment,* hyperactivity,* life-threatening rash*

ACCP Updates in Therapeutics® 2016: Ambulatory Care Pharmacy Preparatory Review and Recertification Course

1-422
Neurology

Table 7. AED Interactions, Pregnancy Category, and Drug-Specific Adverse Eventsa (continued)
Drug-Specific Adverse Events
Interaction with Pregnancy
Drug (can be concentration-dependent or idiosyncratic/
HBC Categoryb
less common*)
Ataxia, gingival hyperplasia,* nystagmus, risk of
Decreases efficacy of osteopenia/osteoporosis, dizziness, sedation, rash,
Phenytoin D
HBC systemic lupus erythematosus–like syndrome,* life-
threatening rash*
Pregabalin None known C Same as gabapentin but potentially worse
Decreases efficacy of
Primidone D Same as phenobarbital
HBC
Decreases efficacy of
Rufinamide C Shortened QT interval,* HA, somnolence, rash
HBC
Encephalopathy, knee-buckling, status epilepticus on
Tiagabine None known C
abrupt withdrawal*
Decreases efficacy Renal stones,* word-finding difficulties, paresthesia,*
Topiramate of ethinyl estradiol at D weight loss,* glaucoma,* metabolic acidosis,*
higher doses oligohidrosis*
HBC may decrease the D Tremor, encephalopathy,* pedal edema, hair loss,
serum concentrations (epilepsy); weight gain, pancreatitis,* hepatotoxicity (in patients
Valproic acid
of valproic acid by X younger than 2 years),* parkinsonism and middle-ear
about 20% (migraine) dysfunction (elderly patients)
Irreversible visual field defects,*
Vigabatrin None known C
drowsiness, fatigue, hyperactivity
Renal stones,* paresthesia,* weight loss,* metabolic
Zonisamide None known C acidosis,* oligohidrosis,* psychosis,* rash, life-
threatening rash*
a
Many AEDs can cause blood dyscrasias (decreased white blood cell count, decreased platelet count, decreased red blood cell count), non–life- threatening
rash, nausea, vomiting, dizziness, ataxia, increased liver function tests, hepatotoxicity, psychiatric comorbidities, and sedation to varying degrees.
b
Pregnancy category C: Risk cannot be ruled out because of inadequate, well-controlled human studies. Animal studies have shown a risk to the fetus.
Consideration of risks and benefits of therapy should be assessed before beginning therapy. Pregnancy category D: Positive evidence of human fetal risk.
Information from: FDA Drug Category Ratings. American Pregnancy Association. Last updated June 2006. Available at www. americanpregnancy.org/
pregnancyhealth/fdadrugratings.html. Accessed December 10, 2014.
Genetic testing for HLA B*1502 recommended for those of Asian ancestry.
c

AED = antiepileptic drug; HA = headache; HBC = hormone-based contraception; OC = oral contraceptive.


Lacy CF, Armstrong LL, Goldman MP, et al. Drug Information Handbook, 19th ed. Hudson, OH: Lexi-Comp, 2010.
Lamotrigine [package insert]. Greenville, NC: GlaxoSmithKline, 2010.
Potiga [package insert]. Greenville, NC: GlaxoSmithKline, 2011.
Aptiom [package insert]. Marlborough, MA: Sunovion, 2013.
Briviact. Micromedex, 2016.

ACCP Updates in Therapeutics® 2016: Ambulatory Care Pharmacy Preparatory Review and Recertification Course

1-423
Neurology

Table 8. Notable Combination Antiepileptic Drug-Drug Interactions (not exhaustive)a


Antiepileptic Severity of Medication Concentration Medication Concentration
Added Agent
Drug Interaction Decreased Increased
Brivaracetam Phenytoin Moderate Brivaracetam Phenytoin
Levetiracetam Minor
Rifampin Severe Brivaracetam
Carbamazepine Moderate Brivaracetam Carbamazepine metabolite
Carbamazepine Phenytoin Moderate Carbamazepine Phenytoin
Felbamate Moderate Felbamate, carbamazepine Carbamazepine metabolite
Phenobarbital Moderate Carbamazepine
Valproic acid Moderate Valproic acid Carbamazepine metabolite
Ethosuximide Moderate Ethosuximide
Lamotrigine Moderate Lamotrigine
Levetiracetam Moderate Carbamazepine
Oxcarbazepine Moderate Oxcarbazepine metabolite
Primidone Moderate Carbamazepine
Topiramate Moderate Topiramate
Tiagabine Moderate Tiagabine
Phenytoin Carbamazepine Moderate Carbamazepine Phenytoin
Vigabatrin Moderate Phenytoin –
Oxcarbazepine Moderate – Phenytoin
Felbamate Moderate Felbamate Phenytoin
Topiramate Moderate Topiramate Phenytoin
Valproic acid Moderate Valproic acid Phenytoin
Methsuximide Moderate Phenytoin Phenytoin
Felbamate Moderate Felbamate Phenytoin
Eslicarbazepine Carbamazepine Moderate Eslicarbazepine
Phenobarbital Moderate Eslicarbazepine
Phenytoin Moderate Eslicarbazepine Phenytoin
Ethosuximide Phenobarbital Moderate Ethosuximide
Phenytoin Moderate Ethosuximide
Carbamazepine Moderate Ethosuximide
Ezogabine Carbamazepine Moderate Ezogabine
Phenytoin Moderate Ezogabine
Felbamate Carbamazepine Moderate Felbamate, carbamazepine Carbamazepine metabolite
Phenytoin Moderate Felbamate Phenytoin
Valproic acid Moderate Valproic acid
Phenobarbital Minor Phenobarbital
Lamotrigineb Valproic acid Major Lamotrigine
Carbamazepine Moderate Lamotrigine
Oxcarbazepine Moderate Lamotrigine
Phenobarbital Moderate Lamotrigine
Phenytoin Moderate Lamotrigine
Primidone Moderate Lamotrigine

ACCP Updates in Therapeutics® 2016: Ambulatory Care Pharmacy Preparatory Review and Recertification Course

1-424
Neurology

Table 8. Notable Combination Antiepileptic Drug-Drug Interactions (not exhaustive)a (continued)


Antiepileptic Severity of Medication Concentration Medication Concentration
Added Agent
Drug Interaction Decreased Increased
Oxcarbazepine Carbamazepine Moderate Oxcarbazepine metabolite
Lamotrigine Moderate Lamotrigine
Phenobarbital Moderate Oxcarbazepine metabolite
Phenytoin Moderate Oxcarbazepine metabolite
Valproic acid Moderate Oxcarbazepine metabolite
Phenobarbitalc Primidone Major Phenobarbital
Carbamazepine Moderate Carbamazepine
Ethosuximide Moderate Ethosuximide
Lamotrigine Moderate Lamotrigine
Oxcarbazepine Moderate Oxcarbazepine metabolite
Tiagabine Moderate Tiagabine
Topiramate Moderate Topiramate
Valproic acid Moderate Valproic acid Phenobarbital
Phenytoin Minor Phenobarbital Phenobarbital
Primidone Phenobarbital Major Phenobarbital
Carbamazepine Moderate Carbamazepine
Lamotrigine Moderate Lamotrigine
Tiagabine Moderate Tiagabine
Valproic acid Major Primidone, phenobarbital
Tiagabine Carbamazepine Moderate Tiagabine
Phenobarbital Moderate Tiagabine
Phenytoin Moderate Tiagabine
Primidone Moderate Tiagabine
Topiramate Carbamazepine Moderate Topiramate
Phenobarbital Moderate Topiramate
Phenytoin Moderate Topiramate
Valproic acid Moderate Valproic acid, topiramate
Valproic acidd Lamotrigine Major Lamotrigine
Primidone Major Primidone, phenobarbital
Carbamazepine Moderate Valproic acid Carbamazepine metabolite
Ethosuximide Moderate Ethosuximide
Felbamate Moderate Valproic acid
Oxcarbazepine Moderate Oxcarbazepine active
metabolite
Phenobarbital Moderate Valproic acid Phenobarbital
Phenytoin Moderate Valproic acid, phenytoin
Topiramate Moderate Topiramate, valproic acid
Zonisamide Carbamazepine Moderate Zonisamide
Phenobarbital Moderate Zonisamide
Phenytoin Moderate Zonisamide

ACCP Updates in Therapeutics® 2016: Ambulatory Care Pharmacy Preparatory Review and Recertification Course

1-425
Neurology

Table 8. Notable Combination Antiepileptic Drug-Drug Interactions (not exhaustive)a (continued)


Antiepileptic Severity of Medication Concentration Medication Concentration
Added Agent
Drug Interaction Decreased Increased
Levetiracetam Carbamazepine Moderate Carbamazepine
Vigabatrine Carbamazepine Major Carbamazepine
Phenytoin Moderate Phenytoin
Lacosamide f
All traditional Varies
sodium channel
blocking drugs
Rufinamide Valproic acid Moderate Rufinamide
Phenobarbital Moderate Rufinamide Phenobarbital
Carbamazepine Moderate Rufinamide, carbamazepine
Phenytoin Moderate Rufinamide Phenytoin
Primidone Moderate Rufinamide
Lamotrigine Moderate Lamotrigine
Perampanel Carbamazepine Moderate Perampanel
Phenytoin Moderate Perampanel
Oxcarbazepine Moderate Perampanel
a
Management of major interactions: Monitor the patient for clinical signs and symptoms of toxicity – This should be done for every patient individually.
b
Management of valproic acid interaction with lamotrigine – Dosage reductions of lamotrigine.
c
Management of phenobarbital interaction with primidone – Monitor for respiratory depression; dose reduction in one or both.
d
Management of valproic acid interaction with primidone – Monitored for excessive CNS depression and neurologic toxicity.
e
Management of vigabatrin interaction with carbamazepine – Concentration of carbamazepine should be monitored and the dosage adjusted accordingly.
f
Management of lacosamide interaction with traditional sodium channel blockers – Patients experiencing dizziness or gastrointestinal
disturbances – Lower the dose of the traditional sodium channel–blocking antiepileptic drug.
CNS = central nervous system.
Micromedex Healthcare Series [Internet version]. Greenwood Village, CO: Thomson Micromedex. Available at http://0-www.thomsonhc. com.library.uchsc.
edu:80. Accessed November 2, 2010.
Lacy CF, Armstrong LL, Goldman MP, et al. Drug Information Handbook, 19th ed. Hudson, OH: Lexi-Comp, 2010.
Potiga [package insert]. Greenville, NC: GlaxoSmithKline, 2011.
Adapted from: Rogers SJ, Cavazos JE. Chapter 40: Epilepsy. In: DiPiro JT, Talbert RL, Yee GC, et al., eds. Pharmacotherapy: A Pathophysiologic Approach,
9th ed. New York, NY: McGraw-Hill, 2014. Available at http://accesspharmacy.mhmedical.com.hsl-ezproxy.ucdenver.edu/content.aspx?bookid=689&Sec
tionid=45310490. Accessed March 2, 2015.

ACCP Updates in Therapeutics® 2016: Ambulatory Care Pharmacy Preparatory Review and Recertification Course

1-426
Neurology

Table 9. First-line Options According to Epilepsy Guidelines 2004a


Partial Seizures Partial Seizures Partial Seizures Generalized Absence
Drug
(newly diagnosed) (refractory) (refractory adjunctive) Seizures Seizures
Carbamazepine P
Ethosuximide P
Gabapentin P P
Lamotrigine P P P P P
Levetiracetam P
Oxcarbazepine P P
Phenobarbital P
Tiagabine P
Topiramate P P P P
Valproate b
P
Zonisamide P
The guidelines were developed in 2004 before many drugs were FDA label approved.
a

b
Not first-line therapy for women of childbearing age.
French JA, Kanner AM, Bautista J, et al. Efficacy and tolerability of the new antiepileptic drugs, I: treatment of new onset epilepsy: report of the Therapeutics
and Technology Assessment Subcommittee and Quality Standards Subcommittee of the American Academy of Neurology and the American Epilepsy
Society. Neurology2004;62:1252-60.

Table 10. Newer Agents Generally Used as First-line Therapy


Drug Partial Seizures Infantile Spasms Lennox-Gastaut Syndrome
Lacosamide P
Rufinamide P
Vigabatrin P
Eslicarbazepine P

ACCP Updates in Therapeutics® 2016: Ambulatory Care Pharmacy Preparatory Review and Recertification Course

1-427
Neurology

Table 11. Treatment of Generalized Convulsive Status Epilepticus


Initial Therapy for Status Epilepticus
Adults Pediatric Population
Anticonvulsant Maintenance
Loading Dose Maintenance Dose Loading Dose
(IV) Dose
Diazepam
0.25 mg/kg
(rectal form Not used 0.25–0.5 mg/kg Not used
Elderly: 2–5 mg
available)
Lorazepam
4 mg
(can be given Not used 0.1 mg/kg Not used
Elderly: 2–5 mg
rectally)
Midazolam
(can be given 200 mcg/kg 50–500 mcg/kg/hr 150 mcg/kg 60–120 mcg/kg/hr
intramuscularly)
Second-Line Treatments
Phenytoina 18–20 mg/kg 4–5 mg/kg/day 10–20 mg/kg 5–10 mg/kg/day
Fosphenytoin b
18–20 mg PE/kg 4–5 mg PE/kg/day 15–20 mg PE/kg 5–10 mg PE/kg/day
Phenobarbital 10–20 mg/kg 1–4 mg/kg/day 15–20 mg/kg 3–5 mg/kg/day
Medications That May Be Useful for Nonresponsive or Refractory Convulsive Status Epilepticus
Levetiracetam 500–2000 mg 750–9000 mg/day 15–70 mg/kg Not used
1 mg/kg (maximal
Lidocaine 50–100 mg 1.5–3.5 mg/kg/hr dose: 3–5 mg/ 1.2–3 mg/kg/hr
kg in first hour)
Midazolam 200 mcg/kgc 50–500 mcg/kg/hr 150 mcg/kg 60–120 mcg/kg/hr
Pentobarbital 10–20 mg/kg 1–5 mg/kg/hr 15–20 mg/kg 1–5 mg/kg/hr
Propofol 2 mg/kg 5–10 mg/kg/hr 3 mg/kg 2–18 mg/kg/hr
Topiramate
300–1600 mg 400–1600 mg/day 5–10 mg/kg 5–10 mg/kg/day
(nasogastric tube)
Valproate 15–45 mg/kg 1–4 mg/kg/hr 20–25 mg/kg 1–4 mg/kg/hr
Lacosamide 200–300 mg 200–400 mg/day – –
NTE 50 mg/minute; dilute only in NS; use a final filter.
a

b
NTE 150 mg PE/minute, can dilute in D W, NS, LR, less tissue necrosis and hypotension than phenytoin, no final filter needed.
Can be administered intramuscularly.
c

D W = 5% dextrose; hr = hour; IV = intravenous; LR = lactated Ringer (solution); NS = normal saline; NTE = not to exceed; PE = phenytoin equivalents.
Albers JM, Moddel G, Dittrich R, et al. Intravenous lacosamide – an effective add-on treatment of refractory status epilepticus. Seizure 2011;20:428-30.
Goodwin H, Hinson HE, Shermock KM, et al. The use of lacosamide in refractory status epilepticus. Neurocrit Care Soc 2011;14:348-53.
Lacy CF, Armstrong LL, Goldman MP, et al. Drug Information Handbook, 19th ed. Hudson, OH: Lexi-Comp, 2010.
Adapted from: Phelps SJ, Wheless JW. Chapter 41: Status epilepticus. In: DiPiro JT, Talbert RL, Yee GC, et al., eds. Pharmacotherapy: A Pathophysiologic
Approach, 9th ed. New York, NY: McGraw-Hill, 2014. Available at http://accesspharmacy.mhmedical.com.hsl-ezproxy.ucdenver.edu/content.aspx?bookid
=689&Sectionid=45310491. Accessed March 2, 2015.

ACCP Updates in Therapeutics® 2016: Ambulatory Care Pharmacy Preparatory Review and Recertification Course

1-428
Neurology

Initial therapy or first-line treatment (1–10 minutes)


Slow intravenous (IV) push or IV drip
Intravenous lorazepam 4 mg (elderly: 2–5 mg; pediatric: 0.03–0.1 mg/kg
Alternatives: Intravenous midazolam 200 mcg/kg (pediatric: 150 mcg/kg)
Intravenous/rectal diazepam 0.25 mg/kg (elderly: 2–5 mg; pediatric: 0.25–0.5 mg/kg)
May repeat in 5 minutes if no response

No response or second-line treatment (10–30 minutes)


Intravenous fosphenytoin 15–20 mg phenytoin sodium equivalent (PE)/kg (pediatric: 15–20 mg PE/kg)
(not to exceed [NTE] 150 mg/minute, normal saline [NS], D5W[5% dextrose] LR [lactated Ringer’s]) or IV
phenytoin 10–20 mg/kg (pediatric: 10–20 mg/kg) (NTE 50 mg/minute, NS only, use final filter)

Established status or third-line treatment (30–60 minutes)


1) Additional intravenous 5-mg/kg dose of either phenytoin or fosphenytoin PE or
2) Intravenous phenobarbital 10–20 mg/kg (pediatric: 15–20 mg/kg) or agents listed belowa

Refractory Status (greater than 60 minutes)


1) Additional IV phenobarbital 10 mg/kg every hour until seizures cease or
2) Intravenous valproate 15–45 mg/kg (pediatric: 20–25 mg/kg), followed by 1–4 mg/kg/hour
or
General anesthesia with one of the following three:
1) Intravenous midazolam 200 mcg/kg (pediatric: 150 mcg/kg) bolus, followed by 50–500 mcg/kg/hour
(pediatric: 60–120 mcg/kg/hour)
2) Intravenous pentobarbital 10–20 mg/kg (pediatric: 15–20 mg/kg) bolus over 1 hour; then 1–5 mg/kg/hour to
burst suppression on EEG (electroencephalogram) or
3) Intravenous propofol 2 mg/kg (pediatric: 3 mg/kg) bolus, followed by ≤4 mg/kg/hour
a
Neither levetiracetam nor lacosamide has sufficient date to support its use, or there are limited data available, but they are used clinically.

Figure 1. Treatment algorithm for status epilepticus.


Adapted from: Phelps SJ, Wheless JW. Chapter 41: Status epilepticus. In: DiPiro JT, Talbert RL, Yee GC, et al., eds. Pharmacotherapy: A Pathophysiologic
Approach, 9th ed. New York, NY: McGraw-Hill, 2014. Available at http://accesspharmacy.mhmedical.com.hsl-ezproxy.ucdenver.edu/content.aspx?bookid
=689&Sectionid=45310491. Accessed March 2, 2015.

ACCP Updates in Therapeutics® 2016: Ambulatory Care Pharmacy Preparatory Review and Recertification Course

1-429
Neurology

Patient Cases

Questions 1 and 2 pertain to the following case.


K.L. is a 65-year-old man with a new diagnosis of complex partial seizures. An EEG was performed that shows
epileptiform abnormalities, confirming a diagnosis of epilepsy. K.L. also has a history of diabetes mellitus and
associated peripheral neuropathy. His renal function is stable. K.L.’s physician would like a recommendation for
an AED.

1. Given his comorbidities, which drug would be most appropriate for K.L.?
A. Felbamate.
B. Gabapentin.
C. Lamotrigine.
D. Phenytoin.

2. As K.L.’s diabetes progresses, his renal function becomes severely compromised, and his physician would
like to avoid renally eliminated AEDs. Given this information, which drug would be best to treat his epilepsy?
A. Topiramate.
B. Lamotrigine.
C. Pregabalin.
D. Vigabatrin.

3. T.H. is a 70-year-old man with a longstanding history of generalized tonic-clonic seizures. He presents to the
clinic today for a follow-up visit after his routine serum laboratory values were obtained 4 weeks ago. His
liver enzyme test values were about 10 times the upper limit of normal. The physician ordered a CT scan
and liver biopsy. T.H. was given a diagnosis of severe liver disease and referred back to neurology for review
of his AEDs. He is currently taking phenobarbital and valproate. Given his new diagnosis of liver disease,
which is the best recommendation for treatment?
A. Continue phenobarbital and valproate; no change is needed.
B. Continue phenobarbital and replace valproate with levetiracetam.
C. Replace phenobarbital with levetiracetam and continue valproate.
D. Replace both phenobarbital and valproate with levetiracetam and pregabalin.

4. J.D., a 68-year-old man, was seen in your clinic today; on leaving, he began to have several seizures lasting
more than 10 minutes. During this episode, he did not regain consciousness at any point. He was taken
to the ED, and the physician there wished to begin intravenous phenytoin. She calls you, asking about
drug information regarding intravenous phenytoin. Which would be best to communicate to the physician
regarding the most appropriated dosing recommendation for this patient?
A. Phenytoin can be given intramuscularly.
B. The infusion rate of intravenous phenytoin cannot exceed 50 mg/minute.
C. Intravenous phenytoin should be diluted before administering to the patient.
D. Intravenous phenytoin can cause tissue necrosis on extravasation.

G. Nonpharmacologic Therapy
1. Ketogenic diet
a. High-fat content, very low carbohydrate, shifts the brain’s metabolism to ketones
b. Best adherence with children

ACCP Updates in Therapeutics® 2016: Ambulatory Care Pharmacy Preparatory Review and Recertification Course

1-430
Neurology

2. Vagus nerve stimulation


3. Resective surgery – Up to 80% effective, depending on the location
4. Responsive neurostimulator system

H. Special Populations
1. Pregnancy (to minimize the risk of teratogenicity)
a. Have a pregnancy or contraceptive plan before AED initiation.
b. Careful initial AED selection before contraception
c. Reduce seizure frequency, seizures during pregnancy are harmful to the fetus and the mother.
d. Monotherapy is recommended, if possible.
e. Avoid the high peak and low trough concentrations of AEDs.
f. Folate supplementation – No less than 0.4 mg/day for women of childbearing age with epilepsy
(American Academy of Neurology [AAN]). Pregnant women with epilepsy are considered high
risk; the general clinical recommendation is to supplement with 3–4 mg/day (American Congress
of Obstetricians and Gynecologists [ACOG]).
g. Minimize valproate owing to its highest incidence of birth defects; this may not be possible, so the
lowest dose is best.
h. It is essential that the treatment of each woman with epilepsy be individualized, assessing the risk-
benefit of seizure control and potential teratogenicity of AEDs.
i. Breastfeeding is usually acceptable.
i. Amount of AED in breast milk depends on the protein binding of the drug.
ii. The fetus is exposed to higher AED concentrations in utero than through breast milk.
2. Elderly
a. Consider concomitant medications.
b. Monitor organ function.
c. Monitor for adverse events that could increase fall risk.
3. Solid-organ transplantation
a. Avoid CYP (cytochrome P450) 3A4 enzyme inducers with cyclosporine, if possible.
b. Closely monitor drug interactions with other immunosuppressants.
4. Human immunodeficiency virus (HIV)
a. Seizure disorders are common in patients with HIV, up to 11%.
b. Potential interactions between antiretroviral (ARV) agents and AEDs that affect the CYP system
are common and extensive.
c. Older AEDs may induce the metabolism of ARVs, including the protease inhibitors, integrase
inhibitors, maraviroc, and nonnucleoside reverse transcriptase inhibitors, thus lowering their
effectiveness.
d. Similarly, ARVs may alter the serum concentrations of AEDs.
e. Some patients with HIV may be coinfected with Mycobacterium tuberculosis or hepatitis C virus
(chronic infection), which adds complexity to DDIs.

I. Patient Education
1. Adherence is very important.
2. The goal of AED therapy is individual to the patient, but in general, it is to stop the patient’s seizures
with no drug adverse effects. However, this may not be a reasonable goal for all patients.
3. AED therapy is lifelong for most patients, depending on their circumstances.
4. AEDs can be withdrawn under strict guidance by the practitioner when the patient is seizure/aura free
for 2–5 years with a normal EEG finding, single seizure type, and normal IQ.
5. Abruptly withdrawing AEDs without practitioner guidance may place the patient at harm of having a
rebound seizure or status epilepticus.

ACCP Updates in Therapeutics® 2016: Ambulatory Care Pharmacy Preparatory Review and Recertification Course

1-431
Neurology

6. If the patient is pregnant or trying to conceive, she should contact her practitioner.
7. Self-monitoring of seizures and drug adverse effects should be assessed by the patient and next-of-kin.
8. Diary for documenting witnessed seizures and drug adverse effects
a. Many of the AEDs cause sedation and can be additive.
b. All AEDs can cause adverse effects, some expected and some specific to the patient – They should
be managed by the practitioner, not the patient.
c. Dermatologic effects – Rashes (e.g., macular papillary) – Call practitioner immediately.
d. Dermatologic effects – Call practitioner immediately – Monitor for life-threatening rash (e.g.,
sometimes may resemble a burn or skin ulceration), which can be located on mucus membranes with
skin lesions (e.g., systemic involvement is generally present – Nausea, vomiting, fever, diarrhea).
e. Mood changes need to be reported to the practitioner immediately, especially depression and
suicidal thoughts.
9. Lifestyle
a. No baths if uncontrolled seizures
b. No swimming
c. No driving if active seizures – There are specific rules in most states about driving.
d. Be careful of stoves/burns/curling irons.
e. No ladders
10. Seizure first aid
a. Never put something in a seizing person’s mouth.
b. Remove items that may cause injury (e.g., chairs, tables).
c. Do not try to restrain the patient.
d. Turn the patient on his or her side once the convulsions stop.
e. Check the patient for injuries.
11. Patient additional information
a. www.aesnet.org
b. www.epilepsyfoundation.org
c. www.epilepsy.com
d. www.drugstore.com
e. www.medWatch.com

J. Practitioner Education
1. www.aesnet.org
2. www.epilepsyfoundation.org
3. AEDs have several DDIs, so it is essential that patients’ over-the-counter and prescription drugs be
screened on a regular basis to avert and manage these interactions.
a. Some notorious DDIs are between the enzyme-inducing AEDs (e.g., phenobarbital, phenytoin,
carbamazepine, and, to a lesser extent, topiramate and oxcarbazepine at higher doses – Not an
exhaustive list) and hormonal contraception, HIV, tuberculosis, solid-organ transplant drugs,
cholesterol-lowering agents, drugs used for HTN, warfarin, and so forth.
b. Drug interactions can also occur with enzyme-inhibiting drugs such as valproic acid – Serum
concentrations of other drugs will be increased.
c. Drug interactions can occur because of protein-binding displacement (e.g., two highly protein-
bound drugs – Phenytoin and valproic acid).
d. Drug interactions can occur in the uridine diphosphate glucuronyltransferase system as well (e.g.,
valproic acid).
e. Drug interactions may occur in the P-glycoprotein system (e.g., efflux pump).
4. It is essential that unexpected adverse reactions be reported on a MedWatch form to the FDA.

ACCP Updates in Therapeutics® 2016: Ambulatory Care Pharmacy Preparatory Review and Recertification Course

1-432
Neurology

5. Adverse events – Monitor for life-threatening rash.


a. Stevens-Johnson syndrome
b. Toxic epidermal necrolysis
c. Potential cross-sensitivity between agents with an aromatic ring
i. Phenytoin
ii. Carbamazepine
iii. Phenobarbital
iv. Primidone
v. Oxcarbazepine
vi. Lamotrigine (theoretical based on aromatic ring)
vii. Zonisamide (theoretical based on sulfonamide moiety)
viii. Lacosamide (theoretical based on aromatic ring)
ix. Rufinamide (theoretical based on aromatic ring)
x. Eslicarbazepine (theoretical based on aromatic ring)
6. Generic-to-generic substitution may be a problem with some patients with epilepsy – This should
be monitored, and patients should have access to brand-name products, if needed.
7. Pharmacists are available to both patients and practitioners to manage their medication therapy
and assist with problem solving with the disorder of epilepsy.

Table 12. Cost of Antiepileptic Drugs


Brand Generic Dosage Formulations Average Wholesale Price, $ (Mfr)
100-mg chew tablet 63.93/100 tablets (Major)
200-mg tablet 111.94/100 tablets (Major)
Tegretol/
Carbamazepine 100-mg/5-mL suspension 88.59/450 mL (Taro)
Carbatrol
200-mg XR tablet 207.86/100 tablets (Taro)
400-mg XR tablet 415.40/100 tablets (Taro)
2.5-mg/mL 832.04/120 mL (Lundbeck)
Onfi Clobazam 10-mg tablet 1526.65/100 tablets (Lundbeck)
20-mg tablet 3053.27/100 tablets (Lundbeck)
250-mg softgel capsule 223.14/100 tablets (Akorn)
Zarontin Ethosuximide
250-mg/5-mL solution 234.75/474 mL (Akorn)
400-mg tablet 516.82/100 tablets (Wallace)
Felbatol Felbamate 600-mg tablet 592.31/100 tablets (Wallace)
600-mg/5-mL suspension 635.18/237 mL (Wallace)
IV PE 50 mg/mL 82.50/10X10 mL (West-ward)
Cerebyx Fosphenytoin
Inj PE 75 mg/mL 149.52/10X10 mL (Fresenius Kabi)
100-mg capsule 53.18/100 capsules (Actavis)
300-mg capsule 132.96/100 capsules (Actavis)
400-mg capsule 159.53/100 capsules (Actavis)
Neurontin Gabapentin
600-mg tablet 252.62/100 capsules (Actavis)
800-mg tablet 301.10/100 capsules (Actavis)
250-mg/5-mL solution 1448.91/470 mL (Akorn)
50-mg tablet 479.76/60 tablets (UCB)
100-mg tablet 750.06/60 tablets (UCB)
Vimpat Lacosamide 150-mg tablet 794.36/60 tablets (UCB)
200-mg tablet 794.59/60 tablets (UCB)
10-mg/mL IV solution 656.28/10X20 mL (UCB)

ACCP Updates in Therapeutics® 2016: Ambulatory Care Pharmacy Preparatory Review and Recertification Course

1-433
Neurology

Table 12. Cost of Antiepileptic Drugs (continued)


Brand Generic Dosage Formulations Average Wholesale Price, $ (Mfr)
25-mg tablet 1125.25/100 tablets (GSK)
100-mg tablet 1285.32/100 tablets (GSK)
150-mg tablet 845.24/60 tablets (GSK)
200-mg tablet 920.15/60 tablets (GSK)
5-mg chew tablet 1088.81/100 tablets (GSK)
25-mg chew tablet 1168.86/100 tablets (GSK)
25-mg ODT 328.02/30 ODT (GSK)
Lamictal Lamotrigine
50-mg ODT 351.34/30 ODT (GSK)
100-mg ODT 374.63/30 ODT (GSK)
200-mg ODT 447.04/30 ODT (GSK)
25-mg XR tablet 298.20/30 tablets (GSK)
50-mg XR tablet 596.35/30 tablets (GSK)
100-mg XR tablet 638.76/30 tablets (GSK)
200-mg XR tablet 681.17/30 tablets (GSK)
250-mg tablet 287.53/100 tablets (Mylan)
500-mg tablet 351.43/100 tablets (Mylan)
750-mg tablet 476.09/100 tablets (Mylan)
Keppra Levetiracetam 1000-mg tablet 398.57/50 tablets (Mylan)
100-mg/mL solution 302.92/473 mL (Lupin)
500-mg XR tablet 266.82/60 tablets (Actavis)
750-mg XR tablet 400.65/60 tablets (Actavis)
150-mg tablet 391.36/100 tablets (Novartis)
300-mg tablet 548.17/100 tablets (Novartis)
600-mg tablet 1007.54/100 tablet (Novartis)
Trileptal/
Oxcarbazepine 300-mg/5-mL suspension 361.85/250 mL (Novartis)
Oxtellar XR
150-mg XR tablet 525.97/100 tablets (Supernus)
300-mg XR tablet 730.73/100 tablets (Supernus)
600-mg XR tablet 1337.89/100 tablets (Supernus)
2-mg tablet 409.32/30 tablets (Eisai)
4-mg tablet 750.96/30 tablets (Eisai)
6-mg tablet 750.96/30 tablets (Eisai)
Fycompa Perampanel
8-mg tablet 750.96/30 tablets (Eisai)
10-mg tablet 750.96/30 tablets (Eisai)
12-mg tablet 750.96/30 tablets (Eisai)
Luminal Phenobarbital 15-mg tablet 84.00/500 tablets (West-ward)
16.2-mg tablet 53.44/100 tablets (Qualitest)
30-mg tablet 105.60/500 tablets (West-ward)
32.4-mg tablet 67.50/100 tablets (Qualitest)
60-mg tablet 132.00/500 tablets (West-ward)
64.8-mg tablet 84.38/100 tablets (Qualitest)
97.2-mg tablet 119.03/100 tablets (Qualitest)
100-mg tablet 186.00/500 tablets (West-ward)
20-mg/5-mL elixir 91.80/473 mL (Qualitest)

ACCP Updates in Therapeutics® 2016: Ambulatory Care Pharmacy Preparatory Review and Recertification Course

1-434
Neurology

Table 12. Cost of Antiepileptic Drugs (continued)


Brand Generic Dosage Formulations Average Wholesale Price, $ (Mfr)
30-mg capsule ER 74.26/100 capsules (Pfizer)
100-mg capsule ER 86.12/100 capsules (Pfizer)
200-mg capsules ER 167.47/100 capsules (Mylan)
Dilantin Phenytoin
300-mg capsules ER 250.92/100 capsules (Mylan)
125-mg/5-mL suspension 107.35/237 mL (Pfizer)
50-mg chew tablet 80.70/100 tablets (Pfizer)
25-mg capsule 567.48/90 capsules (Pfizer)
50-mg capsule 567.48/90 capsules(Pfizer)
75-mg capsule 567.48/90 capsules (Pfizer)
100-mg capsule 567.48/90 capsules (Pfizer)
Lyrica Pregabalin 150-mg capsule 567.48/90 capsules (Pfizer)
200-mg capsule 567.48/90 capsules (Pfizer)
225-mg capsule 567.48/90 capsules (Pfizer)
300-mg capsule 567.48/90 capsules (Pfizer)
20-mg/mL solution 794.47/473 mL (Pfizer)
50-mg tablet 46.49/100 tablets (Qualitest)
Mysoline Primidone
250-mg tablets 99.60/100 tablets (Qualitest)
200-mg tablet 1078.80/120 tablets (Eisai)
Banzel Rufinamide 400-mg tablet 2157.60/120 tablets(Eisai)
40-mg/mL suspension 1198.80/460 mL (Eisai)
2-mg tablet 234.00/30 tablets (Teva)
4-mg tablet 234.00/30 tablets (Teva)
Gabitril Tiagabine
12-mg tablet 302.40/30 tablets (Teva)
16-mg tablet 396.00/30 tablets (Teva)
15-mg capsule sprinkle 1196.00/500 capsules (Mylan)
25-mg capsule sprinkle 1445.92/500 capsules (Mylan)
25-mg tablet 255.48/100 tablets (Mylan)
50-mg tablet 305.90/60 tablets (Mylan)
100-mg tablet 417.75/60 tablets (Mylan)
Topamax Topiramate 200-mg tablet 489.06/60 tablets (Mylan)
25-mg capsule XR 207.00/30 capsules (Upsher-Smith)
50-mg capsule XR 269.6/30 capsules (Upsher-Smith)
100-mg capsule XR 534.24/30 capsules (Upsher-Smith)
150-mg capsule XR 657.12/30 capsules (Upsher-Smith)
200-mg capsule XR 730.80/30 capsules (Upsher-Smith)
250-mg softgel capsule 79.40/100 SGC (Actavis)
250-mg/5-mL syrup 50.00/473 mL (Akorn)
Valproic Acid
125-mg DR tablet 135.78/100 DRT (Noven)
250-mg DR tablet 266.69/100 DRT (Noven)
Depakote
500-mg DR tablet 491.76/100 DRT (Noven)
Divalproex
125-mg capsule DR sprinkles 169.50/100 capsules (Abbvie)
Sodium
250-mg XR tablet 336.10/100 tablets (Abbvie)
500-mg XR tablet 558.61/100 tablets (Abbvie)
500-mg powder/1 packet 5283.84/50 packets (Lunkbeck)
Sabril Vigabatrin
500-mg tablet 10567.69/100 tablets (Lundbeck)

ACCP Updates in Therapeutics® 2016: Ambulatory Care Pharmacy Preparatory Review and Recertification Course

1-435
Neurology

Table 12. Cost of Antiepileptic Drugs (continued)


Brand Generic Dosage Formulations Average Wholesale Price, $ (Mfr)
25-mg capsule 54.75/100 capsules (Sun)
Zonegran Zonisamide 50-mg capsule 109.52/100 capsules (Sun)
100-mg capsule 219.17/100 capsules (Sun)
50-mg tablet 548.38/90 capsules (GSK)
200-mg tablet 1096.73/90 capsules (GSK)
Potiga Ezogabine
300-mg tablet 1096.73/90 capsules (GSK)
400-mg tablet 1096.73/90 capsules (GSK)
200-mg tablet 622.44/30 tablets (Sunovion)
400-mg tablet 829.44/30 tablets (Sunovion)
Aptiom Eslicarbazepine
600-mg tablet 1658.88/60 tablets (Sunovion)
800-mg tablet 829.44/30 tablets (Sunovion)
Red Book Online through Truven. New York: Thomas Reuters, 2015.DR = delayed release; DRT = delayed released tablets; ER = extended release; GSK
= GlaxoSmithKline;Inj=injectable;IV=intravenous; Mfr = manufacturer; ODT = orally disintegrating tablet; PE = phenytoin equivalents; SGC = softgel
capsule; XR = extended release.

II.  HEADACHE/MIGRAINE

A. Epidemiology
1. Headache
a. Very common
b. 45 million people in the United States regularly experience headache.
c. Costly and time-consuming
d. Poor use of EDs and health care resources
2. Migraine
a. 28 million Americans – 21 million women, 7 million men
b. In around 50% of patients, the problem is severe and disabling.
c. Often not diagnosed or treated appropriately
3. Tension-type headache
a. Self-reported in 69% of male individuals and 88% of female individuals
b. Most common of the primary headache disorders, with a lifetime prevalence of 30%–78%
c. Often begins in the early 20s
4. Cluster headache
a. Affects about 200 individuals in 100,000
b. Occurs predominantly in patients older than 30 years
c. Affects male individuals more than female individuals 6:1

B. Types of Headache
1. Primary headaches
a. Cluster headache
b. Tension-type headache
c. Migraine
i. With aura
ii. Without aura

ACCP Updates in Therapeutics® 2016: Ambulatory Care Pharmacy Preparatory Review and Recertification Course

1-436
Neurology

2. Secondary headaches
a. Head trauma
b. Vascular disorder
c. Nonvascular disorders
d. Toxic substances
e. Non-cephalic infection
f. Metabolic disorders
g. Disorders of face and neck
h. Cranial neuralgias
3. Medication-overuse headache
a. “Rebound headache” caused by frequent use of headache medications
b. Withdrawal symptoms on discontinuation of offending drug
i. Offending agents
(a) Analgesics – Especially combination products with caffeine
(b) Ergotamines
(c) Caffeine
(d) Opioids
(e) Triptans
(f) Barbiturates – Especially combination products
ii. Limit use to two or three times weekly for abortive therapy.
c. Toxic effects from medications
d. Escalating use of medications, increasing quantity or severity of headaches, dependence
on and habituation to symptomatic management

C. Cluster Headache
1. Clinical presentations
a. Sudden onset, excruciating, stabbing quality
b. Unilateral location with facial pain
c. Restlessness
d. Often, an attack occurs within 90 minutes of falling asleep
e. Can occur up to eight times within a 24-hour period
f. Typically, no long-term consequences
g. Occur in clusters for weeks to months; then a remission of months to years
2. Treatment options
a. Nonpharmacologic therapy
i. Avoid triggers.
(a) Vasodilators
(b) Alcohol
(c) Bright lights/glare
(d) Stress
ii. Avoid volatile substances.
(a) Gasoline
(b) Oil-based paint
iii. Medications to avoid: Acetazolamide
iv. Cold compress/cold air
v. Trigeminal ganglion procedures (refractory conditions with unilateral attacks)

ACCP Updates in Therapeutics® 2016: Ambulatory Care Pharmacy Preparatory Review and Recertification Course

1-437
Neurology

b. Pharmacologic therapy
i. Acute treatment
(a) Oxygen 6–12 L/minute for 15–20 minutes or 100% oxygen at 7–10 L/minute for 15–30
minutes by non-rebreather face mask
(b) Serotonin agonists
(1) Preferred as first-line therapy
(2) Triptans – Use intranasally or as injectable forms (injectable forms more effective).
(c) Ergotamines: Slow onset of action limits usefulness (no controlled trials support use).
(d) Intranasal lidocaine
(1) Sumatriptan nonresponder
(2) Contraindication to triptans
ii. Preventive treatment
(a) Short term (if patient can anticipate attacks [e.g., seasonal changes, stressful events])
(1) Corticosteroids (burst): Once yearly
(2) Methysergide (episodic)
(A) Young with low cardiovascular risk
(B) Risk of retroperitoneal and pulmonary fibrosis with use
(3) Ergotamines
(b) Long term
(1) Verapamil: Drug of choice for prolonged cluster headaches
(A) Effective in 70% of patients and usually seen after 1 week
(B) 240–960 mg/day for episodic and chronic cluster headaches
(2) Lithium
(A) More effective in chronic cluster headaches than in episodic (78% vs. 63%)
(B) Unknown mechanism of action
(C) Efficacy usually seen in 1 week.
(D) Use the least effective dose, typically 600–900 mg/day in divided doses.
(E) Target serum concentrations of 0.4–0.8 mEq/L
(F) Obtain serum concentrations during first week and occasionally thereafter.
(3) AEDs (gabapentin, topiramate, valproic acid): Can use with triptans
(4) Melatonin 10 mg/day
(5) Combination therapy often required (verapamil and lithium or verapamil
and topiramate)

D. Tension-Type Headache
1. Clinical presentation
a. Attacks on 15 days/month or more (180 days/year)
b. Dull and band-like pain
c. Bilateral
d. No nausea/vomiting or aura
e. Mild photophobia or phonophobia rarely reported
2. Treatment options
a. Nonpharmacologic therapy
i. Stress management
ii. Relaxation therapy
iii. Biofeedback

ACCP Updates in Therapeutics® 2016: Ambulatory Care Pharmacy Preparatory Review and Recertification Course

1-438
Neurology

iv. Modality treatments


(a) Ultrasonography
(b) Transcutaneous electrical nerve stimulation
(c) Hot/cold packs
v. Stretching
vi. Exercise
vii. Traction
viii. Trigger-point injections
b. Pharmacologic therapy
i. Acute treatment
(a) Analgesics (aspirin, acetaminophen)
(b) Nonsteroidal anti-inflammatory drugs (NSAIDs): Naproxen better than analgesics
(c) Combination analgesics with and without caffeine
ii. Preventive treatment
(a) Tricyclic antidepressants (TCAs)
(b) Selective serotonin reuptake inhibitors (SSRIs) if patient has depression
(c) Serotonin-norepinephrine reuptake inhibitors (SNRIs) (e.g., venlafaxine)
(d) Tetracyclic antidepressants: Mirtazapine
(e) Muscle relaxants – No evidence to support use
(f) Onabotulinum toxin A (Botox)

E. Migraine
1. Pathophysiology

Table 13. Migraine Triggers


↑, ↓ Sleep Smoking
Dehydration Strong perfumes
Stress Chocolate
Emotional letdown Caffeine
Skipping meals Cheeses
Medications Hormone changes
Alcohol Physical activity
Weather changes Loud noises

a. Neurovascular pain syndrome: Triggers leading to neurochemical alterations


i. Centers in the brain stem – Central
ii. Trigeminal ganglion (other ganglion) and trigeminal nerve – Central
iii. Reticular activating system
iv. Meningeal blood vessels – Peripheral
v. Thalamus
vi. Cortex-perceived pain
b. Involves the trigeminovascular system
i. Sterile neurogenic inflammation
ii. Induced by potent vasodilator release and leaky blood vessels
(a) Nitric oxide
(b) Substance P
(c) Neurokinin-A
(d) Calcitonin gene–related peptide

ACCP Updates in Therapeutics® 2016: Ambulatory Care Pharmacy Preparatory Review and Recertification Course

1-439
Neurology

2. Clinical presentation
a. Migraine without aura: At least five attacks fulfilling the following:
i. Headache lasting 4–72 hours
ii. Headache has two of the following:
(a) Unilateral location
(b) Pulsating quality
(c) Moderate or severe intensity
(d) Aggravated by routine physical activity
iii. During headache, at least one of the following symptoms:
(a) Nausea and/or vomiting
(b) Photophobia
(c) Phonophobia
b. Migraine with aura – At least two attacks fulfilling three or more of the following:
i. At least one fully reversible aura symptom
(a) Usually visual symptoms
(b) Commonly affects half the visual field
(c) Can include flashing lights (photopsia), zigzag pattern derived from an arc of light that
may enlarge (fortification spectrum), scintillating scotoma that usually begins as a spot
of flickering light in the center of the visual field, preventing vision within the area, and
decreased/loss of vision in half the visual field (hemianopsia)
(d) Sensory symptoms can include paresthesias or numbness.
(e) Motor symptoms can include weakness and aphasia.
ii. At least one aura symptom developing gradually throughout 4 minutes or two symptoms in
succession
iii. No aura symptom lasting more than 60 minutes
iv. Migraine headache follows aura within 60 minutes.

Patient Cases

Questions 5–7 pertain to the following case.


R.L. is a 32-year-old man who presents at the clinic with a headache. The headache has been ongoing for the past
hour, and the patient describes the pain as “an ice pick through my eye.” The patient states the headache is unilateral
and reports no symptoms of nausea or aura. The patient has nasal congestion. The patient states he has had four
headaches of this type during the past 2 years, occurring in the spring and fall. He reports that his father has these
headaches as well. While interviewing the patient, you notice he is having trouble sitting still.

5. Which type of headache is R.L. most likely suffering from?


A. Cluster headache.
B. Tension headache.
C. Migraine with aura.
D. Migraine without aura.

6. Which is the best acute treatment of R.L.’s headache?


A. Sumatriptan oral.
B. Methylsergide.
C. NSAIDs.
D. Oxygen therapy.

ACCP Updates in Therapeutics® 2016: Ambulatory Care Pharmacy Preparatory Review and Recertification Course

1-440
Neurology

Patient Cases (continued)

7. Which triptan would be the best therapy to recommend to R.L.?


A. Almotriptan oral.
B. Rizatriptan oral.
C. Sumatriptan subcutaneous injection.
D. Zolmitriptan orally disintegrating tablet.

3. Treatment options
a. Nonpharmacologic therapy: Relaxation training
i. Thermal biofeedback
ii. Cognitive behavioral therapy (CBT)
iii. Acupuncture
iv. Electrical nerve stimulation
v. Cervical spine manipulation
vi. Exercise
vii. Sleep hygiene
b. Pharmacologic therapy
i. Abortive
(a) Because of differences between patients in frequency, duration, and disability of migraine
headaches, selection of agents is based on the level of disability and patient-specific
characteristics. See Table 14 for options.
(1) NSAIDs, aspirin, and acetaminophen are practical choices for mild to moderate
attacks (no more than three times weekly for NSAIDs and aspirin).
(2) Narcotics should be reserved for those with moderate to severe attacks for whom
other methods are contraindicated.
(3) Migralex (aspirin 500 mg with buffered magnesium), which is not FDA label
approved and is sold online, is used for some patients with migraine, tension-type
headaches, sinus headaches, menstrual headaches, exertion headaches, and sex
headaches. (Because it is not FDA label approved, it is difficult to recommend this
therapy; however, pharmacists may come across it in practice.)
(4) Metoclopramide is listed as an antiemetic; however, it often has efficacy on the
headache pain as well.

ACCP Updates in Therapeutics® 2016: Ambulatory Care Pharmacy Preparatory Review and Recertification Course

1-441
Neurology

Table 14. Abortive Therapies for Migraine


Drug Class Drug Name Doses (mg) Maximum Dose/Day
Aspirin 325–650 –
Analgesics
APAP 325–1000 3000 mg
Aspirin, APAP, caffeine 250/250/65 APAP 3000 mg
Isometheptene, APAP,
Combination 65/325/100 NTE 5 tablets/12 hours
dichloralphenazone (Midrin, Epidrin)
analgesics
Butalbital,a APAP, caffeine (Fioricet) 50/325/40 NTE 6 tablets
Butalbital,a aspirin, caffeine (Fiorinal) 50/325/40 NTE 6 tablets
Diclofenac (Cataflam) 50–100 150 mg
Ibuprofen 200–1200 1200 mg
NSAIDs
Naproxen 250–500 1250 mg of base
Diclofenac (Cambia) 50 50 mg
Butorphanol nasal spray 1; MR in 4 hours 3–4 sprays/day
APAP with codeine
Opiate
Methadone IM Individualize dosing Individualize dosing
analgesics
Meperidine IM/IV
Butorphanol IM
NTE 3 tablets/24 hours;
Ergotamine sublingual (Ergomar) 2; MR after 30 minutes
5 tablets/week
Belladonna alkaloids, ergotamine,
0.2/0.6/40 NTE 16 tablets/week
phenobarbital
Ergotamine Caffeine/ergotamine 1/100 NTE 6 tablets/24 hours
alkaloids Dihydroergotamine IM or 1; MR every hour until
NTE 3 mg/24 hours
subcutaneous symptoms resolve
4 mg/mL
Dihydroergotamine nasal spray
0.5 mg each nostril; MR 2 mg
(Migranal)
in 15 minutes

ACCP Updates in Therapeutics® 2016: Ambulatory Care Pharmacy Preparatory Review and Recertification Course

1-442
Neurology

Table 14. Abortive Therapies for Migraine (continued)


Drug Class Drug Name Doses (mg) Maximum Dose/Day
6.25 or 12.5;
Almotriptan (Axert) 25 mg
MR in 2 hours
Naratriptan (Amerge) 1 or 2.5; MR after 4 hours 5 mg
Frovatriptan (Frova) 2.5; MR in 2 hours 7.5 mg
25, 50, or 100; MR in 2
Sumatriptan tablets (Imitrex) 200 mg
hours
5, 10, or 20;
Sumatriptan nasal spray 40 mg
MR in 2 hours
Sumatriptan subcutaneous injection 4 or 6; MR in 1 hour 12 mg
Triptans
Sumatriptan Dose Pro needle-free
6; MR in 1 hour NTE 12 mg in 24 hours
delivery system
Sumatriptan (Alsuma, Imitrex) 6; MR in 1 hour NTE 12 mg in 24 hours
Sumatriptan/naproxen (Treximet) 85/500; MR in 2 hours NTE 2 tablets/24 hours
Rizatriptan (Maxalt) 5 or 10; MR in 2 hours 30 mg
Eletriptan (Relpax) 20 or 40; MR in 2 hours 80 mg
1.25, 2.5, or 5; MR in 2
Zolmitriptan tablets (Zomig) 10 mg
hours
Zolmitriptan nasal spray 5; MR in 2 hours 10 mg
a
The U.S. Headache Consortium makes recommendations in favor of using these medications without evidence from RCTs to support their use. Note:
Benzodiazepines, opioids, and barbiturates are options, but they are not recommended on a long-term basis because they are habit-forming and can contribute
to the development of rebound headache.
APAP = acetaminophen; IM = intramuscular; IV = intravenous; MR = may repeat; NSAID = nonsteroidal anti-inflammatory drug; NTE = not to exceed; RCT
= randomized controlled trial.
Lacy CF, Armstrong LL, Goldman MP, et al. Drug Information Handbook, 19th ed. Hudson, OH: Lexi-Comp, 2010.
Bigal ME, Lipton RB, Krymchantowski AV. The medical management of migraine. Am J Ther 2004;11:130-40.
Comparison of available triptans. Pharmacist’s Letter/Prescriber’s Letter 2009;25:250509.

(b) Ergotamines/dihydroergotamines
(1) Affect several receptors
(2) Less specific to targeted receptor(s), thus causing more adverse effects
(3) Antiemetics given before injectable therapy
(c) Triptans
(1) Select the agent according to all relevant patient characteristics (e.g., disease states,
current medications, duration of migraine).
(2) If patient is experiencing nausea or vomiting, a non–oral triptan should be used (e.g.,
orally disintegrating tablet or injection).
(3) Sumatriptan needle-free delivery system (e.g., Sumavel Dose Pro) uses a burst of air to
deliver the drug just under the skin, and it should only be given in the stomach or thigh.
(4) Sumatriptan SC autoinjection (e.g., Alsuma and Imitrex): Administer in the lateral
thigh or upper arm; the autoinjector is for single use only.

ACCP Updates in Therapeutics® 2016: Ambulatory Care Pharmacy Preparatory Review and Recertification Course

1-443
Neurology

(5) Drug-drug interactions


(A) SSRIs, SNRIs, monoamine oxidase inhibitors (MAOIs), and other serotonergic
drugs should be used with caution in combination with triptans because there is
a potential risk of serotonin syndrome; Triptans with the longest half-life (e.g.,
frovatriptan > naratriptan) are at higher risk of causing serotonin syndrome
because of their duration of action. Consider recommending a triptan metabolized
by the CYP system or renal elimination if a patient is taking another drug that
uses monoamine oxidase (MAO); triptans at least partly metabolized by MAO
include almotriptan, rizatriptan, sumatriptan, and zolmitriptan.
(B) Symptoms of serotonin syndrome include rapid heart rate (HR), high BP,
tremor, rigidity, increased core temperature, sweating, and headache.
(C) Triptans should not be used together with ergot derivatives within 24 hours
of each other.
(d) Antiemetics
(1) Administer 15–30 minutes before administering abortive treatment, if needed.
(2) Options include metoclopramide, chlorpromazine, and serotonin antagonists.

Table 15. Pharmacokinetic Characteristics of Triptans


Drug Route Onset Duration Half-life Metabolism
Amerge Renal, 70%
Oral 1–3 hours Long 6 hours
(naratriptan) CYP
Axert CYP and
Oral 30–120 minutes Short 3–4 hours
(almotriptan) MAO
Frova
Oral 2–3 hours Long 26 hours Renal, 50%
(frovatriptan)
Oral/nasal spray/SC 20–30 minutes/
2.5 hours/
Imitrex injection 15 minutes/
Short 2 hours/ MAO
(sumatriptan) Needle-free delivery 10–15 minutes
115 minutes
system/autoinjector ~10 minutes
Maxalt/
Maxalt-MLT Oral/ODT 30–120 minutes Short 2–3 hours MAO
(rizatriptan)
Relpax
Oral 30 minutes Short 4 hours CYP3A4
(eletriptan)
Treximet
(sumatriptan/ Oral 20–30 minutes Short 2 hours
naproxen)
Zomig/
Oral and nasal spray/ Oral: 45 minutes CYP and
Zomig-ZMT Short 3 hours
ODT Nasal: 15 minutes MAO
(zolmitriptan)
CYP = cytochrome P450; MAO = monoamine oxidase; ODT = orally disintegrating tablet; SC = subcutaneous.
Comparison of available triptans. Pharmacist’s Letter/Prescriber’s Letter 2009;25:250509.
Lacy CF, Armstrong LL, Goldman MP, et al. Drug Information Handbook, 19th ed. Hudson, OH: Lexi-Comp, 2010.

ACCP Updates in Therapeutics® 2016: Ambulatory Care Pharmacy Preparatory Review and Recertification Course

1-444
Neurology

Patient Cases

Questions 8–10 pertain to the following case.


R.P. is a 35-year-old female athlete with a low HR and a history of migraines who comes to the clinic with a
headache. R.P. describes the headache as being unilateral and pulsating. It has persisted for the past 24 hours
and is aggravated by bright lights and physical activity. The patient also has nausea, but she has not vomited. She
experiences this type of headache about twice per month. R.P. is not currently taking prescription drugs for her
migraines but is interested in it because her over-the-counter Excedrin Migraine tablets are not controlling the pain.

8. Which would be best to recommend to R.P. for migraine prophylaxis?


A. Excedrin Migraine.
B. Topiramate 25 mg orally daily.
C. Sumatriptan 50 mg orally daily.
D. Propranolol 20 mg orally three times daily.

9. R.P. states that because of the nausea, she would prefer not to swallow a tablet. Which would be the best
abortive therapy for R.P.?
A. Almotriptan.
B. Frovatriptan.
C. Naratriptan.
D. Rizatriptan.

10. R.P. was given a prescription for eletriptan 20 mg at the onset of migraine. R.P. often flies out of the country
for business and has noticed that eletriptan does not last long enough for the migraine to be fully aborted.
Which agent would be the most appropriate choice for R.P. on long flights?
A. Almotriptan.
B. Frovatriptan.
C. Rizatriptan.
D. Sumatriptan.

ii. Prevention (2012 AAN guidelines on episodic migraine prevention)


(a) Appropriate if migraines interfere with daily routines, if they last many days, or if agent
is patient’s preference. Adequate trial of 2–6 months to determine efficacy
(b) Selection is based on patient-specific characteristics and history of therapies previously
tried; continue treatment for 3–6 months after frequency of migraines has decreased, at
which point they can be tapered off.
(c) Antiepileptic drugs
(1) Valproate and topiramate have proven efficacy and FDA indications for migraine
prevention—Level A.
(2) Start at low dose and gradually increase; many patients will require a low dose for
efficacy.
(3) Carbamazepine—Level C
(4) Gabapentin has conflicting data—Level U.
(5) Tiagabine, levetiracetam, and zonisamide may provide immediate benefit, yet further
study is needed.
(d) Antidepressants
(1) Amitriptyline, venlafaxine—Level B

ACCP Updates in Therapeutics® 2016: Ambulatory Care Pharmacy Preparatory Review and Recertification Course

1-445
Neurology

(2) Nortriptyline: Less sedating with fewer anticholinergic adverse effects


(3) Fluoxetine, fluvoxamine—Level U
(e) Antihypertensives
(1) β-Blockers
(A) Propranolol, metoprolol, timolol—Level A
(B) Atenolol, nadolol—Level B
(C) Nebivolol, pindolol—Level C
(D) Bisoprolol—Level U
(E) Propranolol and timolol have FDA indications for migraine prevention.
(2) Angiotensin-converting enzyme inhibitor: Lisinopril—Level C
(3) Angiotensin receptor blockers: Candesartan—Level C
(4) α-Agonist: Clonidine, guanfacine—Level C
(5) Calcium channel blockers: Nicardipine, nifedipine, nimodipine, verapamil—Level U
(f) Triptans – For short-term prevention of menstrual migraines
(1) Frovatriptan—Level A
(2) Naratriptan, zolmitriptan—Level B
(g) Miscellaneous drugs
(1) Methysergide
(A) Several adverse effects (chest pain, angina, nausea/vomiting, diarrhea)
(B) Used for refractory migraines
(C) Must be compounded, not commercially available
(2) Cyproheptadine—Level C
(A) Useful in migraines that are difficult to control
(B) Causes weight gain
(3) Magnesium gluconate—Level B
(4) Onabotulinum toxin type A (Botox) – FDA label approved for chronic migraine
(5) Riboflavin—Level B
(6) Butterbur—Level A
(7) Coenzyme Q10—Level C
(8) Feverfew—Level B

Level A: Agents are established as effective and “should be offered for migraine prevention.”
Level B: Agents are “probably effective and should be considered for migraine prevention.”
Level C: Medications are “possibly effective and may be considered for migraine prevention.”
Level U: Agents have evidence that is conflicting or inadequate to support or refute their use.

4. Special populations
a. Pregnancy
i. Recommend acetaminophen as first-line therapy.
ii. Nonresponders may try combination therapy (acetaminophen/metoclopramide,
acetaminophen/codeine)
b. Children
i. Older than 6 years: Ibuprofen and acetaminophen may be used.
ii. Older than 12 years: Sumatriptan nasal spray may be used.
iii. Conflicting data regarding oral triptans in children
5. Patient education
a. Routes of administration – Intramuscularly, subcutaneously, orally, nasally, or needle-free injection
i. Use products at onset of headache (see Table 13 and Table 14).
ii. For detailed patient instructions, see package inserts.

ACCP Updates in Therapeutics® 2016: Ambulatory Care Pharmacy Preparatory Review and Recertification Course

1-446
Neurology

b. Triggers
i. Stress
ii. Hormones in women
iii. Lack of or excess sleep
iv. Alcohol
v. Food (aged cheese, caffeine, cultured dairy, processed meats)
c. Monitoring
i. Adverse events (flushing, sweating, chest pain)
ii. Response to the drug(s) (headache diary)
iii. Headache relief at 2, 4, and 24 hours
iv. Keep a drug and trigger diary.
d. Useful resources: www.achenet.org
6. Practitioner education
a. www.aan.com
b. www.neurology.org/content/78/17/1337.full.pdf+html
7. Conclusions
a. Migraine is a widespread, serious health problem.
b. Only 38% of all migraineurs have been given a diagnosis.
c. Absenteeism and diminished productivity incur a considerable economic burden.

Table 16. Cost of Triptan Abortive Therapy


Average
Brand Generic Dosage Formulations (quantity)
Cost, $
Amerge Naratriptan 1 mg, 2.5 mg (1 box = 9 tablets) 487.48
6.25 mg (1 box = 6 tablets) 266.65
Axert Almotriptan
12.5 mg (1 box = 12 tablets) 533.40
Frova Frovatriptan 2.5 mg (1 box = 9 tablets) 657.79
25 mg (1 box = 9 tablets) 299.23
Sumatriptan
50, 100 mg (1 box = 9 tablets), 520.34
Imitrex
6-mg/0.5-mL refill kit (1 box = 1 vial) 842.34
Needle-free delivery system
6-mg/0.5-mL device (1 syringe) 183.74
Maxalt 5 mg (1 box = 12 tablets) 745.86
10 mg (1 box = 18 tablets) 745.86
Rizatriptan
5 mg (1 box = 3 tablets) 469.51
Maxalt MLT 10 mg (1 box = 3 tablets) 745.86
Relpax Eletriptan 20 mg, 40 mg (1 box = 6 tablets) 272.72
Treximet Sumatriptan/naproxen 85–500 mg (1 bottle = 9 tablets) 750.60
2.5 mg (1 box = 6 tablets) 546.86
Zomig
5 mg (1 box = 3 tablets) 273.42
Zolmitriptan
2.5 mg (1 box = 6 tablets) 546.86
Zomig ZMT
5 mg (1 box = 3 tablets) 273.42
Red Book Online through Truven. New York: Thomas Reuters, 2015.

ACCP Updates in Therapeutics® 2016: Ambulatory Care Pharmacy Preparatory Review and Recertification Course

1-447
Neurology

III.  PAIN

A. Epidemiology
1. Pain is a pervasive and costly problem in the United States.
a. Around 46 million Americans undergo inpatient surgical procedures each year and experience
acute surgical pain, and an estimated 100 million adults have a chronic pain condition.
b. Survey conducted by National Center for Health Statistics (NCHS) in 2009, as reported by adults
18 years and older on causes of chronic pain (preceding 3 months):
i. Severe headache or migraine (16.1%)
ii. Low back pain (28.1%)
iii. Neck pain (15.1%)
iv. Knee pain (19.5%)
v. Shoulder pain (9.0%)
vi. Finger pain (7.6%)
vii. Hip pain (7.1%)
c. Chronic pain rates are anticipated to continue to rise because of myriad factors (e.g., aging
population, prevalence of obesity, shift toward ambulatory surgical procedures, progress in
lifesaving measures for catastrophic injuries, increased access to care).
d. Annual cost of chronic pain estimated at $560–$635 billion
e. About 3.2% of the total workforce experienced a loss in productive time during a 2-week period
because of back pain.

B. Pathophysiology
1. Classification: Four primary types of pain
a. Nociceptive (somatic/visceral): Sensory experience of acute pain caused by a noxious stimulus
b. Inflammatory: Tissue damage beyond nociceptive defense system, leading to inflammation
c. Neuropathic: Arises from lesions to peripheral or CNS
d. Functional: Pain caused by abnormal responsiveness of the nervous system in the absence
of a detectable neurologic deficit
2. Mechanisms of pain
a. Nociception
i. Transduction: Process by which external noxious stimuli are converted to electrophysiologic
activity in the peripheral terminals of nociceptors (pain receptors). These may differ for
different painful conditions. In each situation, neurotransmitters and ions are released,
including the following:
(a) Serotonin
(b) Norepinephrine
(c) Bradykinin
(d) Prostaglandin
(e) Histamine
(f) Substance P
(g) Neurokinin
(h) Potassium and hydrogen
ii. Transmission: Process by which pain signals ascend through the spinothalamic tract
iii. Perception: Process that occurs in the cortex, allowing an awareness of the experience of pain
b. Peripheral sensitization
i. Inflammatory-associated changes after tissue injury in the environment of the nerve fiber
ii. “Inflammatory soup,” which includes endothelin, prostaglandin E, leukotrienes, bradykinin,
cytokines, and serotonin; adrenaline is released, resulting in enhanced excitability

ACCP Updates in Therapeutics® 2016: Ambulatory Care Pharmacy Preparatory Review and Recertification Course

1-448
Neurology

c. Central sensitization
i. Amplification of pain signals after repeated noxious stimuli from peripheral nociceptors
ii. This process is thought to explain pain syndromes such as phantom limb pain, reflex
sympathetic dystrophy, and hyperalgesia.
d. Modulation: Process of modification of pain signals that occurs throughout the
ascending and descending pathway. Endogenous systems of modulation include
inhibitory neurotransmitters/receptors.
i. Endogenous opioids
ii. Blockade of NMDA (N-methyl-D-aspartate)
iii. Serotonin
iv. γ-Aminobutyric acid
e. Neuropathic and functional pain
i. Neuropathic pain (e.g., postherpetic neuralgia, diabetic neuropathy): Additional mechanistic
processes include ectopic excitability, structural reorganization, decreased inhibition.
ii. Functional pain (e.g., FM, irritable bowel syndrome, sympathetic induced pain):
Central sensitization
3. Definitions related to pathophysiology
a. Allodynia – Presence of pain from stimuli that are not normally painful. Example: A patient with
diabetes who experiences pain when placing socks on feet
b. Hyperalgesia – Normal painful stimuli produce exaggerated pain responses in patients with
peripheral or central sensitization.
Example: An immunization in the deltoid muscle causes patients to experience extreme pain for
many days, preventing them from lifting their arm; the injection site has no obvious erythema or
signs of an injection site reaction.
4. Chronic versus acute pain
a. Acute pain
i. Pain is typically a result of an acute disease process (may be part of a mixed
disease process – Acute or chronic); as part of a disease process, acute pain is typically
managed as a symptom.
ii. Pathophysiology is primarily inflammatory.
iii. Treatment goals include cessation of pain or substantial relief.
iv. Anticipatory anxiety may be involved; patients are worried that pain will worsen or
that pain will reoccur when medication wears off.
b. Chronic pain
i. A physiologic cause may not be determinable. Chronic pain should be managed as an
independent disease state with a monitoring and treatment plan apart from other plans.
ii. Most types of chronic pain are mixed—Somatic and neuropathic.
iii. Treatment goals include a reduction in pain and an improved quality of life. Patients with
chronic pain do not usually achieve remission of their pain. A discussion between the health
care provider and patient should involve realistic expectations regarding goal setting.
iv. Anxiety and depression are often comorbidities with chronic pain. Most patients develop a
tolerance to and dependence on medications. Estimates for the incidence of depression in
patients with chronic pain range from 30% to 50% according to the International Association
for the Study of Pain.

ACCP Updates in Therapeutics® 2016: Ambulatory Care Pharmacy Preparatory Review and Recertification Course

1-449
Neurology

Patient Case

11. A 45-year-old woman is seen in the clinic on a regular basis. Her medical history includes FM, arthritis,
constipation, and diabetes. Her chief concern today is severe pain in her shoulder, which is 10/10. The pain
is not believed to be cardiac in origin after undergoing an ECG and physical examination. The pain is worse
when the shoulder is manipulated. She rates it a 9/10. The nurse tells you and the physician that it cannot be
that bad—her BP is normal (130/80 mm Hg), and her HR is 90 beats/minute. Which is the best response to
give the nurse?
A. Patients with chronic pain may not have alterations in their BP and HR.
B. The patient is misrepresenting her pain, which should be documented in her chart.
C. Patients have different thresholds of response to pain—hers may be higher.
D. We have to believe what patients tell us about their pain.

C. Approach to Treatment
1. Key principles from the World Health Organization (WHO) analgesic ladder (Figure 2)
a. Oral administration of analgesics whenever possible
b. Analgesics should be given at regular intervals.
c. Analgesics should be given according to pain intensity, as measured by a pain scale.
d. Individualize the dose of analgesics.
Prescription for analgesics should provide sufficient details for proper administration.
2. Adaptation of WHO analgesic ladder to guide treatment options (Figure 3)
3. Approach based on diagnosis
a. Acute pain
i. Patients should be assessed for the physiologic source of their pain and, after reasonable
efforts, be treated for the underlying cause. Pain management should include an assessment
of pain severity.
ii. Considerations for managing acute pain
b. Chronic pain
i. Arthritis
ii. Fibromyalgia
iii. Chronic back pain
iv. Neuropathy

Strong opioid and nonopioid


(with or without adjuvants)
If pain persists or increases

Weak opioid
with nonopioid
If pain persists or increases
(with or without adjuvants)
Nonopioid
(with or without adjuvants)

Figure 2. Adapted from: World Health Organization analgesic ladder for treating cancer pain.
Vargas-Schaffer G. Is the WHO analgesic ladder still valid: twenty-four years of experience. Can Fam Physician 2010;56:514-7.

ACCP Updates in Therapeutics® 2016: Ambulatory Care Pharmacy Preparatory Review and Recertification Course

1-450
Neurology

Step 4
Neurosurgical
procedures Nerve block
Epidurals
Acute pain Step 3 PCA pump
Chronic pain without control Neuroliric block therapy
Strong opioids Spinal stimulators
Acute crisis of chronic pain
Medhadone
Step 2 Oral administration
Transermal patch
Weak opioids
Chronic pain
Non-malignant pain
Step 1
Cancer pain
Nonopioid NSAIDs
analgesics (with or without adjuvants
NSAIDs at each step

Figure 3. New adaptation of the analgesic ladder.


NSAID = nonsteroidal anti-inflammatory drug; PCA = patient controlled analgesia.
Vargas-Schaffer G. Is the WHO analgesic ladder still valid: twenty-four years of experience. Can Fam Physician 2010;56:514-7.

Mild

Moderate

• Long-acting opioid analgesics and combinations with breakthrough medications


• Around-the-clock-dosing
• May require adjuvants in certain situations as well as potentially agents for breakthrough pain
Severe • Monitor and reassess – Patients are at higher risk for adverse

Figure 4. Considerations for management of acute pain.


NSAID= nonsteroidal anti-inflammatory drug.

D. Fibromyalgia
1. Diagnostic criteria (American College of Rheumatology [ACR] 2010):
a. Widespread pain index (WPI) of 7 or greater* and symptom severity (SS) score** of 5 or greater,
or patients may have a lower WPI (3–6) if SS score is greater than 9
b. Symptoms have been present for greater than 3 months.
c. There is no other diagnostic explanation for the pain/symptoms.
*WPI – Number of anatomic areas with pain – Scale ranges from 0 to 19.
**SS – Patients are rated from 0 to 3 on fatigue, cognitive symptoms, waking unrefreshed, and
somatic symptoms. The final score is from 0 to 12.

ACCP Updates in Therapeutics® 2016: Ambulatory Care Pharmacy Preparatory Review and Recertification Course

1-451
Neurology

2. Pharmacotherapy
a. Tramadol is recommended for the management of moderate to severe pain in FM.
b. Simple analgesics such as acetaminophen and other weak opioids can also be considered in the
treatment of FM. However, corticosteroids and strong opioids are not recommended.
c. Antidepressants: Amitriptyline, fluoxetine, duloxetine, milnacipran; reduce pain and often improve
function; therefore, they are recommended for the treatment of FM.
d. Pramipexole and pregabalin reduce pain; they are recommended for the treatment of FM.
3. Nonpharmacologic therapy
a. Heated pool treatment with or without exercise is effective in FM.
b. Trials show that patient-specific exercise programs, including aerobic exercise and strength
training, can be beneficial to some patients with FM. However, these should be undertaken with
supervision to increase adherence and prevent injuries.
c. CBT may be of benefit to some patients with FM. CBT has the strongest level of evidence among
the nonpharmacologic therapy options in the current guidelines. There is also literature to support
the use of guided relaxation, rehabilitation, physiotherapy, and psychological therapy.

E. Chronic Back Pain


1. Risk factors
a. Obesity
b. Depression
c. Occupations associated with lifting (e.g., health care workers, construction)
2. Cauda equina syndrome (medical emergency) warning symptoms:
a. New-onset urinary retention or incontinence
b. Fecal incontinence
c. Bilateral motor weakness or sensory loss in lower extremities
d. Saddle anesthesia
3. Pharmacotherapy
a. First-line agents – Acetaminophen and NSAIDs
b. Second-tier agents – Tramadol and long-acting opioids (only for patients who are disabled by their
pain or are not candidates for first-line agents)
c. Skeletal muscle relaxants – Limited benefit and high incidence of sedation
d. If neuropathic component, consider adding an SNRI or an anticonvulsant: Duloxetine, gabapentin
(limited data).
e. Other agents: Zonisamide is efficacious in randomized controlled trials.
f. Epidural/facet injections
g. Corticosteroids, opioids, or anesthetics
h. Benefit: Delivery of medication to the site of inflammation
i. Risks: Infections and very small risk of spinal cord damage
j. Precautions: Patients receiving anticoagulation or antiplatelet therapy
4. Nonpharmacotherapy
a. Modalities supported by guidelines: Yoga, massage, acupuncture, relaxation, exercise, and CBT
b. For patients who are obese, weight loss will improve outcomes.

ACCP Updates in Therapeutics® 2016: Ambulatory Care Pharmacy Preparatory Review and Recertification Course

1-452
Neurology

Table 17. Adjuvant Medications for Pain Management


Dosage
Common Adverse Effects Comments
Range, mg
Pregabalin 300–600 Sedation, weight gain
Dose should be reduced in renal
Gabapentin 1800–3600 Sedation, weight gain
dysfunction
Should not be used if CrCl < 30 mL/
Nausea
minute; use with extreme caution
Duloxetine 60–120 Increased BP
with other serotonergic agents; rare
Headache
LFT changes
Dosage adjustment is necessary if
Venlafaxine 75–225 Hypertension
CrCl < 50 mL/minute
Desipramine Doses effective for pain may not be
25–150 Sedation, anticholinergic
Nortriptyline therapeutic for depression
Sedation, paresthesia, nephrolithiasis,
Titration by 25 mg weekly will
secondary angle closure glaucoma,
Topiramate 50–200 reduce word-finding difficulty and
neurocognitive impairment, metabolic
other neurocognitive effects
acidosis
Hypertension, palpitation, nausea,
Milnacipran 100–200
constipation, headache
BP = blood pressure; CrCl = creatinine clearance; LFT = liver function test.

Patient Cases

Questions 12 and 13 pertain to the following case.


L.L., a 44-year-old woman, has chronic pain in both shoulders, her hips, her knees, and her ankles and feet on a
daily basis. She also suffers from obesity, chronic constipation, difficulty concentrating, and insomnia. The pain
has continued to get worse during the past 2 years. While you are taking her medication history, she tells you
that she has seen different physicians during those 2 years. She is now extremely frustrated. L.L. states that she
has two main symptoms bothering her today—her pain and her inability to sleep. She would like both conditions
treated. Her medication list consists of naproxen 250 mg twice daily and sertraline 25 mg daily. You identify a
need for L.L. to be provided patient education regarding her chronic pain disorder.

12. Which topic is most important for L.L. to receive education on?
A. L.L. should be educated about osteoarthritis and the importance of daily exercise.
B. L.L. should be educated about FM and the importance of daily exercise.
C. L.L. should be educated about insomnia and the importance of sleep hygiene.
D. L.L. should be educated about chronic pain and the importance of self-care (i.e., exercise and sleep).

ACCP Updates in Therapeutics® 2016: Ambulatory Care Pharmacy Preparatory Review and Recertification Course

1-453
Neurology

Patient Cases (continued)

13. While L.L. is at the clinic, you recommend that she begin therapy for her chronic pain disorder. Which is the
most appropriate initial therapy for her pain?
A. She should begin chronic opioid therapy with a trial of hydrocodone/acetaminophen 5/325 mg.
B. She should begin a trial of tramadol 50 mg after discontinuing sertraline.
C. She should begin a trial of duloxetine 60 mg after discontinuing sertraline.
D. She should begin a trial of pregabalin 150 mg in addition to her current medications.

14. T.D., a 42-year-old patient with a history of chronic low back pain, obesity, and diabetes, calls the clinic
today. He says his medications are not helping with his pain. He currently takes meloxicam 15 mg daily and
cyclobenzaprine 10 mg daily. Today, he states that he cannot get out of bed because his right leg is completely
numb from hip to toes. T.D. has never experienced this type of symptom before. Which is the best course of
action for this patient?
A. Recommend that T.D. be referred to an interventional pain specialist.
B. Recommend that T.D. be referred for immediate evaluation of his back pain.
C. Recommend that T.D. be referred to an orthopedic specialist for evaluation of his back pain.
D. Recommend that T.D. be referred to a pain specialist for chronic opioid therapy.

F. Chronic Opioid Use: Monitoring and Optimization of Therapy


1. Number of unintended deaths associated with opioids
a. Intentional overdose or criminal overdoses
i. Prescription opioid overdose 2010: About 16,500 deaths and around 40,000 ED visits
ii. For every overdose death that occurred in the United States, there were 733 instances of
nonmedical use, 108 of abuse/dependence, 26 ED visits for misuse or abuse, 10 abuse
treatment admissions.
b. Deaths associated with opioids in 2009 (13 states): 3294 deaths caused by several substances,
including opioids, and 748 single-drug deaths caused by opioids
c. Epidemiologic risk associated with methadone prescribing
i. Centers for Disease Control and Prevention (CDC) on methadone – Should not be used in
patients who are opioid naive because of the risk of respiratory depression
ii. There is also a very slight risk of sudden death with methadone. All patients initiating
methadone therapy should have a baseline ECG and then regular monitoring thereafter
(it is suggested that a repeated ECG be done 30 days after methadone initiation). Patients
should not be given methadone in combination with other medications known to prolong the
corrected QT (QTc) interval.
iii. If the QTc interval is between 450 and 500 milliseconds, patients should be warned of the
potential risks and benefits of methadone. If the QTc interval exceeds 500 milliseconds, not
using methadone should be considered. Estimated number of ED visits for nonmedical use of
opioid analgesics increased 111% during 2004–2008.
2. Risk Evaluation and Mitigation Strategies (REMS)
a. To decrease the number of serious adverse events associated with opioids, REMS programs were
created to increase awareness of the risks and benefits of chronic opioid therapy.
b. These programs encourage prescriber education, patient counseling, and medication guides to be
provided through pharmacy providers.

ACCP Updates in Therapeutics® 2016: Ambulatory Care Pharmacy Preparatory Review and Recertification Course

1-454
Neurology

c. Manufacturers are required to provide these programs for extended-release morphine, extended-
release oxycodone, methadone, buprenorphine, tapentadol, fentanyl, oxymorphone, and
hydromorphone. Several opioid manufacturers have provided class-wide REMS programs,
which serve as universal education for a group of medications such as the fentanyl products—
Transmucosal and immediate release. These programs are intended to increase provider and patient
use of a consistent education source and reduce confusion.
3. Guidelines for opioid use in chronic pain
a. American Pain Society and American Academy of Pain Medicine: Clinical Guidelines for the Use
of Chronic Opioid Therapy in Chronic Noncancer Pain (2009)
b. Federation of State Medical Boards – Model Policy on the use of opioid analgesics in the
treatment of chronic pain (2013)
c. Washington State Agency Medical Directors’ Group (AMDG) – Interagency Guideline on Opioid
Dosing for Chronic Non-cancer Pain (2010)
4. Evaluation of patients for appropriate use. Complete medical history and physical examination,
including an assessment of pain severity, to be documented in the medical records at each clinic visit.
a. Visual Analog Scale (VAS)
i. 10-cm line – Patients indicate what point on the line corresponds to their pain intensity –
Yields a number from 0 to 100
ii. This scale may be difficult for some patients to understand.
iii. Pain numeric rating scale: 0–10 – Patients rank their pain. Most commonly used evaluation
tool. Patients can become desensitized to this tool—“Always a 10.”
b. Brief Pain Inventory
i. Patients indicate on a diagram of the body where they experience pain.
ii. Patients also rate the severity of their pain and the relief they obtain from currently prescribed
agents. This tool requires the patient to have a higher reading level.
iii. Good for ongoing monitoring
c. McGill Pain Questionnaire
i. Patients are asked to rate their pain and then choose descriptor words for their pain.
ii. This tool requires the patient to have a higher reading level.
iii. Good for initial evaluation
5. History should include the following:
a. Past medications – Reasons for discontinuation and degree of efficacy
b. Current medications – Level of efficacy and any adverse effects. Current level of pain relief using
the appropriate pain assessment tool
c. Presence of clear indication for the use of chronic opioid therapy as opposed to other therapy
d. Social history including family support, occupational status and work activities, use of alcohol and
over-the-counter/illicit medications
6. Suitability for chronic opioid therapy should be evaluated.
a. Screener and Opioid Assessment for Patients with Pain (SOAPP)
i. Five-, 14-, or 24-question tool completed by patients to classify them as being at high,
medium, or low risk of misusing opioids
ii. High-risk patients
iii. Should have exhausted all other therapeutic options before opioid initiation
iv. Opioid should be initiated with strict protocols including urine drug screening and regular
counseling—May consider an addiction/pain management specialist

ACCP Updates in Therapeutics® 2016: Ambulatory Care Pharmacy Preparatory Review and Recertification Course

1-455
Neurology

Table 18. Testing Methods Available for Patient Monitoring of Chronic Opioid Therapy
Test Advantages Disadvantages
Less expensive May not detect all medications in
Urine immunoassay Longer detection window than serum testing class (e.g., synthetic opioids)
Less invasive than serum testing More false positives than GC-MS
More precise results
Urine GC-MS Costly
Longer detection window than serum testing
Shows results for metabolites
In patient taking high dose, may detect aberrant
Quantitative serum
metabolism pattern Costly
testing
Provides documentation for provider of serum
concentrations
GC-MS = gas chromatography–mass spectrometry.

Patient Case

15. The quality management team for your practice wants to implement a protocol for monitoring all patients
receiving chronic opioid therapy. The team wants you to evaluate the available patient monitoring laboratory
tests and choose the best test for the preliminary screening of patients. The team would ideally like a test
that could screen for most opioids prescribed in the practice, including fentanyl patches, methadone, and
controlled-release forms (oxycodone and morphine). Which test would be best to recommend for the protocol?
A. Urine immunoassay – EMIT.
B. Gas chromatography–mass spectrometry.
C. Serum quantitative screening test.
D. Saliva testing.

b. Opioid risk tool (ORT)


i. Five-question tool completed by the patient
ii. This tool is easily completed in the office setting and is particularly suited for those
anticipated to be at low risk.
iii. Patients are asked about family and personal history of substance abuse, mental health
history, and age. For these yes/no questions, a score is generated to rank each individual
patient’s risk as high, medium, or low for misuse risk with chronic opioid therapy.
iv. The tool was originally validated in patients who were not expected by their health care
providers to have risks of misusing medication.
c. Diagnosis, intractability, risk, efficacy (DIRE)
i. Seven items completed by patient interview
ii. Correlates with efficacy of chronic therapy
iii. Better evaluation of environmental factors, which may influence the success of chronic
opioid therapy
d. Current opioid misuse measure (COMM)
i. Seventeen-item questionnaire designed to assess patients for addiction, problematic behavior,
emotional volatility
ii. Designed to evaluate patients with a history of substance misuse to determine their level
of risk while receiving chronic opioid therapy
iii. The COMM is helpful in monitoring a patient’s medication-related behavior while receiving
opioid therapy

ACCP Updates in Therapeutics® 2016: Ambulatory Care Pharmacy Preparatory Review and Recertification Course

1-456
Neurology

Patient Case

16. A patient is being evaluated for initial therapy with chronic opioid therapy. She will initially be seen on a
weekly basis. Her physician would like to determine her risk level for opioid therapy to help design her plan
for therapy. She has no history of substance use. The physician is more comfortable with completing the risk
tool during his examination/interview. Which tool is most appropriate for this situation?
A. Screener and Opioid Assessment for Patients with Pain (SOAPP).
B. Diagnosis, intractability, risk, efficacy (DIRE).
C. CAGE questionnaire.
D. Current opioid misuse measure (COMM).

7. Treatment plan – Appropriate treatment goals are crucial to patient success and do not necessarily
change with changes in therapy.
Example: Goal – Patients are able to stay at work 4 days a week for 6 hours. Goal – Patients are able
to get their own groceries each week unassisted.
8. Informed consent and agreement for treatment
a. Initial agreements
b. Patient responsibilities
i. Patients should attend all appointments.
ii. Patients should provide an accurate accounting of their symptoms.
iii. Patients should keep all medications in a safe place—Medication locker or cabinet.
iv. Patients will use only one pharmacy. All medications for pain management will be obtained
from one pharmacy.
v. On a regular basis, urine or blood samples may be needed to monitor for safety and efficacy
of the medication.
vi. Patients should follow the treatment plan for medications and other sections of the treatment
plan, including exercise, physical therapy, CBT, and counseling.
vii. Do not share medications or take medications received from others.
c. Provider responsibilities
i. Provider will listen respectfully to all patient concerns.
ii. Information will be given to the patient regarding all tests, diagnoses, and medications in a
language the patient can understand.
iii. Provider will communicate information regarding the patient’s medication therapy to other
health care providers or employers if requested by the patient.
iv. Each treatment plan will likely include medication as well as several forms of
nonpharmacologic therapy.
d. Punitive agreements – Agreements formed in response to problem behaviors should be patient-
specific, and they usually focus on improper use of opioids or illicit substances. Example sentences
may include the following:
i. A positive urine drug screen for marijuana, cocaine, or other medications that have not been
prescribed to the patient will result in termination of the current patient-provider relationship
with XYZ clinic.
ii. A quantity of X medication has been provided to treat a patient’s chronic pain. This
medication will be prescribed on a monthly basis after an appointment and evaluation of
the patient’s pain. Any pain medications received from other pain prescribers outside this
prescription (including x, y, z) will result in termination of the patient-provider relationship.

ACCP Updates in Therapeutics® 2016: Ambulatory Care Pharmacy Preparatory Review and Recertification Course

1-457
Neurology

9. Periodic review
a. Patients who are receiving opioids on a regular basis should have their therapy evaluated for efficacy
and safety during a patient visit. These discussions should be documented in the patient record.
b. Throughout chronic opioid therapy, it is important to monitor the patient periodically for each of
the following:
i. Pain severity should be monitored on a routine basis to determine progress toward goals.
ii. Functional ability – A patient’s functional ability should be determined at baseline and then
monitored throughout therapy.
iii. Progress toward treatment goals
iv. Adverse effects
v. Effects secondary to comorbid conditions
vi. Aberrant drug-related behaviors
vii. Identify patients who might benefit from restructuring of the treatment plan.
viii. Identify patients who might benefit from receiving additional services such as addiction
counseling or treatment.
ix. Document ongoing vigilance for continuation of safe prescribing of the opioids.
x. Diversion – Many cases of unintended overdose occur in children and young adults who had
access to a family member’s medication. It is very important for patients to maintain control
of their medications.
xi. Misuse – Using pain medication to treat other conditions such as anxiety or sleep
xii. Addiction – A compulsive desire to use a drug despite continued harm. Addiction is a chronic,
neurobiologic disease that is influenced by genetic and environmental factors.
10. Consultation
a. For patients with a history of substance abuse or significant comorbid mental illness – A referral
for management by a pain specialist or pain/addiction specialist should be considered.
b. Many types of pain require interdisciplinary management to reach therapeutic goals. This may
include physical therapy, occupational therapy, psychologist/psychiatrist, social work, physician,
pharmacist, massage therapist/chiropractic manipulation, acupuncture, and nursing.
11. Documentation – All discussions and therapeutic plans must be documented in the patient chart, and
consideration of evaluation must be made by using symptom analysis (PQRST) as well as assessment
of the “4 A’s”: Analgesia, adverse effects, aberrant behavior, and activity.
12. Compliance with all controlled substance rules and regulations for state and federal prescribing authorities
13. Screening for adverse reactions
14. Choose the right opioid – May need to change agents to achieve therapeutic efficacy, minimize adverse
effects, with suitable pharmacokinetic properties in a suitable dosage form at an affordable cost to patient
a. Opioid conversions/rotations
b. Determine the total daily dose of all opioids being administered.
c. Using a published equianalgesic ratio, convert the current total daily dose to the new opioid.
d. Round down the new analgesic dose by 25%–50% (75%–90% lower if switching to methadone).
Patients do not respond to a new opioid with complete tolerance for an equianalgesic dose. Lower
adjustments are needed if the patient’s pain is severe at the time of conversion.
e. Determine a dosing schedule for the new regimen.
f. Schedule increased monitoring – Clinic visits or telephone follow-up during the period until
the patient’s new medication reaches steady state
g. Conversion ratios are used as a guide – Variability in interindividual pharmacokinetics and opioid
drug pharmacodynamics

ACCP Updates in Therapeutics® 2016: Ambulatory Care Pharmacy Preparatory Review and Recertification Course

1-458
Neurology

h. CYP2D6 polymorphism
i. Codeine (converts to morphine; poor metabolizers – Reduced response)
ii. Oxycodone (converts to oxymorphone; poor metabolizers – Increased adverse effects)
iii. Hydrocodone (converts to hydromorphone; extensive metabolizers – May have increased
adverse effects)
iv. Potential DDIs

Table 19. Equianalgesic Doses of Outpatient Opioids


Equianalgesic Available
Notes Regarding Use of This Agent
Oral Dose, mg Brand Names
MS Contin, Oramorph Adverse effects often limit use – Nausea, itching;
Morphine 30 SR – Long acting, may require tid dosing in some patients; should
Avinza, Kadian not be used in patients with renal compromise
Zohydro -
Hydrocodone alone
This short-acting agent may be suitable to test
Lortab, Norco, Vicodin
Hydrocodone 30 opioid responsiveness, but it is not
– With acetaminophen
usually suitable for chronic opioid therapy
Reprexain, Vicoprofen
– With ibuprofen
Patients often report itching and nausea <
Various generic
Codeine 200 constipation with codeine. Conversion to
products
morphine by 2D6 required for analgesia
Hydromorphone 7.5 Dilaudid – Short acting
OxyContin, Percocet,
Oxycodone 20
Xartemis, others
Oxymorphone 10 Opana
Can cause nausea and dysphoria in some patients;
should be avoided with other serotonergic agents;
Tramadol 120 Ultram, Ultram ER classified as a CIII drug in some states; conversion
to O-desmethyl metabolite through 2D6 required
for analgesia
Has fewer overall GI adverse effects than other
Nucynta and Nucynta
Tapentadol 100 opioids but is a CII drug; approved for both acute
ER
and chronic pain
ER = extended release; GI = gastrointestinal; SR = sustained release; tid = three times daily.

ACCP Updates in Therapeutics® 2016: Ambulatory Care Pharmacy Preparatory Review and Recertification Course

1-459
Neurology

Table 20. Initial Methadone Dose Conversion According to Oral MEDD


Initial Dose Ratio
Oral MEDD, mg/day
(oral morphine/oral methadone)
<30 2:1
30–99 4:1
100–299 8:1
300–499 12:1
500–999 15:1
>1000 ≥20:1a
a
When very high opioid doses have been used, great caution should be exercised. Consider converting a portion of the total opioid dose initially, with further
conversions for several days to weeks.
MEDD = morphine equivalent daily dose.
Adapted from: Pain (PDQ). Bethesda, MD: National Cancer Institute, September 25, 2013.

15. Dosage forms – The appropriate dosage form should be chosen according to the ability of the patient to
adhere to the dosing regimen, affordability, and history of efficacy with the agent.
a. Oral – The most appropriate dosage form for most patients. However, it can be difficult to
maintain a consistent concentration if the patient does not take the dose consistently.
i. Immediate-release tablets – These dosage forms are not ideal for chronic pain. Immediate-
release forms should be used for acute pain and breakthrough pain.
ii. Extended release
iii. Buccal/sublingual – Fentanyl dosage forms are available, but they should only be used in
opioid-tolerant patients for relief of severe breakthrough pain in cancer.

Table 21. Recommendations for Conversion from Morphine to Transdermal Fentanyl


Total Opioid Dose in Fentanyl Patch Dose,
Morphine Equivalents, mg/day mcg/hour
60–134 25
135–224 50
225–314 75
315–404 100
405–494 125
495–584 150
585–674 175
675–764 200

iv. Conversion for fentanyl transdermal – Fentanyl package insert


b. Nasal spray
i. Butorphanol – Useful for some patients with migraines for acute relief. However, can lead
to medication-overuse headaches and is not recommended by AAN guidelines
ii. Fentanyl nasal spray – Indicated only for cancer breakthrough pain in opioid-tolerant patients

ACCP Updates in Therapeutics® 2016: Ambulatory Care Pharmacy Preparatory Review and Recertification Course

1-460
Neurology

Patient Case

17. Z.T. has taken oxycodone controlled release for the past 8 years, 90 mg twice daily, for chronic low back pain
and diabetic neuropathy. Z.T. informs you today that there is a national shortage of his medication. Which
dose is best to suggest when changing Z.T.’s medication to morphine controlled release?
A. 60 mg twice daily.
B. 30 mg four times daily.
C. 120 mg daily.
D. 100 mg twice daily.

Table 22. Average Prices for Oral Medications Used for Pain
Brand Name Generic Name Dosage Form Average Wholesale Price, $
MS IR 15-mg tablet 27.07/100 tablets
30-mg tablet 46.12/100 tablets
MS Contin 15-mg tablet 167.57/100 tablets
30-mg tablet 317.19/100 tablets
60-mg tablet 621.33/100 tablets
100-mg tablet 919.95/100 tablets
200-mg tablet 1838.19/100 tablets
Avinza 30-mg capsule 572.96/100 capsules
45-mg capsule 849.55/100 capsules
60-mg capsule 1112.63/100 capsules
75-mg capsule 1415.90/100 capsules
90-mg capsule 1672.92/100 capsules 1973.87/100
Morphine sulfate
120-mg capsule capsules
Kadian 10-mg capsule 473.21/100 capsules
20-mg capsule 522.89/100 capsules
30-mg capsule 568.70/100 capsules
40-mg capsule 976.80/100 capsules
50-mg capsule 950.38/100 capsules
60-mg capsule 1137.38/100 capsules 1515.25/100
80-mg capsule capsules 1900.74/100 capsules
100-mg capsule 4946.40/100 capsules
200-mg capsule 12.29/100 mL
Roxanol 10-mg/5-mL solution 71.53/100 mL
20-mg/5-mL solution

ACCP Updates in Therapeutics® 2016: Ambulatory Care Pharmacy Preparatory Review and Recertification Course

1-461
Neurology

Table 22. Average Prices for Oral Medications Used for Pain (continued)
Brand Name Generic Name Dosage Form Average Wholesale Price, $
Subsys Fentanyl 100-mcg spray 1115.96/30 sprays
200-mcg spray 1762.86/30 sprays
400-mcg spray 2672.51/30 sprays
600-mcg spray 3553.10/30 sprays
800-mcg spray 4439.82/30 sprays
1200-mcg spray 3553.10/30 sprays
1600-mcg spray 4439.82/30 sprays
12-mcg patch 101.51/5 patches
Duragesic 25-mcg patch 72.10/5 patches
50-mcg patch 131.80/5 patches
75-mcg patch 201.05/5 patches
100-mcg patch 266.80/5 patches
Dilaudid Hydromorphone 2-mg tablets 49.88/100 tablets
4-mg tablets 72.81/100 tablets
8-mg tablets 131.93/100 tablets
1-mg/mL solution 189.12/473 mL
Demerol Meperidine 50-mg tablets 68.50/100 tablets
100-mg tablets 129.90/100 tablets
Dolophine Methadone 5-mg tablets 38.45/100 tablets
10-mg tablets 44.22/100 tablets
5 mg/5 mL 46.80/500 mL
10 mg/5 mL 86.40/500 mL
Roxicodone Oxycodone 5-mg tablet 54.11/100 tablets
10-mg tablet 62.50/100 tablets
15-mg tablet 189.48/100 tablets
20-mg tablet 110.30/100 tablets
30-mg tablet 358.93/100 tablets
5-mg capsule 184.90/100 capsules
5-mg/5-mL solution 142.50/500 mL
10-mg tablet 246.34/100 tablets
OxyContin 15-mg tablet 475.54/100 tablets
20-mg tablet 471.40/100 tablets
30-mg tablet 837.78/100 tablets
40-mg tablet 836.48/100 tablets
60-mg tablet 1460.66/100 tablets
Xartemis XR Oxycodone/ 80-mg tablet 1573.01/100 tablets
acetaminophen 7.5-mg/325-mg tablet 276.00/100 tablets

ACCP Updates in Therapeutics® 2016: Ambulatory Care Pharmacy Preparatory Review and Recertification Course

1-462
Neurology

Table 22. Average Prices for Oral Medications Used for Pain (continued)
Brand Name Generic Name Dosage Form Average Wholesale Price, $
Opana Oxymorphone 5-mg tablet 357.00/100 tablets
10-mg tablet 672.00/100 tablets
Opana ER 5-mg tablet 181.26/100 tablets
7.5-mg tablet 264.65/100 tablets
10-mg tablet 348.06/100 tablets
15-mg tablet 482.69/100 tablets
20-mg tablet 617.33/100 tablets
30-mg tablet 888.54/100 tablets
40-mg tablet 1159.78/100 tablets
Generic Codeine 15-mg tablet 57.73/100 tablets
30-mg tablet 62.15/100 tablets
60-mg tablet 113.83/100 tablets
30-mg/5-mL solution 128.44/500 mL
Zohydro Hydrocodone 10-mg tablet 442.08/60 tablets
15-mg tablet 472.32/60 tablets
20-mg tablet 487.08/60 tablets
30-mg tablet 502.56/60 tablets
40-mg tablet 517.68/60 tablets
50-mg tablet 540.00/60 tablets
5-mg/300-mg tablet 191.04/100 tablets
Vicodin Hydrocodone/ 7.5-mg/300-mg tablet 214.27/100 tablets
Vicodin ES Acetaminophen 10-mg/300-mg tablet 276.44/100 tablets
Vicodin HP 10-mg/300-mg/15-mL elixir 220.50/473 mL
Lortab 7.5-mg/650-mg tablet 130.45/100 tablets
Lorcet Plus 10-mg/650-mg tablet 183.54/100 tablets
Lorcet 10/650 5-mg/325-mg tablet 54.20/100 tablets
Norco 7.5-mg/325-mg tablet 61.82/100 tablets
10-mg/325-mg tablet 110.00/100 tablets
Vicoprofen Hydrocodone/ 2.5-mg/200-mg tablet 292.13/100 tablets
Ibuprofen 5-mg/200-mg tablet 310.11/100 tablets
7.5-mg/200-mg tablet 114.65/100 tablets
Ultram Tramadol 50-mg tablet 83.75/100 tablets
Ultram ER 100-mg tablet 423.16/100 tablets
200-mg tablet 699.84/100 tablets
300-mg tablet 976.44/100 tablets
Generic 150-mg capsule 4635.08/500 capsules
ConZip 100-mg capsule 287.00/30 capsules
200-mg capsule 376.10/30 capsules
300-mg capsule 520.22/30 capsules
ER/XR = extended release; IR = immediate release.
Red Book Online through Truven. New York: Thomas Reuters, 2015.

ACCP Updates in Therapeutics® 2016: Ambulatory Care Pharmacy Preparatory Review and Recertification Course

1-463
Neurology

IV.  MYASTHENIA GRAVIS (MG)

A. Epidemiology
1. Relatively rare: 10–20 new cases per million
2. Prevalence: 150–200 cases per million, which has increasing in the past 5 decades
3. Age at onset
a. Autoimmune juvenile MG: 10%–15% of total cases
b. Can occur in neonates
c. Sex distribution
i. Male individuals: Sixth to eighth decade of life
ii. Female individuals: Second and third decade of life

B. Pathophysiology
1. Decreased transmission at the neuromuscular junction, leading to weakness of muscle contraction
a. Caused by the formation of antibodies binding to ACh receptors (anti-AChR Abs) in the
postsynaptic muscle membrane
b. Seronegative patients (who lack anti-AChR Abs) may have anti–muscle-specific tyrosine kinase
(MuSK) antibodies; the agrin/MuSK signaling pathway maintains the structural and functional
integrity of postsynaptic neuromuscular junction; anti-MuSK antibodies affect the agrin-
dependent AChR cluster and reduce AChR numbers.

C. Types of MG
1. Ocular – Limited to eyelids and extraocular muscles
2. Generalized – Affects ocular muscles and a combination of bulbar, limb, and/or respiratory muscles

D. Clinical Presentations
1. Fluctuating weakness and fatigue of skeletal muscle
a. Worse in evening or after exercise
b. Improves with rest
2. Muscle fatigue caused by worsening contractile force
3. Presenting symptoms
a. Ptosis and diplopia in more than 50%
b. Bulbar symptoms – Dysarthria, dysphagia, fatigable chewing
c. Proximal limb weakness
d. Neck and respiratory muscle weakness

E. Treatment Goals
1. Goal is remission.
2. MG is unpredictable, but there are three stages of the disease.
a. Active: Most severe symptoms
b. Stable: Stable, but persistent symptoms that may worsen with infection or medication changes
c. Remission: Symptom free with or without medications

F. Treatment Options
1. Symptomatic therapy
a. Place in therapy: First-line treatment
b. Advantages/disadvantages: Rapid onset; do not alter disease progression or outcome
c. Pharmacologic agent(s): Pyridostigmine – ACh inhibitor, increases available ACh at the
neuromuscular junction

ACCP Updates in Therapeutics® 2016: Ambulatory Care Pharmacy Preparatory Review and Recertification Course

1-464
Neurology

2. Rapid immunotherapies
a. Place in therapy: Acute management of severe muscular weakness, myasthenic crisis, preoperative
treatment before thymectomy, intermittent treatment to maintain remission when MG not well
controlled by chronic immunotherapies
b. Advantages/disadvantages: Rapid onset, but effect is transient.
c. Pharmacologic agent/modality
i. Plasmapheresis: Directly removes anti-AChR Abs
ii. Intravenous immunoglobulins: Inhibit cytokine competition with autoantibodies; inhibit
complement deposition; interfere with binding of Fc receptor on macrophages and
immunoglobulin receptor on B cells; interfere with antigen recognition by sensitized T cells
3. Chronic immunotherapies
a. Place in therapy: Induce and maintain remission
b. Advantages/disadvantages: Adverse effects may limit use in some patients.
c. Pharmacologic agent(s)
i. Prednisone: Corticosteroid, several mechanisms, reduces anti-AChR Abs concentrations
ii. Azathioprine: Purine analog, interferes with T- and B-cell proliferation through reduction of
nucleic acid synthesis
iii. Mycophenolate mofetil: Blocks purine synthesis, suppresses T- and B-cell proliferation
iv. Cyclosporine: Blocks synthesis and release of interleukin-2 and interferes with proliferation of
CD4+ T cells
v. Tacrolimus: Inhibits T-cell activation; binds with FKBP-12 to form a complex that inhibits
calcineurin phosphatase activity
4. Surgery – Thymectomy
a. Place in therapy: Patients with thymoma
b. Advantages/disadvantages: May not be a viable therapeutic approach for anti-MuSK antibody−
positive patients

Table 23. Current Therapies for Myasthenia Gravis Management


Therapy Type/ Time to Onset Time to
Adverse Effect(s)
Specific Agent(s) of Effect Maximal Effect
Symptomatic therapy
Pyridostigmine 10–15 minutes 2 hours Abdominal cramping, diarrhea, increased
salivation and bronchial secretions,
nausea, sweating, bradycardia
Rapid immunotherapies
Plasmapheresis 1–7 days 1–3 weeks Hypotension, paresthesias, infections,
thrombotic complications related to
venous access, bleeding tendencies

IVIG 1–2 weeks 1–3 weeks Thrombosis caused by increased blood


viscosity (rare)

ACCP Updates in Therapeutics® 2016: Ambulatory Care Pharmacy Preparatory Review and Recertification Course

1-465
Neurology

Table 23. Current Therapies for Myasthenia Gravis Management (continued)


Therapy Type/ Time to Onset Time to
Adverse Effect(s)
Specific Agent(s) of Effect Maximal Effect
Chronic immunotherapies
Prednisone 2–3 weeks 5–6 months Hypertension, fluid retention, impaired
glucose tolerance, mood disturbances,
osteoporosis

Azathioprine 6–12 months 1–2 years Hepatotoxicity and leukopenia

Mycophenolate mofetil 4–12 months 1 year Nausea, vomiting, diarrhea, increased


risk of infections, myelosuppression

Cyclosporine 4–6 months 8–12 months Nephrotoxicity

Tacrolimus 4–6 months Not defined Induces/exacerbates diabetes mellitus


Surgery
Thymectomy 1–10 years 1–10 years Surgical complications
IVIG = intravenous immunoglobulin.

Patient Case

18. E.T. is a 50-year-old woman with MG of 2 years’ duration, with the chief complaint of neck and upper spine
pain, shoulder and upper arm weakness. During the past 6 months, she has had two admissions for acute
exacerbations of severe muscle weakness affecting her lower extremities and shortness of breath. She is
taking pyridostigmine 60 mg every 6 hours. The decision is made to add mycophenolate mofetil 500 mg
three times daily. Which of the following is a known adverse effect that is associated with mycophenolate
mofetil?
A. Alopecia.
B. Increased risk of infection.
C. Renal impairment.
D. Thrombocytosis.

Table 24. Average Prices for Oral Medications Used for Myasthenia Gravis
Generic Name Drug Name Dosage Form Average Wholesale Price, $
Mestinon 60-mg tablet 127.50/100 tablets
Pyridostigmine 60-mg/5-mL solution 1181.74/473 mL
Mestinon Timespan 180-mg tablet 241.78/30 tablets
1-mg tablet 19.42/100 tablets
2.5-mg tablet 16.54/100 tablets
Prednisone Deltasone 5-mg tablet 20.36/100 tablets
10-mg tablet 21.83/100 tablets
20-mg tablet 25.90/100 tablets

ACCP Updates in Therapeutics® 2016: Ambulatory Care Pharmacy Preparatory Review and Recertification Course

1-466
Neurology

Table 24. Average Prices for Oral Medications Used for Myasthenia Gravis (continued)
50-mg tablet 211.00/100 tablets
Imuran
Azathioprine 75-mg tablet 1496.40/100 tablets
Azasan
100-mg tablet 667.13/100 tablets
200-mg/mL powder 1101.40/160 mL
Mycophenolate mofetil CellCept 250-mg capsule 392.32/100 tablets
500-mg tablet 784.65/100 tablets
25-mg capsule 41.25/30 capsules
100-mg capsule 164.89/30 capsules
Cyclosporine Sandimmune 25-mg liquid capsule 41.26/30 liquid capsules
100-mg liquid capsule 164.88/30 liquid capsules
100-mg/mL solution 299.55/50 mL
0.5-mg capsule 222.98/100 capsules
Tacrolimus Prograf 1-mg capsule 445.95/100 capsules
5-mg capsule 2229.75/100 capsules
Red Book Online through Truven. New York: Thomas Reuters, 2015.

V.  MULTIPLE SCLEROSIS (MS)

A. Epidemiology
1. More than 400,000 Americans, 2.5 million worldwide
2. 200 new cases of MS are diagnosed each week in the United States.
3. Age at onset: Most people given a diagnosis of MS are between 20 and 50 years of age.
4. Sex distribution: Female/male risk ratio is approximately 2:1 to 3:1.

B. Pathophysiology
1. Etiology
a. Combination of genetics, altered immune system, environmental (e.g., measles, mumps, rubella,
Epstein-Barr virus, human herpes virus 6)
b. Increasing distance from the equator associated with increased risk of MS
2. Key physiologic changes
a. Stripping of the myelin sheath surrounding CNS axons
b. Associated with an inflammatory, perivenular infiltrate of T and B lymphocytes, macrophages,
antibodies, and complement
c. Demyelination renders axons susceptible to damage, becomes irreversible when they are severed.
d. Irreversible axonal damage correlates with disability; visualized as hypointense lesions, or
“black holes,” on T-weighted MRI.

C. Clinical Presentations and Course of Illness


1. Signs and symptoms
a. Primary: Caused by demyelination and axonal damage, reflects the area of the brain or spinal cord
that is damaged (e.g., urinary retention, ataxia, dysarthria, bladder or bowel dysfunction)
b. Secondary: Complications from primary symptoms (e.g., urinary tract infection)
c. Tertiary: Effect of disease on everyday life

ACCP Updates in Therapeutics® 2016: Ambulatory Care Pharmacy Preparatory Review and Recertification Course

1-467
Neurology

2. Course of illness
a. Relapsing-remitting MS (RRMS)
i. Attacks/exacerbations—New symptoms lasting at least 24 hours and separated from other
new symptoms by at least 30 days, followed by complete or incomplete remissions
ii. First attack: Clinically isolated syndrome (CIS); subsequent attacks also called relapses
iii. Radiologically isolated syndrome (RIS): Occurs in patients who have an MRI scan done for
another reason and have lesions consistent with MS
iv. New brain MRI lesions correlate with clinical attacks.
v. Attack frequency decreases over time and becomes independent of the development of
progressive disabilities.
vi. Neurologic recovery after an acute exacerbation is often quite good in the early phase but
tends to be less complete after repeated relapses.
b. Secondary-progressive MS (SPMS)
i. Progressive phase, exacerbations and remissions less difficult to identify, disability
accumulates significantly
ii. New brain MRI lesions are less common; brain atrophy and T1 holes increase.
c. Primary-progressive MS (PPMS)
i. Presents with symptoms (especially spastic paraparesis) that may worsen rapidly or relatively
slowly over time
ii. Accrue progressively more disability; worse prognosis than RRMS
d. Progressive-relapsing MS (PRMS): Mixture of both progression and relapses

D. Pharmacologic Treatment
1. Treatment of acute exacerbations: Shorten disease duration and possibly severity
a. Methylprednisolone
i. Route/dose: Intravenously/500–1000 mg/day for 3–5 days
ii. Adverse effects: Sleep disturbance, a metallic taste, GI upset (rarely), impaired blood glucose control
b. Plasma exchange: For patients with more severe attacks or whose condition is not responding to
intravenous corticosteroids
2. Disease-modifying therapies (DMTs): Alter the course of the illness, diminish progressive disability
over time (see Table 25)
3. Symptomatic therapy: Maintain quality of life (see Table 26)

ACCP Updates in Therapeutics® 2016: Ambulatory Care Pharmacy Preparatory Review and Recertification Course

1-468
Table 25. Pharmacologic Agents for Disease-Modifying Therapy in MS
Efficacy
Pharmacologic
Mechanism of Action (% decrease compared Route/Dose Safety Considerations Monitoring
Agent/Indications
with placebo)
ARR IM (Avonex):
Avonex – 32% 7.5 mcg weekly,
Rebif 22 titrating up
mcg – 29% 7.5 mcg weekly to
Interferon β-1a Rebif 44 30 mcg weekly
(Avonex, Rebif) mcg – 32%
SC (Rebif):
Betaseron – 28% AEs: Flulike symptoms,* fatigue,
Avonex: 4.4 mcg three
Reduce activation and EDSS times weekly x injection
CIS, RRMS
entry of T cells into the Avonex – 37% 2 wk, 11 mcg site reactions,
CNS, reduce adhesion Rebif 22 three times LFT abnormalities, depression
Rebif: RRMS CBC and LFTs
molecules and helper T mcg – 22% weekly x *Preinjection and postinjection every 6 months
cell proinflammatory Rebif 44 2 wk, 22–44 acetaminophen or NSAIDs may
cytokines, and reduce mcg – 30% mcg three times reduce flu-like symptoms.
matrix metalloproteinases Betaseron – 24% weekly

1-469
Neurology

Gd-MRI FDA pregnancy category C


SC: 62.5 mcg
Avonex – 35%
Interferon every other day,
Rebif 22
β-1b (Betaseron) increasing by
mcg – 67%
62.5 mcg/
Rebif 44
CIS, RRMS 2 wk to 250 mcg
mcg – 78%
every other day
Betaseron – 63%
Analog of MS- associated AEs: Injection site reactions
antigen (MBP); binds (immediate): Pain, flushing, chest
Glatiramer acetate MHC class II molecules ARR: 29% pain, rapid heartbeat, noncardiac
SC: 20 mg daily shortness of
(Copaxone) on peripheral MBP-
EDSS: 28% or 40 mg three breath, anxiety, tightness in throat
recognizing APCs,
times weekly (symptoms typically remit in about 15
CIS, RRMS promotes shift to Gd-MRI: N/A
type 2 helper T cell minutes); lipoatrophy
inflammatory profile FDA pregnancy category B

ACCP Updates in Therapeutics® 2016: Ambulatory Care Pharmacy Preparatory Review and Recertification Course
Table 25. Pharmacologic Agents for Disease-Modifying Therapy in MS (continued)
Efficacy
Pharmacologic
Mechanism of Action (% decrease compared Route/Dose Safety Considerations Monitoring
Agent/Indications
with placebo)
Prior initiation:
CBC, varicella
zoster antibody,
LFT
AEs: Bradyarrhythmia, AV block,
nasopharyngitis, dyspnea, headaches, First dose
S1P analog; reduces diarrhea, nausea, skin cancer, macular monitoring
Fingolimod ARR: 40%–62% edema, increased liver enzymes
autoaggressive (6 hours):
(Gilenya)
lymphocytes in REMS ECG before dose and
EDSS: 15%–40% PO: 0.5 mg daily
circulation and CNS by at 6 hours, hourly
Relapsingforms of Varicella zoster vaccination (if
down- regulation of SIP HR and BP, observe
MS Gd-MRI: 35%–74% antibody negative); avoid live,
receptor for bradycardia/
attenuated vaccine bradyarrhythmia
FDA pregnancy category C

1-470
Ophthalmology
Neurology

examination at
baseline and 3–4
months after
AEs: Headache, back pain, infusion
Selective adhesion
reaction,
molecule inhibitor;
ARR: 68% anaphylactoid reaction (rare),
Natalizumab inhibits leukocyte
immune reconstitutioninflammatory CBC and LFTs every
(Tysabri) adhesion and migration IV: 300 mg
EDSS: 41% syndrome, PML 6 months, anti- JCV
Relapsing forms across blood-brain barrier every 4 weeks
antibody test
of MS by blocking adhesion Black box warning: PML TOUCH
Gd-MRI: 83%
molecule very late program (REMS)
antigen-4
FDA pregnancy category C

ACCP Updates in Therapeutics® 2016: Ambulatory Care Pharmacy Preparatory Review and Recertification Course
Table 25. Pharmacologic Agents for Disease-Modifying Therapy in MS (continued)
Efficacy
Pharmacologic
Mechanism of Action (% decrease compared Route/Dose Safety Considerations Monitoring
Agent/Indications
with placebo)
Attenuates
proinflammatory stimuli;
AEs: Diarrhea, cramps,
Dimethyl fumarate therefore, neuroprotection
ARR: 44%–53% PO: 120 mg nausea, and flushing*
(Tecfidera) – Antioxidant and anti-
twice daily for 7
inflammatory effects EDSS: 21%–38% CBC every 6 months
days; then 240 mg *Administer with food
Relapsing forms mediated through nuclear
Gd-MRI:57%–85% twice daily
of MS factor
FDA pregnancy category C
E2–related factor
signaling pathway
AEs: Nasopharyngitis, alopecia,
nausea,
limb pain, diarrhea, arthralgia,
neutropenia, agranulocytosis,
pancytopenia, and thrombocytopenia Prior initiation: LFT,

1-471
CBC,
Neurology

Teriflunomide Inhibits pyrimidine ARR: 31% Black box warning:Contraindicated pregnancy test,
(Aubagio) synthesis, therefore in pregnancy/women of childbearing tuberculin skin test
PO: 7–14 mg
preventing the EDSS: 24%–30% age not using reliable contraception; to exclude latent
once daily
Relapsing forms proliferation of T cells hepatotoxicity tuberculosis
of MS and B cells Gd-MRI:17%–31%
Accelerated elimination with LFT monthly for
cholestyramine or activated charcoal first 6 months
if drug discontinuation required
Avoid live, attenuated vaccine
FDA pregnancy category X

ACCP Updates in Therapeutics® 2016: Ambulatory Care Pharmacy Preparatory Review and Recertification Course
Table 25. Pharmacologic Agents for Disease-Modifying Therapy in MS (continued)
Efficacy
Pharmacologic
Mechanism of Action (% decrease compared Route/Dose Safety Considerations Monitoring
Agent/Indications
with placebo)
AEs: Rash, headache, pyrexia,
nasopharyngitis, nausea, urinary
tract infection, fatigue, insomnia,
upper respiratory tract infection,
herpes viral infection, urticaria, Before use and
IV for 4 hours
Binds to the cell surface pruritus, thyroid gland disorders, monthly intervals
for 2 treatment
antigen CD52 on the fungal infection, arthralgia, pain thereafter:
courses:
surface of B and T in extremity, back pain, diarrhea, CBC with
Alemtuzumab lymphocytes, monocytes, First course: sinusitis, oropharyngeal pain, differential, SCr,
ARR: 49%–54%*
(Lemtrada) macrophages, and 12 mg/day on 5 paresthesia, dizziness, abdominal urine analysis
natural killer cells, EDSS: 28%–42%* consecutive days pain, flushing, vomiting
Thyroid function
Relapsing forms which is followed by
Gd-MRI: N/A Second course: Black Box Warning: Fatal (TSH) at baseline
of MS an antibody-dependent
12 mg/day on 3 autoimmune conditions, infusion and every 3 months
cellular cytolysis and

1-472
consecutive days reactions, malignancies
complement-mediated Skin examination
Neurology

12 months after
lysis REMS program to monitor for
first course
melanoma yearly
Varicella zoster vaccination (if
antibody negative); avoid live,
attenuated vaccine
FDA pregnancy category C
*Compared to INF β-1a SC.
AE = adverse effect; APC = antigen-presenting cell; ARR = annualized relapse rate; AV = atrioventricular; BP = blood pressure; CBC = complete blood cell count; CIS = clinically isolated syndrome; CNS = central
nervous system; ECG = electrocardiogram; EDSS = Expanded Disability Status Scale; FDA = U.S. Food and Drug Administration; Gd-MRI = gadolinium-enhanced magnetic resonance imaging (T2 lesions);
HR = heart rate; IFN = interferon; IM = intramuscular(ly); IV = intravenous(ly); JCV = JC virus; LFT = liver function test; MBP = myelin basic protein; MHC = major histocompatibility complex; MS = multiple
sclerosis; N/A = not applicable; NSAID = nonsteroidal anti-inflammatory drug; PML = progressive multifocal leukoencephalopathy; PO = orally; REMS = Risk Evaluation and Mitigation Strategies; RRMS =
relapsing-remitting MS; SC = subcutaneous(ly); SCr = serum creatinine concentration; S1P = sphingosine-1- phosphate; SC = subcutaneous; TSH = thyroid-stimulating hormone; wk = week(s).
Micromedex 2.0 [Internet version]. Greenwood Village, CO: Truven Health Analytics. Available at www.micromedexsolutions.com/. Accessed January 20, 2014.
Aubagio [package insert]. Cambridge, MA: Genzyme, 2012.
Gilenya [package insert]. Greenville, NC: GlaxoSmithKline, 2011.
Adapted from: Sanford M, Lyseng-Williamson KA. Subcutaneous recombinant interferon-β-1a (Rebif): a review of its use in the treatment of relapsing multiple sclerosis. Drugs 2011;71:1865-91 (Table VII).
Adapted from: Damal K, Stoker E, Foley JF. Optimizing therapeutics in the management of patients with multiple sclerosis: a review of drug efficacy, dosing, and mechanisms of action. Biologics Targets Ther

ACCP Updates in Therapeutics® 2016: Ambulatory Care Pharmacy Preparatory Review and Recertification Course
2013;7:247-58 (Table 2).
Neurology

Patient Cases

Questions 19 and 20 pertain to the following case.


M.B. is a 33-year-old woman with recently diagnosed CIS. Her presenting symptom was optic neuritis. An MRI
shows two white matter lesions in the brain. Her mother had a diagnosis of MS.

19. Which is the most appropriate therapy currently?


A. Glatiramer acetate.
B. Teriflunomide.
C. Dimethyl fumarate.
D. Fingolimod.

20. M.B. has been receiving a first-generation DMT for 3 years. Initially, the first-generation DMT worked well,
and she reported no relapses; however, during the past year, she has reported four attacks. An MRI reveals
several new lesions, and she has missed several weeks of work. Which is the best DMT to recommend
currently (assuming all will be effective)?
A. Mitoxantrone.
B. Interferon β-1a.
C. Teriflunomide.
D. Natalizumab.

Table 26. Pharmacologic Agents for Symptomatic Management in MS


Pharmacologic
Route/Dose Clinical Use in MS Adverse Effects
Agent
Insomnia, flushing, metallic
Amantadine PO: 100–200 mg/day Fatigue taste, fluid retention, electrolyte
abnormalities, hyperglycemia
Insomnia, nightmares, dizziness,
Modafinil PO: 200–400 mg/day Fatigue nausea, livedo reticularis, orthostatic
hypotension, peripheral edema
PO: 5–30 mg/day
Transdermal (3.9 mg/24
Oxybutynin Bladder dysfunction Insomnia, headache, nausea, asthenia
hour):
1 patch, every 3–4 days
Dry mouth, constipation, decreased
Tolterodine PO: 2–8 mg/day Bladder dysfunction
sweating, drowsiness, headache

Dry mouth, headache, dyspepsia,


Desmopressin PO: 0.2 mg at bedtime Nocturia
constipation, dizziness, blurred vision
PO: 25–100 mg Flushing, headache, nausea,
Sildenafil about 1 hour before sexual Erectile dysfunction hypotension, hypertension,
activity tachycardia, hyponatremia

ACCP Updates in Therapeutics® 2016: Ambulatory Care Pharmacy Preparatory Review and Recertification Course

1-473
Neurology

Table 26. Pharmacologic Agents for Symptomatic Management in MS (continued)


Pharmacologic
Route/Dose Clinical Use in MS Adverse Effects
Agent
Headache, flushing, diarrhea,
PO: 5–80 mg/day, usually dyspepsia, rash, visual dysfunction,
Baclofen Spasticity
in three divided doses myocardial infarction, priapism,
abrupt withdrawal can cause seizures
PO: 2–36 mg/day (every
Fatigue, somnolence, muscle
Tizanidine 6–8 hours as needed, up to Spasticity
weakness/hypotonia, nausea, seizure
three times daily)
Dry mouth, somnolence, mild
hypotension, constipation, nausea,
PO: 200–1200 mg/
Pain (trigeminal neuralgia/ muscle weakness, abnormal LFT
Carbamazepine day, in two or three divided
chronic central pain) results, blurred vision or diplopia,
doses
rash, hyponatremia caused by
SIADH, osteoporosis
Fatigue, sedation, nausea, rash,
dizziness, blurred vision or diplopia,
PO: 100–600 mg Pain (trigeminal neuralgia/ rash, hyponatremia caused by
Oxcarbazepine
twice daily chronic central pain) SIADH; rarely, Stevens-Johnson
syndrome or aplastic anemia/bone
marrow suppression, osteoporosis
Pain (trigeminal neuralgia/ Fatigue, sedation, nausea, rash,
Phenytoin PO: 300 mg twice daily chronic central pain), dizziness, gingival hyperplasia,
paroxysmal spasms osteoporosis, hirsutism, ataxia
Pain (trigeminal neuralgia/ Fatigue, ataxia, dizziness, sedation,
Gabapentin PO: 100–600 mg/day chronic central pain), par- nausea, myoclonus, headache, weight
oxysmal spasms, spasticity gain, pedal edema, irritability
Pain (trigeminal neuralgia/
chronic central pain),
Pregabalin PO: 75–150 mg twice daily Same as for gabapentin
paroxysmal spasms,
spasticity
Ataxia, nausea, dizziness,
PO: 100–1200 mg
Duloxetine Neuropathic pain somnolence, emotional lability,
three times daily
blurred vision/diplopia, edema
Nausea, dry mouth, constipation,
insomnia, dysuria, dizziness,
decreased appetite, gastritis,
Donepezil PO: 20–60 mg/day Memory function
hypertension; increased risk of
suicidal ideation, worsening of
depression
Urinary tract infection, insomnia,
Dalfampridine PO: 10 mg twice daily Improve walking dizziness, headache, nausea,
vomiting, weakness, seizures
LFT = liver function test; MS = multiple sclerosis; PO = oral; SIADH = syndrome of inappropriate secretion of antidiuretic.

ACCP Updates in Therapeutics® 2016: Ambulatory Care Pharmacy Preparatory Review and Recertification Course

1-474
Neurology

Table 27. Average Prices for Medications (DMT) Used for Multiple Sclerosis
Average Wholesale
Brand Name Generic Name Dosage Form
Price, $
Betaseron IFN β-1b (SC) 0.3-mg kit 6777.46/kit
Extavia 0.3-mg kit 6057.83/kit
Avonex IM IFN β-1a 30-mcg/0.5-mL kit 6403.20/kit
Rebif SC IFN β-1a 44-mcg/0.5-mL solution 6911.66/12 syringes
22-mcg/0.5-mL solution 6911.66/12 syringes
Copaxone Glatiramer acetate 20-mg/mL solution 7332.60//30 syringes
40-mg/mL solution 6009.60/12 syringes
Gilenya Fingolimod 0.5-mg capsule 6997.46/30 capsules
Tysabri Natalizumab 20-mg/mL solution 6309.60/15 mL
Tecfidera Dimethyl fumarate 120 mg 1620.00/14 capsules
240 mg 6945.60/60 capsules
Aubagio Teriflunomide 7 mg 6459.50/28 tablets
14 mg 6459.50/28 tablets
Lemtrada Alemtuzumab 10-mg/mL solution 23700.00/1.2 mL
DMT = disease-modifying therapy; IFN = interferon; IM = intramuscular; SC = subcutaneous.
Red Book Online through Truven. New York: Thomas Reuters, 2015.

VI.  SPINAL CORD INJURIES (SCIs)

A. Epidemiology
1. Annual incidence of 4 per 100,000 with around 250,000 survivors
2. Prevalence: Estimated at 3–4 times greater than the annual incidence
3. An estimated 40% are nontraumatic myelopathies.

B. Pathophysiology
1. A wide variety of processes can cause injury to the spinal cord.
2. Chronic myelopathy caused by the following:
a. Degenerative disorders of the spinal canal with impingement on the spinal cord
b. Vascular diseases
c. Metabolic disorder
d. Infections
e. Benign and malignant tumors
f. Immunologic/inflammatory disease

C. Common Complications Caused by SCIs


1. Cardiopulmonary
a. Autonomic dysreflexia
i. Associated with SCI above T6
ii. Uninhibited sympathetic responses lead to vasoconstriction/HTN – Severe HTN potentially
life-threatening.

ACCP Updates in Therapeutics® 2016: Ambulatory Care Pharmacy Preparatory Review and Recertification Course

1-475
Neurology

iii. Common precipitating stimuli: Bladder distention, bowel impaction, pressure sores, occult
bone fractures, visceral disturbances
iv. Acute management: Sit patient upright to lower BP orthostatically, identify and eliminate
causative stimulus, use short-acting antihypertensive agents with rapid onset of action if severe.
v. See Acute Management of Autonomic Dysreflexia Clinical Practice Guidelines from
Consortium for Spinal Cord Medicine.
b. Coronary artery disease (CAD)
i. Decreased muscle mass, increased fat, and inactivity increase the risk of CAD.
ii. Management strategy: Control risk factors, statins if indicated.
iii. See the Adult Treatment Panel IV clinical practice guidelines.
c. Pulmonary disorders
i. Cervical and high thoracic myelopathies affect respiratory muscles.
ii. Increased risk of pneumonia, deep vein thrombosis, and pulmonary embolism
iii. See the current Infectious Diseases Society of America (IDSA) and/or CHEST guidelines.
2. Urinary tract
a. Bladder dysfunction
i. Myelopathies often produce bladder dysfunction or neurogenic bladder.
ii. Management strategy: Intermittent self-catheterization if voluntary voiding not possible,
anticholinergics and/or α-blockers
iii. See Bladder Management for Adults with Spinal Cord Injury from Consortium for Spinal
Cord Medicine.
b. Urinary tract infections
i. Common in all SCIs. Catheterization increases risk of infection.
ii. Common treatment: Antibiotics
iii. See the current IDSA guidelines.
3. Immobility or reduced mobility
a. Contractures
i. Reorganization of periarticular collagen tissue
ii. No pharmacologic treatment
b. Repetitive motion injuries
i. Overuse of arms or other compensatory activities
ii. No pharmacologic treatment
c. Osteoporosis
i. Increased risk of leg or hip fractures occurring below the SCI
ii. Common treatment: Bisphosphonates
iii. See the current National Osteoporosis Foundation (NOF) clinical practice guidelines.
d. Heterotopic ossification
i. Deposition of bone with soft tissue occurs in up to half of traumatic SCIs.
ii. Common treatment: NSAIDs for inflammation
e. Pressure ulcers
i. Tissue damage caused by unrelieved pressure occurring over bony prominences.
ii. No pharmacologic treatment
f. Spasticity
i. Increased deep tendon reflexes, augmented cutaneous and autonomic reflexes, involuntary
spasm, or clonus
ii. Common treatment: Baclofen, tizanidine, diazepam, or dantrolene
iii. See Cochrane Review for pharmacologic interventions for spasticity after an SCI.

ACCP Updates in Therapeutics® 2016: Ambulatory Care Pharmacy Preparatory Review and Recertification Course

1-476
Neurology

g. Pain
i. An estimated 40% of patients will experience neurogenic or neuropathic pain.
ii. Common treatment: Depends on severity of pain
iii. See above pain section (III. PAIN).

D. Clinical Presentations
1. Level of injury dictates which bodily functions are altered or lost.
2. Damage can cause changes in movement, feeling, bladder control, or other bodily functions.

E. Treatment Goals
1. Goal is management of complications.
2. Complications will be patient-specific and will vary.

VII.  ALZHEIMER DISEASE (AD)

A. Epidemiology
1. Most common cause of dementia in the United States; currently affects 5 million adults
2. Incidence
a. Annually, 1% of adults between the age of 60 and 70 years
b. 6%–8% of adults 85 years and older
3. Prevalence
a. Affects 10% of adults 60–70 years and older
b. Affects almost 50% of adults 85 years and older

B. Etiology and Risk Factors


1. Advanced age
2. Family history of AD
3. Genetics
a. β-Amyloid
i. Associated with early-onset disease (before age 65 years)
ii. Associated with senile plaques
iii. Overproduction linked to chromosomes 1, 14, 21
iv. Amyloid precursor protein (APP) is encoded on chromosome 21, presenilin-1 is related to a gene
mutation on chromosome 14, and presenilin-2 is related to gene mutation on chromosome 1.
b. Apolipoprotein E (APOE)
i. Late-onset disease (after age 65 years)
ii. Associated with neurofibrillary tangles
iii. Mutation associated with chromosome 19; three variants: E2, E3, and E4
(a) E3 normal variant
(b) E4 associated with increased risk of AD
(1) Heterozygotes 2- to 4-fold risk
(2) Homozygotes 5- to 19-fold risk
(c) E2 protective for AD (rare)

ACCP Updates in Therapeutics® 2016: Ambulatory Care Pharmacy Preparatory Review and Recertification Course

1-477
Neurology

c. Tau – Soluble protein that supports microtubule function


i. Hyperphosphorylated tau (insoluble) is associated with neurofibrillary tangles.
ii. Aggregates of abnormal tau are cytotoxic.
iii. Mutations on chromosome 17 are associated with dementia in PD; however, no tau mutations
specifically noted in AD.
d. Cholinergic hypothesis – Loss of choline-acetyl transferase and cholinergic neurons; basis for
currently available therapies (cholinesterase inhibitors); it has become clear, based on the results
of clinical trials of medications that specifically target acetylcholine, that this hypothesis does not
adequately explain the underlying pathophysiology of AD.
4. Environmental factors – Limited evidence to support

C. Clinical Presentation
1. Cognition
a. Mild cognitive impairment (MCI)
i. Preclinical AD
ii. Memory impairment that may be noticeable to others but that does not interfere with daily
life; not severe enough to meet the definition of dementia
iii. Patients with MCI develop AD at a higher rate; however, not all patients with MCI go on to
develop dementia.
iv. Lack of clear evidence to recommend the use of medications to slow the progression of MCI
to dementia
b. Cognitive loss in AD
i. Insidious onset; chronic gradual loss of memory and other cognitive abilities
ii. Inability to retain new information; remote memory spared until late disease
iii. Affects many areas of cognition
(a) Language
(b) Abstract reasoning
(c) Executive function
(d) Decision-making
iv. Of sufficient severity to affect daily life, work, social interactions
2. Activities of daily living (ADLs) – Typically occurs later in the course of disease compared with
cognitive loss
a. Gradual loss of ability to perform self-care activities
i. Bathing
ii. Toileting
iii. Feeding
iv. Dressing
v. Transfer
vi. Ambulation
b. Instrumental ADLs
i. Managing finances
ii. Managing medications
iii. Cooking
iv. Shopping
v. Using telephone

ACCP Updates in Therapeutics® 2016: Ambulatory Care Pharmacy Preparatory Review and Recertification Course

1-478
Neurology

3. Behaviors and psychiatric symptoms


a. Early disease
i. Depression
ii. Anxiety
iii. Less commonly, delusions or hallucinations
b. Later disease
i. Delusions/hallucinations
ii. Wandering
iii. Agitation or aggression – Frequent cause for nursing home placement

Patient Case

21. D.T. is a 76-year-old widowed woman in the clinic today, accompanied by her daughter, for evaluation of cognitive
concerns. D.T. has a history of osteoarthritis, hypertension, and atrial fibrillation. The daughter states that D.T. has
had difficulties with her memory for almost a year. Initially, the symptoms were minor; D.T. would forget names
or recent events, but more recently, the memory concerns have been more severe, and D.T. is becoming less able
to manage at home alone, according to her daughter. D.T. states that she does not think she has memory problems.
There is no known history of AD in their family. D.T.’s father died of a stroke, and her mother died of colon
cancer. There is no recent history of falls, head trauma, or substance abuse. On evaluation today, the neurologic
examination is normal. An MMSE is performed, and D.T. scores 22/30. She has a 12th-grade education. Her score
on the Geriatric Depression Scale is 2/30. Blood is drawn for laboratory testing, and D.T. is scheduled for a CT scan.
Which best describes the findings observed in this case?
A. Pseudodementia.
B. Alzheimer disease.
C. Multi-infarct dementia.
D. Cognitive impairment.

D. Evaluation and Diagnosis


1. Medical history and physical examination: Optimally obtained from a family member or caregiver
a. Family history of dementia
b. Head injuries, falls
c. Alcohol or substance abuse
d. Depression (pseudodementia)
e. Acute illness (delirium)
f. Medication review
g. Language impairment
h. Extrapyramidal signs (tremor, rigidity, bradykinesia)
i. Focal weakness, gait disturbances
2. Differential diagnosis
a. Cerebrovascular disease and vascular brain injury (vascular dementia or multi-infarct dementia) –
Usually, there is evidence of cerebrovascular disease or vascular brain injury in patients who die of
AD. Difficult to distinguish relative contribution of AD or vascular injury to cognitive impairment
in an individual
b. Lewy body dementia – Form of dementia related to protein deposits (α-synuclein) in the brain
(Lewy bodies) that are identified postmortem. Patients with this type of dementia can have
hallucinations as an early symptom, but they may also have cognitive deficits, PD-like movement
abnormalities, sleep disturbances, and attention fluctuation.

ACCP Updates in Therapeutics® 2016: Ambulatory Care Pharmacy Preparatory Review and Recertification Course

1-479
Neurology

c. Parkinson disease (see Parkinson Disease section)


d. Normal pressure hydrocephalus – A form of hydrocephalus that accounts for a small portion (less
than 5%) of dementias. The syndrome is often characterized by a triad of symptoms, including
cognitive impairment, urinary or fecal incontinence, and gait apraxia; identifiable by CT or
magnetic resonance imaging (MRI); treatment involves placement of a ventriculoperitoneal shunt
to route excessive cerebrospinal fluid out of the brain ventricles.
e. Mixed dementia – Usually, mixed dementia includes characteristic changes or pathology consistent
with AD and vascular dementia; however, it may not always be the case.
f. Pick disease or frontotemporal dementia is a relatively rare form of dementia, but it includes
neurodegeneration that primarily affects the frontotemporal regions of the brain. Patients with
this type of dementia usually have difficulty with speech and thinking, personality changes,
and disinhibition. With this type of dementia, the personality changes can precede the memory
changes, helping distinguish it from AD. Pick disease is often used as a general term to describe
frontotemporal dementia, but it really describes the presence of tau protein accumulations (Pick
bodies) in the affected parts of the brain.
g. Huntington disease is a genetic neurodegenerative disorder associated with cognitive decline, loss
of muscle coordination, and psychiatric symptoms. Symptom onset may be earlier in life (20s,
30s, 40s), but it can occur at any point. Random jerky movements (chorea) are typically the initial
symptom. Cognition changes will appear later, and they are progressive in nature.
h. Other causes, including “reversible” causes
i. Depression
ii. Thyroid disease (specifically hypothyroidism)
iii. Vitamin deficiencies (B12, folate); some recent evident to suggest Vitamin D deficiency is
associated with an increased risk of AD
iv. Drug-induced cognitive impairment (Table 28)
(a) Numerous medications can cause cognitive impairment or make cognition worse in
patients who have cognitive issues at baseline.
(b) Anticholinergic medications classically cause cognitive impairment, but several other
medications/mechanisms can produce cognitive changes if acting in the CNS.
(c) Studies suggest that about 10% of patients who present to memory impairment clinics
have symptoms caused, at least in part, by medication-related effects.
(d) If a medication is thought to be contributing to cognitive impairment, it should be
discontinued, or at a minimum, the dose should be reduced and reevaluated closely.
v. Substance abuse
vi. CNS infections
3. Diagnosis
a. AD is usually considered “probable” AD, after other possible causes of cognitive impairment have
been ruled out; it is usually based on presentation and history because there is no definitive test to
diagnose AD. A diagnosis of “definite” AD has been reserved for histopathologic evidence from a
biopsy or autopsy confirming the diagnosis
b. DSM-IV (Diagnostic and Statistical Manual of Mental Disorders) criteria
i. The development of several cognitive deficits manifested by the following:
(a) Memory impairment
(b) One or more of the following cognitive disturbances: Aphasia, apraxia, agnosia, impaired
executive functioning
ii. The cognitive deficits cause considerable impairment in social or occupational functioning
and represent a decline from previous functioning.
iii. The onset is gradual, with continuing cognitive decline.

ACCP Updates in Therapeutics® 2016: Ambulatory Care Pharmacy Preparatory Review and Recertification Course

1-480
Neurology

iv. The cognitive deficits cannot be attributable to other causes (e.g., medications, stroke, tumors,
vitamin deficiencies).
v. The deficits are not caused by delirium.
c. The National Institute of Neurological and Communicative Disorders and Stroke (NINCDS) and
the Alzheimer’s disease and Related Disorders Association (ADRDA) criteria, published in 1984,
have long been used as a basis for AD diagnosis. In May 2011, the National Institute on Aging and
the Alzheimer’s Association jointly updated the NINCDS-ADRDA criteria.

Table 28. Medications That May Contribute to Impaired Cognition (not exhaustive)
Anticholinergic Effects Benzodiazepines Other CNS Effects
Amitriptyline Clorazepate Carisoprodol
Benztropine Chlordiazepoxide Chlorzoxazone
Chlorpheniramine Diazepam Cimetidine
Cyclobenzaprine Flurazepam Clonidine
Darifenacin Lorazepam Esomeprazole
Desipramine Oxazepam Guanethidine
Dicyclomine Temazepam Guanadrel
Diphenhydramine Indomethacin
Doxepin Meperidine
Fesoterodine Methocarbamol
Hydroxyzine Omeprazole
Imipramine Pantoprazole
Nortriptyline Phenobarbital
Oxybutynin Propoxyphene
Solifenacin Reserpine
Thioridazine
Tolterodine
Trospium
CNS = central nervous system.

4. Neuroimaging: CT or MRI – Most useful for ruling out other causes such as stroke or tumor
a. Not diagnostic for AD
b. Yield is relatively low, but more likely to find structural lesions if:
i. Younger than 60 years
ii. Neurologic signs or symptoms
iii. Rapid decline
iv. Underlying risks (anticoagulant use, malignancy)
5. Laboratory testing to identify underlying factors that may be contributing to symptoms
a. Complete blood cell count
b. Thyroid-stimulating hormone
c. Vitamin B12, folate
d. Calcium
e. Liver and kidney function
f. Electrolytes, glucose
g. Serologic testing for neurosyphilis (select individuals)
6. Assessment instruments
a. Mini-Mental State Examination (MMSE) (most common test used in clinical practice)
i. Screening tool for cognitive impairment; not intended to diagnose AD/dementia

ACCP Updates in Therapeutics® 2016: Ambulatory Care Pharmacy Preparatory Review and Recertification Course

1-481
Neurology

ii. Range of scores 0–30; 0 is worst


(a) 24–30 considered normal
(b) 17–23 considered mild impairment
(c) 10–16 considered moderate impairment
(d) Less than 10 considered severe impairment
iii. Educational bias – For patients with low levels of education, a maximum score of 30 may not
be feasible; consequently, depending on the level of education, a lower score may not suggest
cognitive impairment; conversely, if a patient has a very high level of education, a score of 27
or 28 may indicate cognitive changes.
b. Alzheimer Disease Assessment Scale (ADAS, ADAS-cog)
i. Longer, more comprehensive test, relative to MMSE; mostly used in research
ii. Language and memory skills
iii. ADAS often used for drug studies to evaluate change over time
(a) The FDA recognizes it as a primary indicator of drug response.
(b) ADAS-cog = Cognitive subscale
iv. ADAS-cog range of scores 0–70; 70 is the worst
(a) MMSE score of 24 = ADAS-cog score of 14–15
(b) ADAS-cog score = 72.2 − (2.41 × MMSE score)
c. Neuropsychiatric Inventory (NPI)
i. Assessment of psychiatric and behavioral symptoms; most commonly in research
ii. Twelve disturbances common in dementia: Delusions, hallucinations, agitation, dysphoria,
anxiety, apathy, irritability, euphoria, disinhibition, aberrant motor behavior, nighttime
behavior disturbances, and eating disturbances
iii. Range of scores: 0–144, higher score is associated with worse symptoms; frequency of
12 symptoms rated 1–4, and severity of symptoms rated 1–3; product of frequency × severity
and summed for the 12 symptoms gives total score.
d. Clinician Interview-Based Impression of Change (CIBIC)
i. Global subjective assessment by provider; may include caregiver assessments (CIBIC-Plus);
mainly used for research purposes
ii. Scored 1–7; score of 1 = very much improved; 2 = much improved; 3 = minimally improved;
4 = no change; 5 = minimally worse; 6 = moderately worse; 7 = markedly worse
e. Severe impairment battery
i. Assessment of cognitive function in those with severe neurologic impairment
ii. Composed of simple one-step commands presented together with gestures
iii. Assesses several cognitive areas including attention, orientation, language, memory, visual-
spatial abilities, construction, praxis, and social interaction
f. Geriatric Depression Scale
i. Depression scale developed specifically for geriatric patients
ii. Thirty questions, self-rated; also available in short form with 15 questions
iii. Higher score indicates depressive symptoms; score of 20/30 or 5/15 indicates depression.

E. Clinical Management
1. Treatment goals
a. Improve quality of life.
b. Maximize/maintain functional status and independence.
c. Maintain/enhance cognitive status.
d. Minimize mood and behavioral problems.
e. Minimize safety hazards (driving, cooking, wandering).

ACCP Updates in Therapeutics® 2016: Ambulatory Care Pharmacy Preparatory Review and Recertification Course

1-482
Neurology

2. Nonpharmacologic therapy
a. Group support (i.e., Alzheimer’s Association – www.alz.org)
b. Patient, family, caregiver education
i. Expectations
ii. Planning
iii. Realistic goals
c. Physical and mental activities, including aerobic exercise, socialization, cognitive activities such
as crossword puzzles, and reading
d. Avoid inappropriate medications (see Table 28).

Patient Case

22. The daughter of an 81-year-old woman with AD asks the physician to begin treating the patient with a drug
for her difficulties with memory. Her mother was given a diagnosis of probable AD 4 years ago and is now in
a nursing home; she can perform some of her ADLs, but only with assistance. Her most recent MMSE score
was 14/30. She has been admitted to the emergency department twice in the past 6 months for bradycardia
secondary to sick sinus syndrome. Which is the safest AD treatment to recommend for this patient?
A. Donepezil.
B. Memantine.
C. Rivastigmine.
D. Galantamine.

3. Pharmacotherapy
a. Cholinesterase inhibitors
i. Block the esterase-mediated metabolism of acetylcholine to choline and acetate in the
synaptic cleft = Increased acetylcholine availability to bind at postsynaptic muscarinic
receptors
ii. Specific agents – See Table 30.
(a) Donepezil
(b) Rivastigmine
(c) Galantamine
(d) Tacrine – More for historical interest; still available as brand name only, but not
marketed. Very poorly tolerated from a gastrointestinal (GI) standpoint and associated
with liver toxicity
(e) Combination of memantine 14 mg/donepezil 10 mg ER (for severe renal impairment—
CrCL 5–29 mls/min) and memantine 28 mg/donepezil 10 mg ER capsules given daily—
can be opened and sprinkled on food if necessary. You must be stable on memantine (14
(renal impairment) or 28 mg) and donepezil (10 mg) prior to starting this product.
iii. Indications
(a) Oral rivastigmine and galantamine approved for treatment of mild to moderate AD
(b) Donepezil and rivastigmine patch are approved for treatment of mild, moderate, and severe AD.
iv. Initiate therapy at the starting dose (Table 30), and titrate to the target dose. Guidelines suggest
initiating therapy early, as soon as the diagnosis is made, to maximize clinical benefits. Initiating
therapy at higher doses increases the risk of intolerable adverse effects, including nausea and
vomiting, which may be severe enough to cause esophageal ruptures. Interruption of therapy
for more than a few days requires retitration from the starting dose and dose increases at the
recommended intervals to avoid the possibility of significant adverse effects.

ACCP Updates in Therapeutics® 2016: Ambulatory Care Pharmacy Preparatory Review and Recertification Course

1-483
Neurology

(a) The choice of agent has little to do with clinical efficacy because the agents are essentially
equally efficacious; choice is more often related to adverse effect profile and tolerability.
Donepezil is the most commonly used agent because it is generally tolerated the best.
(b) The newer high-dose agents (donepezil 23 mg and rivastigmine patch 13 mg) should be
used cautiously because they are associated with an increased incidence of adverse effects.
For many patients, a dose increase does not lead to a significant increase in efficacy.
(c) It is important to understand, for both clinicians and patients, that these medications
do not reverse the underlying pathophysiology of the disease. Patients’ condition will
continue to progress with or without treatment. The benefits of treatment may not be
clearly apparent to patients and family members. At best, these medications may very
modestly slow cognitive decline relative to no treatment, and in some patients, little or
no cognitive/functional benefit may be achieved. As such, treatment should be reviewed
periodically for continued benefit. In addition, these medications can be associated with
significant adverse effects that must be balanced with clinical benefit. The American
Geriatrics Society Choosing Wisely Workgroup has highlighted the use of cholinesterase
inhibitors as one of 10 items that should be questioned by clinicians and patients (see
references at end of chapter).
v. Cautions/warnings
(a) Chronic obstructive pulmonary disease or asthma – Cholinergic effects can increase
bronchoconstriction and secretions.
(b) Sick sinus syndrome or bradycardia – Cholinergic effects can worsen bradycardia, which
can lead to hypotension or syncope in certain individuals; also, use caution in individuals
taking β-blockers or non-dihydropyridine calcium channel blockers.
(c) Peptic ulcer disease – Cholinergic effects increase gastric acid production; of particular
concern in patients with a history of ulcer disease or in those taking steroids or NSAIDs
vi. Monitoring
(a) Efficacy and expectations – Patients and their families should understand that the
medications used to treat AD are not cures; they will not significantly repair the damage
that is done, and they will not prevent the ultimate progression of the disease. Changes in
MMSE scores with treatment can be variable. May see slight improvement with MMSE
scores (1 or 2 points) in some patients, but many patients will not experience significant
changes in MMSE scores. With time, MMSE scores will continue to decline, despite
continued treatment.
(b) Safety – See Adverse Effects in Table 30 and Cautions/Warnings.
vii. Discontinuing therapy
(a) Lack of clear recommendations for when to discontinue therapy – Important to include family
and/or caregivers when discussing therapy discontinuance, including risks versus benefits
(b) May see clinical deterioration when discontinued, depending at what stage medications
are discontinued. Rather than discontinuing therapy abruptly, consideration should be
given to tapering treatment over a few weeks to avoid withdrawal syndrome.
(c) When patients are unable to speak, ambulate, or provide any self-care, there is little
reason to continue these medications; the risks of therapy exceed the benefits at this point.
Some suggest that the need for nursing home care defines the limit of usefulness for these
medications, thus signifying the need to discontinue. Guidelines from the American
Geriatrics Society’s A Guide to Dementia Diagnosis and Treatment recommend
treatment discontinuation when the functional assessment staging (FAST) score is 7 (see
below). A score of 7 correlates to a loss of speech and ambulation and is consistent with
end-stage dementia.

ACCP Updates in Therapeutics® 2016: Ambulatory Care Pharmacy Preparatory Review and Recertification Course

1-484
Neurology

Table 29. Functional Assessment Staging (FAST) (Check highest consecutive level of disability.)

1.  No difficulty, either subjectively or objectively


2.  Complains of forgetting location of objects; subjective work difficulties
3.  Decreased job functioning evident to coworkers; difficulty in traveling to new locations; decreased
organizational capacity*
4.  Decreased ability to perform complex tasks, for example, planning dinner for guests, handling personal
finances (such as forgetting to pay bills), difficulty marketing*
5.  Requires assistance in choosing proper clothing to wear for the day, season, or occasion, for example,
patient may wear the same clothing repeatedly, unless supervised*
6. (a) Improperly putting on clothes without assistance or cuing (e.g., may put street clothes on overnight
clothes, or put shoes on wrong feet, or have difficulty buttoning clothing) occasionally or more frequently
during the past weeks*
(b)  Unable to bathe properly (e.g., difficulty adjusting bathwater temperature) occasionally or more
frequently during the past weeks*
(c)  Inability to handle mechanics of toileting (e.g., forgets to flush the toilet, does not wipe properly or
properly dispose of toilet tissue) occasionally or more frequently during the past weeks*
(d)  Urinary incontinence (occasionally or more frequently during the past weeks)*
(e)  Fecal incontinence (occasionally or more frequently during the past weeks)*
7.  (a)  Ability to speak limited to about half a dozen intelligible different words or fewer, in the course of
an average day or in the course of an intensive interview
(b)  Speech ability limited to the use of a single intelligible word in an average day or in the course of an
intensive interview (the person may repeat the word over and over)
(c)  Ambulatory ability lost (cannot walk without personal assistance)
(d)  Cannot sit up without assistance (e.g., the individual will fall over if there are no lateral rests [arms]
on the chair)
(e)  Loss of ability to smile
(f)  Loss of ability to hold up head independently

*Scored primarily on the basis of information obtained from a knowledgeable informant and/or caregiver.
Reisberg B. Functional assessment staging (FAST). Psychopharmacol Bull 1988;24:653-9.

ACCP Updates in Therapeutics® 2016: Ambulatory Care Pharmacy Preparatory Review and Recertification Course

1-485
Table 30. Medications Approved for the Treatment of Alzheimer Disease
Drug Name Mechanism Dose/Titration Dosage Forms Adverse Effects Comments
Donepezil AChEI Start at 5 mg/day at bedtime; Tablets 5, 10, 23 Nausea, vomiting, diarrhea, Starting dose of 5 mg is clinically
(Aricept) after 4–6 weeks, may increase mg; orally disinte- dyspepsia, dizziness, effective; 23-mg tablets should not
to 10 mg/day; after 3 months grating tablets 5, headache, syncope, be split or crushed
of tolerating 10 mg/day, may 10 mg; solution 5 bradycardia, muscle
consider increase to 23 mg/ mg/5 mL; generic weakness; nausea, vomiting,
day for patients with moderate available for 5- and and weight loss are more
to severe disease for carefully 10-mg doses common with the 23-mg
selected patients; increase to dose, particularly among
23-mg dosage form associated patients who weigh less than
with limited clinical benefit 55 kg
Galantamine AChEI Start at 4 mg twice daily (8 Tablets 4, 8, 12 mg; Nausea, vomiting, diarrhea, 16 mg/day is the minimally
(Razadyne, mg/day with ER product); ER capsules 8, 16, dyspepsia, dizziness, effective dose (target dose);
Razadyne ER) after ≥4 weeks, increase to 24 mg; solution 4 headache, syncope, presynaptic nicotinic receptor
8 mg twice daily (16 mg/day mg/mL; generic bradycardia, muscle effects, but of unknown clinical
with ER); after ≥4 weeks, may available weakness relevance

1-486
increase to max dose of 12 mg
Neurology

twice daily (24 mg/day with


ER); 24 mg/day is max dose
Rivastigmine AChEI Start oral dosing at 1.5 mg Capsules 1.5, 3, Nausea, vomiting, diarrhea, Patch seems to be better tolerated
(Exelon, Exelon twice daily and titrate at 4.5, 6 mg; solu- dyspepsia, dizziness, than oral dosage forms; oral dose
Patch) 2-week intervals; max dose tion 2 mg/mL; headache, syncope, of 6 mg twice daily or patch dose
of 6 mg twice daily; for transdermal bradycardia, muscle of 9.5 mg/day is the recommended
patch, begin with 4.6 mg/24 patch 4.6-, 9.5-, weakness; adverse effects effective (target) dose; for oral
hours; after 4 weeks, may or 13.3-mg/24- increase with increasing daily dose < 6 mg, may switch
increase to 9.5-mg/24-hour hour patch; patch dose to 4.6-mg/24-hour patch; for
patch; if tolerated well and is removed and oral dose ≥ 6 mg, may switch to
after an additional 4 weeks, changed to a new 9.5-mg/24-hour patch
may consider increase to the patch every 24
13.3-mg/24-hour patch dose hours; generic
in carefully selected patients; available for cap-
increase to 13.3-mg dose sule dosage forms
is associated with limited

ACCP Updates in Therapeutics® 2016: Ambulatory Care Pharmacy Preparatory Review and Recertification Course
clinical benefit
Table 30. Medications Approved for the Treatment of Alzheimer Disease (continued)
Drug Name Mechanism Dose/Titration Dosage Forms Adverse Effects Comments
Memantine NMDA Start at 5 mg/day; at weekly Capsules 5, 10 mg; Constipation, confusion, Can be used as monotherapy, or in
(Namenda, receptor intervals, increase by 5 mg/ oral solution 2 mg/ dizziness, headache, halluci- combination with AChEI; for CrCl
Namenda XR) modulator day in divided doses every 12 mL; XR capsules 7, nations, coughing < 30 mL/minute max dose of
hours until max dose of 10 14, 21, 28 mg 5 mg bid or 14 mg XR daily
mg twice daily achieved; XR recommended; for patients whose
start at 7 mg/day, and increase condition is stabilized with 10 mg
weekly by 7 mg/day up to twice daily, conversion to XR 28
max dose of 28 mg/day mg/day can occur the day after the
last 10-mg IR dose; for patients
with CrCl < 30 mL/minute taking
5 mg twice daily, conversion to
XR 14 mg daily can occur the day
after the last dose of 5 mg
Namzaric Combination Once daily in the evening 14-mg Nausea, vomiting, diar- Patients with renal impairment
(memantine/ AchEI and (28-mg memantine XR/10-mg memantine/10-mg rhea, dyspepsia, dizziness, (CrCl 5–29 mL/minute): Dose 14
donepezil) NMDA donepezil) for patients already donepezil headache, syncope, brady- mg/10 mg

1-487
Neurology

receptor tolerating memetadine 10 cardia, muscle weakness;


There is no recommendation for
agonist mg twice daily (immediate- 28-mg nausea, vomiting, weight
patients with CrCl < 5 mL/minute
release) or 28 mg daily (XR) memantine/10-mg loss, constipation, confusion,
and also tolerating 10-mg donepezil hallucinations, and coughing
donepezil daily
AChEI = acetylcholinesterase inhibitor; bid = twice daily; CrCl = creatinine clearance; ER/XR = extended release; IR = immediate release; max = maximal; NMDA = N-methyl-d-aspartate.

ACCP Updates in Therapeutics® 2016: Ambulatory Care Pharmacy Preparatory Review and Recertification Course
Neurology

Patient Case

23. B.L. is a 72-year-old man who received a diagnosis of probable AD 1 year ago. He received initial treatment
with galantamine ER 8 mg/day shortly after his initial diagnosis, and about 8 months ago, his medication
was titrated upward to galantamine ER 24 mg. His most recent MMSE score was 23/30. He has tolerated this
drug well to this point. His insurance coverage for medications is Medicare Part D; 2 months ago, he entered
the “donut hole” and was unable to afford to pay out of pocket to continue taking galantamine ER. He is in
the clinic today, after the first of the year, and his physician is going to have B.L. restart galantamine. Which
is the most appropriate recommendation for B.L. to restart galantamine?
A. Restart at 8 mg/day for 4 weeks because he stopped taking the medication for a period longer than
several days.
B. Restart at 16 mg/day for 4 weeks because it has been less than 3 months since he stopped taking the
medication.
C. Restart at 24 mg/day because he has tolerated this dose.
D. Restart at 8 mg/day, but the medication may be titrated after 1 week.

b. NMDA receptor antagonists


i. Stimulation of NMDA receptor by glutamate leads to influx of calcium, which is associated
with neurotoxicity.
ii. Memantine – See Table 30.
(a) Noncompetitive modulation of NMDA receptor
(1) Does not impair physiologic function of NMDA receptor
(2) Reduces calcium influx
(b) Approved for treatment of moderate to severe AD
(c) See Table 30 for combination product
(d) Initiate treatment at 5 mg/day, and titrate 5 mg/day/week to target dose at weekly intervals.
iii. Indication
(a) Approved for moderate to severe disease
(b) Lack of efficacy data in mild disease
(c) Approved for monotherapy or as add-on therapy to cholinesterase inhibitor
iv. Initiating therapy – Typically added to a cholinesterase inhibitor when a patient’s condition
progresses to moderate disease or when it continues to decline, despite treatment with
a cholinesterase inhibitor; also as monotherapy for patients who are unable to tolerate
cholinesterase inhibitors or for whom cholinesterase inhibitors may be contraindicated
v. Cautions/contraindications
(a) Caution recommended in patients with seizure disorders or hepatic impairment
(b) Alkalinizing the urine (carbonic anhydrase inhibitors) decreases the clearance of memantine.
(c) Dosage adjustment recommended for CrCl less than 30 mL/minute
vi. Monitoring
(a) Efficacy – Monitoring response. Improvements may be quite modest as monotherapy or if
added to a cholinesterase inhibitor; of the same magnitude as cholinesterase inhibitors or
even less; realistic expectations should be discussed with family members and patients.
(b) Safety – See Table 30.
(c) Drug interactions: Alkalinization of urine (e.g., carbonic anhydrase inhibitors) –
Increases exposure to memantine
vii. Discontinuing therapy – Similar to cholinesterase inhibitors; consideration should also be
given to tapering memantine therapy when a decision is made to discontinue therapy, as with
acethylcholinesterase inhibitors.

ACCP Updates in Therapeutics® 2016: Ambulatory Care Pharmacy Preparatory Review and Recertification Course

1-488
Neurology

c. Vitamin E and selegiline


i. An older study showed clinical benefits of using high-dose vitamin E (2000 international
units/day) – Delaying the development of severe dementia as rated by the CDR (Clinical
Dementia Rating) Scale; the selegiline arm showed a similar benefit, but selegiline is seldom
used in this instance because it is associated with greater risks than vitamin E.
ii. More recent studies suggest high-dose vitamin E (more than 400 international units/day) is
associated with an increase in all-cause mortality.
iii. In a very recent study, the TEAM-AD VA Cooperative Randomized Trial evaluated the
effects of memantine 20 mg/day, vitamin E 2000 international units/day, the combination,
or placebo in 613 patients with mild to moderate AD. Functional decline was the primary
outcome and was assessed by using the Alzheimer’s Disease Cooperative Study/Activities of
Daily Living Inventory. The condition of patients who received vitamin E alone declined less
than those receiving placebo. There was no difference observed in those receiving memantine
alone or the combination when compared to placebo. No difference in all-cause mortality
between groups was observed. Although not described in this study, it is known that higher
doses of vitamin E may increase the risk of bleeding in patients taking anticoagulants or
antiplatelet agents.
d. Ginkgo biloba
i. Evidence supporting the use of ginkgo for prevention or treatment is mixed, but it is generally
lacking; more recent, larger, well-designed studies suggest little to no benefit. Two recent studies,
one published in 2009 from the United States and one in 2012 from France, showed that ginkgo
was not beneficial for preventing the development of AD in patients with memory concerns.
ii. The most common extract used is called EGb 761; the dose commonly used is 120–240 mg
twice daily.
iii. Antiplatelet effects: Avoid use in patients taking aspirin, clopidogrel, warfarin,
and other anticoagulants.
iv. Possible adverse effects include nausea, vomiting, diarrhea, headache, dizziness,
and palpitations.
e. Medical foods – A newer category of medical products that is regulated by the FDA as part of
the Orphan Drug Act; must be prescribed, but cannot make drug claims unless supported by
solid laboratory and clinical data; the requirements of the disease cannot be met by modifying a
normal diet. These are generally not currently recommended because of lack of supporting data
and relatively high costs. A statement from the Alzheimer’s Association Medical and Scientific
Advisory Council regarding medical foods is available at www.alz.org/documents_custom/
statements/medical_foods.pdf.
i. Axona (caprylic triglyceride) – Converted to ketone bodies in the liver and can be used as
an alternative fuel for brain cells
ii. Cerefolin NAC (methylcobalamin, l-methyl folate, N-acetylcysteine) – Intended to reduce
oxidative stress
iii. Souvenaid (uridine monophosphate, choline, omega-3 fatty acids, phospholipids, B vitamins,
antioxidants)
4. Psychiatric and behavioral problems
a. Depression
i. May be seen in up to 40% of patients with AD.
ii. Symptoms may include guilt, sadness, tearfulness, loss of appetite and weight loss, anhedonia,
and anxiety or irritability.
iii. SSRIs and SNRIs generally preferred; tricyclic antidepressants typically not used because of
anticholinergic effects

ACCP Updates in Therapeutics® 2016: Ambulatory Care Pharmacy Preparatory Review and Recertification Course

1-489
Neurology

iv. Choice of agent may be assisted by adverse effect profiles, which in some instances may be of
benefit to individual patients (e.g., mirtazapine may help with sleep issues or loss of appetite/
weight loss).
v. A 2011 meta-analysis of the treatment of depression in patients with dementia suggested
limited benefits to treatment; however, the analysis included only about 300 patients. The
studies included were inadequately powered to detect differences. Treatment should still be
considered for patients with depression because it may improve depressive symptoms and
cognition and may reduce behavioral problems in patients with AD.
b. Hallucinations and delusions
i. Consider whether hallucinations or delusions are caused by misinterpretation of environment
(patient in nursing home hears voices; however, the nursing home uses an overhead speaker/pager).
ii. Symptoms that are not disturbing or harmful to patient/patient functioning or to others do not
require pharmacologic treatment.
iii. Use of antipsychotic medications in patients with dementia has been associated with an
increase in the risk of stroke (number needed to harm -NNH about 111) and death (NNH
about 84) (black box warning from FDA), and they should be avoided if not clearly warranted.
c. Anxiety and sleep disorders
i. Consider nondrug solutions initially such as environmental factors, daytime napping, and depression.
ii. Trazodone 25–100 mg at bedtime for sleep or buspirone 30–60 mg/day for anxiety symptoms
can be considered.
iii. Avoid use of benzodiazepines for sleep or anxiety, if possible, because of effects on cognition
and increased risk of falls.
d. Agitation or aggression
i. Consider context of symptoms; opportunities for nonpharmacologic interventions
ii. If absolutely necessary for the patient’s safety or the safety of others, may consider low doses
of antipsychotics (risperidone, olanzapine, quetiapine, aripiprazole) when patient is at risk of
harming self or others; associated with increased risk of stroke and death in patients with AD.
This is off-label use; black box warning for use in patients with AD. The Clinical Antipsychotic
Trials in Intervention Effectiveness – Alzheimer Disease (CATIE-AD) study showed little
benefit of using antipsychotics for AD behaviors, considering both clinical benefits and adverse
effects. Minimizing the use of antipsychotics in long-term care is currently a big focus of
the Centers for Medicare & Medicaid Services (CMS). CMS has established the National
Partnership to Improve Dementia Care and has set goals of reducing antipsychotics in long-term
care facilities by 25% by the end of 2015 and by 30% by the end of 2016.
iii. Other possible options: Valproic acid, carbamazepine, but very limited data to show benefits
in patients with AD
5. Investigational therapies – APP, β-amyloid, and the amyloid cascade have been a major focus of AD
research recently, decreasing production or increasing clearance of β-amyloid. None of the currently
available therapies to treat AD addresses the underlying pathology surrounding β-amyloid.
a. Tramiprosate (Alzhemed) – Amino acid taurine analog; thought to prevent aggregation of
β-amyloid into plaques. Has been abandoned as a prescription drug, but is being marketed as a
“medical food”
b. Tarenflurbil (Flurizan) – Alters γ-secretase, an enzyme involved with the production of β-amyloid
from APP. Evidence so far has failed to show benefits in memory or functioning.
c. Semagacestat – γ-Secretase inhibitor; phase II studies showed lowering of cerebrospinal fluid
biomarkers, but phase III studies were terminated after showing that actively treated patients
deteriorated more than did placebo recipients.

ACCP Updates in Therapeutics® 2016: Ambulatory Care Pharmacy Preparatory Review and Recertification Course

1-490
Neurology

d. RAGE (receptor for advanced glycation end products) inhibitors – Are being studied as a way to
mitigate β-amyloid–induced brain dysfunction
e. Bapineuzumab – Monoclonal antibody that targets β-amyloid; studies actively recruiting; some
evidence suggests greater benefit in those who do not carry the APOE4 risk gene; trials with
bapineuzumab were suspended in the fall of 2012 because of disappointing trial results. Trials
with a similar drug, solanezumab, are continuing.
f. Statins – Studies have not shown benefits of statin agents for treatment or prevention of AD; not
recommended for this specific purpose
g. NSAIDs – Studies have not shown benefits of NSAIDs for treatment or prevention of AD; not
recommended for this specific purpose
h. Insulin; thiazolidinediones – Evidence suggests that a relationship exists between insulin resistance
and AD. A recent (September 2011) well-publicized pilot study of nasally administered insulin to
patients with AD showed stabilization or improvements in memory and physical functioning, relative
to placebo. This was a small pilot study that used a novel device currently not available to deliver the
insulin as studied, so using insulin in patients with AD should not be advocated at this time.
i. AN-1792 and ACC-001; CAD 106 – “Vaccines” to stimulate the immune system to develop
antibodies to β-amyloid. AN-1792 study was the initial vaccine study that was terminated because
6% of patients developed encephalopathy; ACC-001 study is actively recruiting; its mechanism is
similar to AN1792, but it is a different formulation. Studies involving CAD 106 are ongoing.

Patient Cases

24. An 87-year-old woman with severe AD is in the nursing home and is having disturbing visual hallucinations
at night that keep her awake. She is often disruptive to other residents on the unit. She has gotten out of bed on
many occasions, as though she were trying to get away from something or someone, and she has fallen twice.
She has taken lorazepam 0.5 mg at bedtime for 1 week, which has not improved her sleep or hallucinations.
Which is the most appropriate recommendation for this patient at this point in her disease course?
A. Stop lorazepam and begin zolpidem 5 mg at bedtime.
B. Stop lorazepam and begin trazodone 50 mg at bedtime.
C. Stop lorazepam and begin risperidone 0.25 mg at bedtime.
D. Stop lorazepam and begin temazepam 15 mg at bedtime.

25. B.D. is a 74-year-old woman with AD who has been taking galantamine 24 mg/day for 1 year. When she
was seen 2 months ago, her condition was stable and she was doing well with this drug. She is in the clinic
today with her daughter, who states that her mother’s memory and daily functioning have been noticeably
worse during the past 2–3 weeks. Additionally new since the past visit, B.D. has not been sleeping well at
night, so her daughter started giving her an over-the-counter sleeping agent, which has helped. Her MMSE
score today is 18/30. Two months ago, it was 21/30. Which statement most likely explains this recent change
in B.D.’s symptoms?
A. This represents the normal progression of the disease.
B. She is experiencing adverse effects of galantamine.
C. The sleep agent likely contains an antihistamine.
D. This represents the effects of sleep deprivation.

ACCP Updates in Therapeutics® 2016: Ambulatory Care Pharmacy Preparatory Review and Recertification Course

1-491
Neurology

VIII.  TRAUMATIC BRAIN INJURY (TBI)

A. Epidemiology
1. Definition – External mechanical forces leading to brain injury
2. TBI is the most common cause of death and disability in individuals between 15 and 30 years of age.
High-risk groups include children 0–4 years, teenagers and young adults 15–24 years, and adults 75
years and older.
3. According to CDC statistics, 1.7 million individuals sustain a TBI annually in the United States, with
75% experiencing mild injury.
4. Common causes of TBI include the following:
a. Military-related injuries (explosive blasts and other combat injuries)
b. Sports-related injuries
c. Motor vehicle accidents
d. Falls
e. Violence (gunshot injury, domestic violence)
f. Poststroke injury

B. Clinical and Diagnostic Issues


1. Severity – Most studies have evaluated interventions in moderate to severe TBI.
a. Mild
i. Brief loss of consciousness; dazed, confused, disoriented
ii. Memory or concentration problems
iii. Headache, dizziness, loss of balance
iv. Nausea or vomiting
v. Blurred vision, sensitivity to light or sound
vi. Fatigue, drowsiness
b. Moderate to severe
i. Loss of consciousness for several minutes to hours
ii. Profound confusion
iii. Agitation, combativeness, or unusual behaviors
iv. Slurred speech
v. Persistent headache, repeated nausea and vomiting
vi. Convulsions or seizures
vii. Pupil dilation
viii. Clear fluids from nose or ears
2. Stage
a. Acute
b. Subacute
c. Postacute
d. Chronic
3. Complications
a. Seizures, fluid accumulation, infections (penetrating wounds)
b. Vascular damage, nerve damage
c. Cognitive problems, communications problems, emotional changes
d. Sensory problems
e. Degenerative brain diseases
i. Alzheimer disease, Parkinson Disease
ii. Dementia pugilistica (repetitive blows to head [e.g., boxing])

ACCP Updates in Therapeutics® 2016: Ambulatory Care Pharmacy Preparatory Review and Recertification Course

1-492
Neurology

C. Pharmacologic Management
1. Treatment goals: Minimize the long-term complications (disability) related to the injury
2. Pharmacologic options
a. Acetylcholinesterase inhibitors
i. Limited data available for donepezil, rivastigmine, and physostigmine
ii. May have potential benefits in patients with chronic moderate and severe TBI (closed head
injury) with persistent cognitive deficits
iii. Preclinical TBI studies suggest reduced TBI-induced neuronal death, preservation of neurons,
and reduced blood-brain barrier disruption; preservation of neurologic and motor function
iv. General lack of well-designed studies and studies evaluating initiation weeks to months after
injury. Limited benefits for up to 38 weeks on neuropsychological testing outcomes
b. Amantadine
i. Anti-PD drug which effect may include blocking dopamine reuptake and influencing
dopamine synthesis; weak noncompetitive inhibitor of NMDA receptors
ii. Evidence of efficacy during the postacute period of severe TBI and disorders of
consciousness, including vegetative state and minimally conscious state; within 4–16 weeks of
TBI, patients treated with amantadine initiated at 100 mg twice daily and titrated to 200 mg
twice daily for 4 weeks improved, compared with placebo, on the Disability Rating Scale.
c. Statins
i. Potential benefits in TBI include effects on acute injury such as brain edema, blood-brain
barrier integrity, cerebral blood flow, neuroinflammation, axonal injury, and cell death.
ii. Simvastatin and atorvastatin have the most evidence supporting potential efficacy.
iii. Preclinical data provide evidence for acute or subacute administration in severe TBI; however,
lack of data testing delayed administration weeks or months after injury.
d. Methylphenidate
i. Methylphenidate promotes striatal dopaminergic neurotransmission after TBI and enhances
spatial learning, retention, and motor performance.
ii. Several trials have evaluated the effects of methylphenidate in mild to severe TBI in the
subacute or chronic phase (weeks to years post injury).
iii. End points are mixed, and heterogeneity of studies limits usefulness of data.
e. Other agents under investigation
i. Huperzine A: Chinese herb believed to have NMDA antagonist and antiseizure properties
ii. Cyclosporine A/FK 506: Animal models suggest preservation of mitochondrial function and
reduction of reactive oxygen species
iii. Erythropoietin: Potential activity may include attenuation of glutamate and nitric oxide
toxicity and antiapoptotic, antioxidant, and anti-inflammatory activity.
iv. Glyburide: Studies show that glyburide reduces inflammation, hemorrhage, and vasogenic
edema. Retrospective studies suggest that patients with diabetes treated with sulfonylureas
experience better recovery after non-lacunar stroke than do nonusers of sulfonylureas.
v. Growth hormone: Growth hormone deficiency is the most common anterior pituitary
abnormality post TBI.
vi. Lithium: May produce neuroprotective effects through reduction of excitotoxicity, ischemic
damage, and apoptosis; potential role in reducing β-amyloid accumulation
vii. vii. Progesterone: Preclinical models of TBI suggest that progesterone has neuroprotective
properties, enhancing behavioral and functional outcomes and decreasing cerebral edema,
apoptosis, and proinflammatory cytokines.

ACCP Updates in Therapeutics® 2016: Ambulatory Care Pharmacy Preparatory Review and Recertification Course

1-493
Neurology

viii. Minocycline: Shows efficacy in a variety of animal models of neurodegenerative diseases,


including TBI, cerebral ischemia, amyotrophic lateral sclerosis, PD, AD, Huntington disease,
and multiple sclerosis
ix. N-acetylcysteine
(a) Strong antioxidant and anti-inflammatory activity; may lower cytokine concentrations
(b) A small study of patient’s postconcussive blast exposure showed enhanced symptom resolution.

IX.  PARKINSON DISEASE

A. Epidemiology
1. Third most common neurologic disorder, behind AD and stroke
a. Average age at onset is 60 years.
b. More common in men, approaching a 2:1 ratio
2. Incidence
a. Age-dependent, increased with age
b. Annual incidence of 20/100,000 in adults older than 50 years
3. Prevalence
a. Between 2% and 3% of adults older than 65 years
b. Affects around 1 million people in the United States, 4 million worldwide

B. Etiology and Risk Factors


1. Idiopathic disease
a. Genetic factors (several possible genetic links and mutations)
b. Aging-related factors (oxidative stress, mitochondrial dysfunction)
c. Environmental factors (heavy metals, pesticides)
2. Drug-induced effects on dopamine
a. Antipsychotics – Phenothiazines, butyrophenones, atypical agents
b. Antiemetics – Metoclopramide, prochlorperazine
c. Toxic substances – Manganese dust, carbon monoxide poisoning, MPTP (1-methyl-4-phenyl-
1,2,5,6-tetrahydropyridine)

C. Clinical Presentation
1. Cardinal features
a. Resting tremor – Unilateral or bilateral; reduced/absent with movement and sleep
b. Rigidity – Limb muscles, cogwheeling
c. Bradykinesia – Slowed movement
2. Motor symptoms
a. Gait abnormalities, stooped posture, shuffling, festinations, lack of arm swing
b. Impaired fine movements (buttoning shirt)
c. Micrographia (small handwriting)
d. Masked face, dysarthric hypophonic speech
e. Decreased blinking, dysphagia, drooling
3. Autonomic symptoms
a. Orthostatic hypotension – Can be problematic because both the disease and the drugs used to treat
it can cause orthostatic changes

ACCP Updates in Therapeutics® 2016: Ambulatory Care Pharmacy Preparatory Review and Recertification Course

1-494
Neurology

b. Impaired GI motility, constipation


c. Bladder dysfunction, sexual dysfunction
4. Cognitive and psychiatric symptoms
a. Cognitive decline
b. Hallucinations – Can also be disease related or treatment related (dopaminergic medications)
c. Anxiety, depression, sleep disorders
d. Behavioral symptoms, agitation
5. Similar features in other neurologic diseases
a. Benign essential tremor
b. Wilson disease
c. Huntington disease
d. Lewy body dementia
e. Progressive supranuclear palsy
f. Creutzfeldt-Jacob disease

Patient Case

26. A 72-year-old female patient is in the clinic for assessment after a fall 1 week ago. She was seen in the
emergency department at that time, but no significant injuries were noted. She states that she was dizzy
before her fall. She has a history of hypertension, PD, and osteoarthritis. Her current medications include
hydrochlorothiazide 25 mg/day, metoprolol XL (extended release) 50 mg/day, lisinopril 10 mg/day, tramadol
50 mg three times daily as needed for pain, levodopa/carbidopa CR (controlled release) 200/50 mg twice
daily, and pramipexole 0.125 mg twice daily. She states that her PD symptoms are much better controlled
since adding pramipexole and decreasing levodopa/carbidopa 1 month ago. On physical examination, blood
pressure is 136/72 mm Hg, with a heart rate of 60 beats/minute sitting, and 118/60 mm Hg, with a heart rate
of 62 beats/minute standing. Her gait looks good, and her strength is good. Which is the most appropriate
recommendation to reduce her risk of future falls?
A. Discontinue pramipexole.
B. Decrease metoprolol dose.
C. Add midodrine.
D. Add fludrocortisone.

D. Diagnosis
1. Medical history and physical examination are important, as in AD.
2. Neurologic examination: Cardinal features (resting tremor, bradykinesia, rigidity)
a. Possible – One cardinal feature
b. Probable – At least two cardinal features
c. Definite – At least two cardinal features and a positive response to levodopa
3. Neuroimaging mainly useful to rule out other causes
4. Staging and assessment instruments
a. Hoen and Yahr staging – Seldom used clinically
i. Stage 0 = No clinical signs evident
ii. Stage I = Unilateral involvement
iii. Stage II = Bilateral involvement without balance impairment
iv. Stage III = Bilateral involvement; mild postural imbalance; leads independent life
v. Stage IV = Bilateral involvement; postural instability; requires help with daily activities
vi. Stage V = Severe disease; restricted to chair or bed unless assisted

ACCP Updates in Therapeutics® 2016: Ambulatory Care Pharmacy Preparatory Review and Recertification Course

1-495
Neurology

b. United Parkinson’s Disease Rating Scale (UPDRS)


i. Mainly used for evaluation of symptom severity in PD studies
ii. Assesses mood and behavior, daily function, and motor symptoms and complications
iii. Scoring: 42 components scored 0 (best) to 4 (worst) severity; higher score indicates worsening
disease and symptoms.

Patient Case

27. A 68-year-old woman with PD has been taking levodopa/carbidopa 100/25 mg four times daily for 2 weeks.
Previously, she was taking levodopa/carbidopa 100/25 mg three times daily. She is calling your clinic to see
what she can do about the symptoms she describes, which include nausea, light-headedness, and involuntary
movements, which sound like dyskinesias. Her PD symptoms were fairly well controlled on the three-times-
daily schedule, but her physician increased the dose to four times daily to achieve additional benefit. Which
is the best recommendation to address this woman’s symptoms?
A. Add rasagiline.
B. Decrease the levodopa/carbidopa dose to 100/25 mg three times daily.
C. Add ropinirole.
D. Change the levodopa/carbidopa dose to 100/10 mg four times daily.

E. Clinical Management
1. Treatment goals
a. Minimize motor and nonmotor symptoms.
b. Maximize functional status and quality of life.
c. Minimize medication-related adverse effects.
d. Maximize safety (reduce fall risk).
2. Nonpharmacologic Therapy
a. Physical therapy
b. Balance and gait training
3. Pharmacologic Therapy – See Table 31.
a. Anticholinergics (benztropine, diphenhydramine)
i. Help correct imbalance between dopamine and acetylcholine
ii. Mainly beneficial for tremors; can be used as initial therapy if tremors are predominant
iii. Limited utility because of adverse effects, particularly among older patients (confusion,
urinary retention, constipation)
b. Dopamine precursor
i. Levodopa is the most clinically effective therapy for PD symptoms.
ii. Effective for all three cardinal features.
iii. Dopa decarboxylase inhibitor (carbidopa) added to prevent peripheral conversion of levodopa
to dopamine
(a) Reduces levodopa dose requirement
(b) Improves tolerability (nausea, orthostasis, cardiac adverse effects)
(c) Need 75–100 mg of carbidopa daily to saturate dopa decarboxylase enzyme peripherally
iv. Most patients eventually develop motor complications (wearing off, dyskinesias, on-off
phenomenon, freezing) with this drug after several years of use; some clinicians advocate
delaying use of levodopa until symptoms become quite bothersome or more severe because of
the high likelihood of developing motor complications with long-term use of levodopa. This
practice may be more relevant to younger patients with longer life expectancy.

ACCP Updates in Therapeutics® 2016: Ambulatory Care Pharmacy Preparatory Review and Recertification Course

1-496
Neurology

(a) Wearing off describes loss of the clinical effect toward the end of the dosing interval; as the
disease progresses, the wearing-off effect occurs earlier, necessitating shorter dosing intervals.
(b) Dyskinesias are involuntary choreiform movements involving the neck, trunk, and upper
extremities; usually associated with peak drug effects
(c) On-off phenomenon or fluctuations describe rapid transitions from normal or controlled
motor activity to bradykinetic or uncontrolled motor activity.
(d) Freezing describes a drug-resistant off period or inability to initiate motor function;
start hesitations
c. Dopamine agonists
i. Directly stimulate dopamine receptors
ii. Can be used for initial therapy or as adjunctive therapy with levodopa
iii. Not as clinically effective as levodopa, but have fewer long-term motor complications relative
to levodopa
iv. Nonergot derivatives (pramipexole, ropinirole) preferred to ergot derivatives (bromocriptine,
pergolide) because of the risk of cardiac valvulopathy and fibrosis
v. When adding to levodopa, may need to decrease levodopa dose to avoid dopaminergic adverse
effects such as nausea, hallucinations, or dyskinesias
d. Monoamine oxidase B (MAO-B) inhibitors
i. Block oxidative degradation of dopamine through MAO-B inhibition; theorized to slow
disease progression caused by reduced oxidative stress
ii. Can be used as initial therapy or as adjunctive therapy with levodopa
iii. The Deprenyl and Tocopherol Antioxidative Therapy of Parkinsonism (DATATOP) study
showed a slowing in progression of early PD with selegiline (by a delay in the need for
levodopa therapy). The study was done to evaluate whether selegiline had neuroprotective
properties caused by reduced oxidative metabolism of dopamine. However, the reduction in
dopamine metabolism produces a symptomatic benefit in PD that cannot clearly be attributed
to increased dopamine, neuroprotective properties, or both.
iv. Control of motor symptoms with MAO-B agents considered inferior to dopaminergic
agents (levodopa, dopamine agonists)
v. As adjunctive therapy, may improve motor complications related to chronic levodopa therapy.
vi. When adding to levodopa, may need to decrease levodopa dose because of excessive
dopaminergic effects (nausea, vomiting, hallucinations, dyskinesias)
vii. Selegiline is metabolized to an amphetamine metabolite, which can cause insomnia; second
daily dose is usually given at noontime rather than later in the day.
viii. MAO-B selective inhibition should not be an issue with tyramine-containing foods when used at
recommended doses.
ix. Many of the drug interactions with MAO-B selective agents are theoretical (e.g., amphetamines,
anorexiants, antidepressants, dextromethorphan, meperidine, methadone, propoxyphene,
trazodone, St. John’s wort, sympathomimetics).
(a) When used at recommended doses, potential for drug interactions is relatively low for
traditional oral administration.
(b) Orally disintegrating selegiline tablets are contraindicated for use with
dextromethorphan, methadone, propoxyphene, tramadol, and other MAO inhibitors.
e. Catechol-O-methyl transferase (COMT) inhibitors
i. Inhibit metabolism of levodopa
ii. Not useful as monotherapy; useful only in combination with levodopa – May need to decrease
levodopa dose when added to avoid adverse dopaminergic effects

ACCP Updates in Therapeutics® 2016: Ambulatory Care Pharmacy Preparatory Review and Recertification Course

1-497
Neurology

iii. Entacapone preferred to tolcapone because tolcapone is associated with fatal hepatotoxicity;
severe diarrhea is more common with tolcapone.
iv. Brown-orange urine discoloration may occur.
f. Amantadine
i. Thought to increase the synthesis and release of dopamine; decrease dopamine reuptake;
anticholinergic effects; some evidence of NMDA modulation (similar to memantine for AD)
ii. Can be used as initial monotherapy for mild to moderate disease or as adjunctive therapy with
levodopa in advanced disease
iii. May improve tremor, rigidity, and bradykinesia; generally not considered an appropriate
option as initial monotherapy in treatment guidelines
iv. May be most useful for treating dyskinesias (motor complications) associated with long-term
dopaminergic treatment
v. Requires dose adjustment for patients with renal impairment
(a) Typically dose in the range of 200–300 mg/day with normal renal function
(b) For CrCl of 30–50 mL/minute, 100 mg/day; for CrCl of 15–29 mL/minute, 100 mg every
other day; for CrCl less than 15 mL/minute, 200 mg once weekly
g. Apomorphine
i. Nonergot dopamine agonist
ii. Used for treatment of acute, intermittent “off” episodes (freezing) associated with advanced PD
iii. Administered subcutaneously at a starting dose of 2 mg (0.2 mL)
h. Coenzyme Q10
i. Endogenous antioxidant and a lipid-soluble electron carrier
ii. Produced by fermentation of beets
iii. Supports mitochondrial function and the electron transport chain
iv. A 2002 study evaluated the effects of placebo, 300 mg/day, 600 mg/day, 900 mg/day, and
1200 mg/day in patients with PD; patients were observed for 16 months and were assessed
by using the UPDRS; the group receiving the 1200-mg/day dose showed improved UPDRS
scores relative to placebo, with the greatest effects in daily function.
v. A 2014 study evaluated the effects of placebo, 1200 mg/day and 2400 mg/day in patients with
PD; the16-month study evaluated patients by using the UPDRS but failed to demonstrate a
clinically or statistically significant improvement.

Patient Case

28. T.B. is a 63-year-old man who received a diagnosis of early PD about 6 months ago but who is otherwise
healthy. He did not receive treatment with any medications when his PD was first diagnosed, but on the
advice of his physician he started therapy with selegiline 5 mg twice daily about 4 weeks ago. He is in the
clinic today because of difficulty sleeping and difficulty with his memory. He states that, on most days, he
feels tired but just cannot fall asleep. He states that his wife has a prescription for lorazepam 0.5 mg and that
he has taken one tablet when he has had difficulty sleeping. He is asking for a prescription for lorazepam to
help him sleep. Which is the best recommendation for this patient?
A. Give him a prescription for lorazepam 0.5 mg at bedtime.
B. Have him take diphenhydramine 50 mg at bedtime.
C. Change the selegiline dosing from twice daily to morning and noon.
D. Add levodopa/carbidopa to selegiline.

ACCP Updates in Therapeutics® 2016: Ambulatory Care Pharmacy Preparatory Review and Recertification Course

1-498
Table 31. Medications for the Treatment of Symptoms in PD
Min-Max Daily
Drug Name Mechanism Dosage Forms Adverse Effects Comments
Dose
Benztropine Anticholinergic/ 0.5–4 mg Tablet, injections Confusion, dry mouth, constipation, Most useful for tremor, but not
(Cogentin) antimuscarinic blurred vision, tachycardia, urinary well tolerated, particularly in
retention patients > 65–70 years
Trihexyphenidyl Anticholinergic/ 1–6 mg Tablet, elixir Confusion, dry mouth, constipation, Most useful for tremor, but not
(Artane) antimuscarinic blurred vision, tachycardia, urinary well tolerated, particularly in
retention patients > 65–70 years
Diphenhydramine Anticholinergic/ 75–200 mg; Caplet, capsule, liquid Confusion, dry mouth, constipation, Most useful for tremor, but not
(Benadryl) antimuscarinic divided doses blurred vision, tachycardia, urinary well tolerated, particularly in
retention patients > 65–70 years
Levodopa/carbi- Dopamine 300–1000 mg of IR tablet and orally Nausea, vomiting, hallucinations, Most clinically effective
dopa (Sinemet, precursor levodopa; several disintegrating tablets: delusions, syncope, arrhythmias, treatment available for PD
Rytary, Duopa) daily doses 100/10, 100/25, 250/25 edema, hypotension motor symptoms
mg;
ER tablet: 100/25,
200/50 mg

1-499
Neurology

Rytary: 95/23.75,
145/36.25, 195/48.75,
245/61.25 mg;
Duopa: 4.63–20 mg/
mL
Pramipexole Dopamine 0.375–4.5 mg; IR tablet: 0.125, 0.25, Nausea, vomiting, hallucinations, Not as clinically effective as
(Mirapex; agonist divided doses 0.5, 1, 1.5 mg somnolence, postural hypotension, levodopa, but associated with
Mirapex ER) ER tablet: 0.375, 0.75, dizziness, confusion, edema, fewer motor complications long
1.5, 3, 4.5 mg dyskinesia term; when switching from IR
to ER, use ER dose that most
closely matches daily IR dose
Ropinirole Dopamine 0.75–24 mg; IR tablet: 0.25, 0.5, Nausea, vomiting, hallucinations, Not as clinically effective as
(Requip, Requip agonist divided doses 1, 2, 3, 4, 5 mg; XL somnolence, postural hypotension, levodopa, but associated with
XL) tablets: 2, 4, 6, 8, 12 mg dizziness, confusion, edema, fewer motor complications long
dyskinesia term; when switching from IR
to XL, use XL dose that most

ACCP Updates in Therapeutics® 2016: Ambulatory Care Pharmacy Preparatory Review and Recertification Course
closely matches daily IR dose
Table 31. Medications for the Treatment of Symptoms in PD (continued)
Min-Max Daily
Drug Name Mechanism Dosage Forms Adverse Effects Comments
Dose
Apomorphine Dopamine 2–6 mg per Injection; 10 mg/mL, Angina, drowsiness, hypotension, Administered subcutaneously
(Apokyn) agonist injection 2- or 3-mL multidose nausea, vomiting, edema, falls for intermittent freezing
cartridge episodes
Entacapone COMT inhibitor 200–1600 mg in Tablet: 200 mg; Nausea, dyskinesias, postural Not to be used as monotherapy;
(Comtan, Stalevo) divided doses; combination prod- hypotension, diarrhea, abdominal only in combination with
200 mg with uct with levodopa/ pain, brown-orange urine, levodopa treatment
each levodopa carbidopa/entacapone hyperkinesia
dose (Stalevo); 50/12.5/200,
100/25/200,
150/37.5/200 mg
Selegiline MAO-B 5–10 mg Tablet: 5 mg; orally Headache, insomnia, hallucinations, Orally disintegrating tablet
(Eldepryl) inhibitor disintegrating tablet: dizziness has better bioavailability than
1.25 mg oral tablet; risk of tyramine
reactions very low within
recommended dosage range

1-500
Neurology

Rasagiline MAO-B 0.5–1 mg Tablet: 0.5, 1 mg Postural hypotension, dyskinesias, Risk of tyramine reactions
(Azilect) inhibitor headache, nausea, weight loss very low within recommended
dosage range
COMT = catechol-O-methyl transferase; ER/XL = extended release; IR = immediate release; MAO = monoamine oxidase; max = maximal; min = minimal; PD = Parkinson disease.

ACCP Updates in Therapeutics® 2016: Ambulatory Care Pharmacy Preparatory Review and Recertification Course
Neurology

i. Initial therapy
i. Therapy generally initiated when symptoms become sufficiently bothersome to patient
functioning
ii. Dopaminergic agents (levodopa, dopamine agonists) are generally preferred as initial therapy
because of their superior control of motor symptoms.
iii. MAO-B inhibitors (selegiline, rasagiline) can be considered in patients with mild symptoms
before initiating dopaminergic treatments.
iv. Motor control: Levodopa > dopamine agonist > MAO-B
v. Risk of dyskinesias or motor complications: Levodopa > dopamine agonists
vi. Hallucinations: Dopamine agonists > levodopa
vii. No clear evidence that using sustained-release levodopa provides advantages relative to
immediate-release levodopa
j. Add-on therapy
i. If treatment is initiated with a dopamine agonist and unable to control motor symptoms,
levodopa should be considered.
ii. If treatment is initiated with levodopa, can consider adding agents if total daily levodopa dose
is 800–1000 mg; choice of agent depends on patient characteristics.
iii. If adding on therapy, need to watch closely for complications of excess dopaminergic response
(e.g., dyskinesias, nausea and vomiting, hallucinations); adding therapy to levodopa may
necessitate a decrease in the levodopa dose.
k. Motor complications (wearing off, on-off phenomenon, freezing)
i. To reduce off time, a MAO-B inhibitor, COMT inhibitor, or dopamine agonist can be considered.
ii. End-of-dose wearing off occurs because of shorter duration of individual doses of levodopa;
can consider increasing frequency of levodopa dosing; if not helpful, consider adding MAO-B
inhibitor, COMT inhibitor, or dopamine agonist.
iii. Freezing episodes: Add a dopamine agonist or MAO-B inhibitor; intermittent apomorphine;
physiotherapy and assistive walking devices and sensory cues.
l. Psychiatric symptoms
i. Depression
(a) Can occur in 40%–70% of patients with PD
(b) Limited evidence to support use of specific antidepressants in PD; medications studied
include amitriptyline, nortriptyline, citalopram, fluoxetine, sertraline, and nefazodone.
(c) Use of tricyclic antidepressants may be theoretically beneficial because of anticholinergic
effects; however, cognitive effects limit utility.
ii. Hallucinations
(a) Can be associated with PD itself, as well as with therapies to treat PD
(b) Can try to decrease dopaminergic therapies; however, will likely result in poorer motor
symptom control. Dopamine agonist is more likely to cause hallucinations relative to
dopamine; selegiline is associated with hallucinations as well.
(c) Clozapine and quetiapine may be considered to treat troublesome hallucinations in PD.
Use of antipsychotics in this situation is not without risk, and each situation requires a
careful review of risk-benefit. Hallucinations that are particularly troubling for a patient
may justify the use of an antipsychotic; metabolic complications have been associated
with the use of atypical agents, and clozapine is associated with agranulocytosis;
olanzapine worsens motor symptom control when used to treat hallucinations in PD.
m. Orthostasis
i. Orthostatic hypotension can be a complication of the disease itself, as well as the medications
to treat the disease, particularly dopaminergic treatments.

ACCP Updates in Therapeutics® 2016: Ambulatory Care Pharmacy Preparatory Review and Recertification Course

1-501
Neurology

ii. Blood pressure (sitting and standing) should be evaluated at each visit. Patients being treated for
hypertension should have antihypertensive therapies evaluated closely, particularly if used in
combination with dopaminergic treatments. Falls and fall-related injuries are common among
patients with PD.
iii. In patients with problematic orthostasis, medications that can cause orthostasis should be
evaluated closely (e.g., dopaminergic therapies, antihypertensives) to determine continued need;
if this does not adequately address the problem, other considerations may include compression
stockings, increased sodium intake, or fludrocortisone or midodrine treatment.
n. Neuroprotection
i. Selegiline, rasagiline – Inconclusive evidence that these provide neuroprotection; theory
related to decrease in oxidative metabolism of dopamine; DATATOP study suggested delay
in progression of disease with selegiline; however, symptomatic benefit of the drug clouds the
interpretation of neuroprotective effects.
ii. Dopamine agonists – Inconclusive evidence of neuroprotective effect; imaging studies (REAL-
PET, CALM-PD) suggest neuroprotective effects, but it is difficult to translate these findings to
clinical practice.
iii. Several other agents studied for neuroprotection (nicotine, caffeine, coenzyme Q10, exenatide,
deferiprone,amantadine, and NMDA receptor modulators)

F. Treatment Algorithm for PD

Figure 5. Simplified treatment algorithm for Parkinson disease.


The above schematic provides general guidance regarding treatment decisions based on patient symptoms. When patients develop marked functional
impairment, dopaminergic treatments should be considered (dopamine agonists or levodopa). The decision to use one or the other is based on the severity of
symptoms and the anticipated treatment duration. For younger patients who have prolonged life expectancy, dopamine agonists may be chosen initially and
levodopa therapy delayed until later in the disease process when symptoms may be more severe. Medications such as anticholinergics may be chosen on the
basis of symptoms and age for patients with significant tremor as a primary symptom. As the disease progresses or motor complications occur, additional
agents (dopamine agonist, COMT inhibitors) may need to be added to improve symptom control. COMT = catechol-O-methyl transferase; MAO-B =
monoamine oxidase B.

ACCP Updates in Therapeutics® 2016: Ambulatory Care Pharmacy Preparatory Review and Recertification Course

1-502
Neurology

Patient Cases

29. A 66-year-old man with a diagnosis of PD is being examined today in the clinic. He has been taking levodopa/
carbidopa for 6 years. His current levodopa/carbidopa dose is 100/25 mg, 1½ tablets in the morning, 1 tablet at
11 a.m., 1 tablet at 2 p.m., 1 tablet at 5 p.m., and ½ tablet at 8 p.m. He has been experiencing motor complications
for about 3 months, including on-off symptoms and freezing episodes. On physical examination, he has some
weakness, gait and balance abnormalities, and rigidity. His ability to ambulate and perform self-care activities
during the past 3 months has continued to decline. Which is the most appropriate recommendation for this
man’s symptoms?
A. Add benztropine to levodopa/carbidopa.
B. Decrease the levodopa/carbidopa dose to 4 tablets daily.
C. Switch to levodopa/carbidopa CR.
D. Add entacapone to levodopa/carbidopa.

30. The 66-year-old patient in the previous question returns to the clinic 2 weeks after your recommendation
above. He states that, overall, he thinks he is doing better, but that he often feels nauseated and occasionally
feels light-headed or dizzy. He also describes some abnormal movements, which are identified as dyskinesias
on physical examination. He also states that he has experienced hallucinations on two occasions, which was
rather disturbing to him. Which is the most appropriate recommendation for this man?
A. Add prochlorperazine for nausea.
B. Decrease the daily dose of levodopa/carbidopa.
C. Initiate rasagiline therapy.
D. Initiate ropinirole therapy.

X.  ESSENTIAL TREMORS (ETs)

A. Epidemiology and Pathophysiology


1. Considered the most common pathologic tremor, with a prevalence of 2%–5%
2. The pathophysiology of ET is not clear
a. Disruption of the olivocerebellar circuit
b. Hypermetabolism of the olivocerebellar system
c. Purkinje cell loss in the cerebellum and presence of brain stem Lewy bodies

B. Clinical Presentation and Diagnosis


1. Primarily a clinical diagnosis because no gold standard test or biological marker available
2. Probable ET from consensus statement of the Movement Disorder Society on tremor: Presence of an
action tremor of greater severity than enhanced physiologic tremor, without other identifiable causes

C. Treatment Options
1. AAN has published guidelines for the treatment of ET
2. Level A – Established as effective
a. Propranolol (40–160 mg/day) – The only treatment that is FDA approved for ET
i. Titrate to response.
ii. Absence of β1 selectivity is preferred when treating tremors.

ACCP Updates in Therapeutics® 2016: Ambulatory Care Pharmacy Preparatory Review and Recertification Course

1-503
Neurology

iii. Short-acting nature of propranolol allows as-needed use.


iv. Adverse effects include hypotension, bradycardia, fatigue, and erectile dysfunction; caution
when using in patients with reactive airway disease.
b. Primidone (200–250 mg/day)
i. Metabolized to phenobarbital and phenylethylmalonamide
ii. Mechanism in ET thought to be similar to that in seizure disorders
iii. Unlike with β-blockers, response does not appear to be dose related.
iv. Common adverse effects with primidone use include nausea, dizziness, and sedation; may be
reduced by starting at a lower dose and titrating to a target of 200–250 mg/day.
c. Between 30% and 50% of patients may not respond adequately to either propranolol or primidone.
3. Level B – Probably effective
a. Alprazolam
b. Atenolol
c. Gabapentin
d. Sotalol
e. Topiramate
4. Level C – Possibly effective
a. Nadolol
b. Nimodipine
c. Clonazepam
d. Botulinum toxin A
e. Deep brain stimulation, thalamotomy
5. Should not be considered: Levetiracetam and 3,4-diaminopyridine, flunarizine
6. Insufficient evidence to support or refute effectiveness: Pregabalin, zonisamide, clozapine, olanzapine

Patient Case

31. A 62-year-old patient with a diagnosis of ET is in the clinic today for an evaluation. His tremor was initially
treated with propranolol, but he was unable to tolerate it because of significant dizziness and orthostasis.
He has been treated with primidone 250 mg for almost 12 weeks, but his symptoms are not appreciably
improved by either his report or his physical examination. Which treatment would be most appropriate to
consider for this man’s symptoms?
A. Nimodipine.
B. Gabapentin.
C. Levetiracetam.
D. Zonisamide.

Special Acknowledgments
Felecia Hart, Pharm.D., Neurology Clinical Research Fellow

ACCP Updates in Therapeutics® 2016: Ambulatory Care Pharmacy Preparatory Review and Recertification Course

1-504
Neurology

REFERENCES
Epilepsy evidence-based review): vitamin K, folic acid,
1. Arain AM. Pregabalin in the management of partial blood levels, and breastfeeding: report of the
epilepsy. Neuropsychiatr Dis Treat 2009;5:407-13. Quality Standards Subcommittee and Therapeutics
2. Birbeck GL, French JA, Perucca E, et al. Evidence- and Technology Assessment Subcommittee of the
based guideline: antiepileptic drug selection for American Academy of Neurology and American
people with HIV/AIDS: report of the Quality Epilepsy Society. Neurology 2009;73:142-9.
Standards Subcommittee of the American 8. Hernandez-Diaz S, Smith CR, Shen A, et al.
Academy of Neurology and the Ad Hoc Task Force Comparative safety of antiepileptic drugs during
of the Commission on Therapeutic Strategies of the pregnancy. Neurology 2012;78:1692
International League Against Epilepsy. Neurology 9. Lowenstein DH, Alldredge BK. Status epilepticus.
2012;78:139-45. N Engl J Med 1998;338:970-6.
3. French JA, Kanner AM, Bautista J, et al. 10. Potiga [package insert]. Greenville, NC:
Efficacy and tolerability of the new antiepilep- GlaxoSmithKline, 2011.
tic drugs, I: treatment of new onset epilepsy: 11. Rogers SJ, Cavazos JE. Chapter 40: Epilepsy.
report of the Therapeutics and Technology In: DiPiro JT, Talbert RL, Yee GC,et al., eds.
Assessment Subcommittee and Quality Standards Pharmacotherapy: A Pathophysiologic Approach,
Subcommittee of the American Academy of 9th ed. New York, NY: McGraw-Hill, 2014.
Neurology and the American Epilepsy Society. Available at http://accesspharmacy.mhmedi-
Neurology 2004;62:1252-60. ca l.com /cont ent.aspx?book id= 689&Se ct io
4. French JA, Kanner AM, Bautista J, et al. nid=45310490. Accessed March 2, 2015.
Efficacy and tolerability of the new antiepilep-
tic drugs, II: treatment of refractory epilepsy: Headaches/Migraine
report of the Therapeutics and Technology 1. Comparison of available triptans. Pharmacist’s
Assessment Subcommittee and Quality Standards Letter/Prescriber’s Letter 2009;25:250509.
Subcommittee for the American Academy of 2. Derry CJ, Derry S, Moore RA. Sumatriptan (subcu-
Neurology and the American Epilepsy Society. taneous route of administration) for acute migraine
Neurology 2004;62:1261-73. attacks in adults. Cochrane Database Syst Rev
5. Harden CL, Hopp TY, Ting TY, et al. Practice 2012;2:CD009665.
parameter update: management issues for women 3. Dowson AJ, Lipscombe S, Sender J, et al. New
with epilepsy: focus on pregnancy (an evidence- guidelines for the management of migraine in pri-
based review): obstetrical complications and mary care. Curr Med Res Opin 2002;18:414-39.
change in seizure frequency: report of the Quality 4. Lipton RB, Bigal ME, Rush SR, et al. Migraine
Standards Subcommittee and Therapeutics and practice patterns among neurologists. Neurology
Technology Assessment Subcommittee of the 2004;62:1926-31.
American Academy of Neurology and American 5. Minor DS, Wofford MR. Chapter 45: Headache
Epilepsy Society. Neurology 2009;73:126-32. disorders. In: DiPiro JT, Talbert RL, Yee GC, et
6. Harden CL, Meador KJ, Pennell PB, et al. Practice al., eds. Pharmacotherapy: A Pathophysiologic
parameter update: management issues for women Approach, 9th ed. New York, NY: McGraw-
with epilepsy focus on pregnancy (an Hill, 2014. Available at http://accesspharmacy.
evidence-based review): teratogenesis and peri- mhmedical.com/content.aspx?bookid=689&Sectio
natal outcomes: report of the Quality Standards nid=45310495. Accessed March 2, 2015.
Subcommittee and Therapeutics and Technology 6. Morey SS. Practice guidelines: guidelines on
Assessment Subcommittee of the American migraine, part 3: recommendations for individ-
Academy of Neurology and American Epilepsy ual drugs. Am Fam Physician 2000;15. Available
Society. Neurology 2009;73:133-41. at www.aafp.org/ afp/20001101/practice.html.
7. Harden CL, Meador KJ, Pennell PB, et al. Accessed February 10, 2012.
Practice parameter update: management issues
for women with epilepsy focus on pregnancy (an

ACCP Updates in Therapeutics® 2016: Ambulatory Care Pharmacy Preparatory Review and Recertification Course

1-505
Neurology

7. Papetti L, Spalice A, Nicita F, et al. Migraine treat- 6. Federation of State Medical Boards of the United
ment in developmental age: guidelines update. J States. Model Policy on the Use of Opioid
Headache Pain 2010;11:267-76. Analgesics in the Treatment of Chronic Pain. July
8. Ramadan NM, Silberstein SD, Freitag FG, et al. 2013. Available at www.fsmb.org/pdf/pain_policy_
Evidence-Based Guidelines for Migraine Headache july2013.pdf. Accessed December 15, 2013.
in the Primary Care Setting: Pharmacological 7. Fitzcharles MA, Ste-Marie PA, Goldenberg DL, et
Management for Prevention of Migraine. St. Paul, al. 2012 Canadian Guidelines for the Diagnosis and
MN: American Academy of Neurology, 2000. Management of Fibromyalgia Syndrome. Available
Available at www.aan.com/professionals/practice/ at www.canadianpainsociety.ca/pdf/Fibromyalgia_
pdfs/gl0090.pdf. Accessed February 10, 2012. Guidelines_2012.pdf. Accessed October 30, 2013.
9. Silberstein SD. Practice parameter: evidence-based 8. Fudin J. Urine Drug Screen Algorithm. Available
guidelines for migraine headache (an evidence- at http://paindr.com/wp-content/uploads/2012/05/
based review): report of the Quality Standards Urine-Drug-Screen-Algorithm.pdf. Accessed
Subcommittee of the American Academy of October 30, 2012.
Neurology. Neurology 2000;55:754-62. 9. Hachberg MC, Altman R, April KT, et al. American
10. Silbertein SD, Holland S, Freitag F, et al. Evidence- College of Rheumatology 2012 recommendations
based guideline update: pharmacologic treatment for the use of nonpharmacologic and pharmaco-
for episodic migraine prevention in adults: report logic therapies in osteoarthritis of the hand, hip,
of the Quality Standards Subcommittee of the and knee. Arthritis Care Res 2012;64:465-74.
American Academy of Neurology and the American 10. Heit H, Gourlay D. Urine drug testing in pain med-
Headache Society. Neurology 2012;78:1337. icine. J Pain Symptom Manage 2004;27:260-7.
11. Krantz MJ, Martin J, Stimmel B, et al. QTc interval
Pain Management screening in methadone treatment. Ann Intern Med
1. Carville SF, Arendt-Nielsen, Bliddal H, et al. 2009;150:387-95.
EULAR evidence-based recommendations for 12. McPherson ML. Demystifying Opioid Conversion
the management of fibromyalgia syndrome. Ann Calculations: A Guide for Effective Dosing.
Rheum Dis 2008;67:536-41. Bethesda, MD: American Society of Health-
2. Chou R, Fanciullo GJ, Fine PG, et al. Clinical System Pharmacists, 2010:1-196.
guidelines for the use of chronic opioid therapy in 13. Moeller KE, Lee KC, Kissack JC. Urine drug
chronic noncancer pain. J Pain 2009;10:113-30. screening: practical guide for clinicians. Mayo Clin
3. Chou R, Huffman LH. Medications for acute and Proc 2008;83:66-76.
chronic low back pain: a review of the evidence for 14. Nalamachu SR. Opioid rotation in clinical practice.
an American Pain Society/American College of Adv Ther 2012;29:849-63.
Physicians clinical practice guideline. Ann Intern 15. Vargas-Schaeffer G. Is the WHO analgesic ladder
Med 2007;147:505-14. still valid? Can Fam Physician 2010;56:514-7.
4. Chou R, Qaseem A, Snow V, et al. Diagnosis and 16. Washington State Agency Medical Directors’
treatment of low back pain: a joint clinical practice Group (AMDG). Interagency Guideline on Opioid
guideline from the American College of Physicians Dosing for Chronic Non-cancer Pain: 2010 Update.
and the American Pain Society. Ann Intern Med Available at www.agencymeddirectors.wa.gov/
2007;147:478-91. Files/OpioidGdline.pdf. Accessed December 15,
5. Department of Veterans Affairs. VA/DoD Clinical 2013.
Practice Guideline for Management of Opioid 17. Woolf CJ. Pain: moving from symptom control
Therapy for Chronic Pain. May 2010. Available toward mechanism-specific pharmacologic man-
at www.healthquality.va.gov/cot/cot_310_sum.pdf. agement. Ann Intern Med 2004;140:441-51.
Accessed December 15, 2013.

ACCP Updates in Therapeutics® 2016: Ambulatory Care Pharmacy Preparatory Review and Recertification Course

1-506
Neurology

Neuromuscular Diseases 2. Taricco M, Pagliacci MC, Telaro E, et al.


1. Bainbridge JL, Miravalle A, Corboy JR. Chapter 39: Pharmacological interventions for spasticity fol-
Multiple sclerosis. In: DiPiro JT, Talbert RL, Yee GC, lowing spinal cord injury: results of a Cochrane
et al., eds. Pharmacotherapy: A Pathophysiologic systematic review. Eura Medicophys 2006;42:5-15.
Approach, 9th ed. New York, NY: McGraw-
Hill, 2014. Available at http://accesspharmacy. Alzheimer Disease
mhmedical.com/content.aspx?bookid=689&Sectio 1. AGS Choosing Wisely Workgroup. American
nid=45310489. Accessed March 2, 2015. Geriatrics Society identifies five things that health-
2. Chataway J, Miller DH. Natalizumab ther- care providers and patients should question. J Am
apy for multiple sclerosis. Neurotherapeutics Geriatr Soc 2013;61:622-31.
2013;10:19-28. 2. AGS Choosing Wisely Workgroup. American
3. Compston A, Coles A. Multiple sclerosis. Lancet Geriatrics Society identifies another five things that
2008;372:1502-17. healthcare providers and patients should question. J
4. Confavreux C, O’Connor P, Comi G, et al. Oral Am Geriatr Soc 2014;62:950-960.
teriflunomide for patients with relapsing multiple 3. American Geriatrics Society. A Guide to Dementia
sclerosis (TOWER): a randomized, double-blind, Diagnosis and Treatment. New York: American
placebo-controlled, phase 3 trial. Lancet Neurol Geriatrics Society, 2010.
2014;13:247-56. 4. Gold Standard, Inc. Donepezil; memantine.
5. Damal K, Stoker E, Foley JF. Optimizing therapeu- Clinical Pharmacology [Internet database].
tics in the management of patients with multiple Available at www.clinicalpharmacology.com.
sclerosis: a review of drug efficacy, dosing and Accessed September 30, 2015.
mechanisms of action. Biologics 2013;7:247-58. 5. Hansen RA, Gartlehner G, Webb AP, et al. Efficacy
6. Fox RJ, Miller DH, Phillips JT, et al. Placebo- and safety of donepezil, galantamine, and rivastig-
controlled phase 3 study of oral BG-12 or mine for the treatment of Alzheimer’s disease: a
glatiramer in multiple sclerosis. N Engl J Med systematic review and meta-analysis. Clin Interv
2012;367:1087-97. Aging 2008;3:211-25.
7. Gold R, Kappos L, Arnold D, et al. Placebo- 6. Lyketsos CG, Colenda CC, Beck C, et al. Position
controlled phase 3 study of oral BG-12 for relapsing statement of the American Association for Geriatric
multiple sclerosis. N Engl J Med 2012;367:1098-107. Psychiatry regarding principles of care for patients
8. Jayam TA, Dabi A, Solieman N, et al. with dementia resulting from Alzheimer disease.
Myasthenia gravis: a review. Autoimmune Dis Am J Geriatr Psychiatry 2006;14:561-72.
2012;2012:874680. 7. Mayeaux R. Early Alzheimer’s disease. N Engl J
9. O’Connor P, Wolinsky JS, Confavreux C, et Med 2010;362:2194-201.
al. Randomized trial of oral teriflunomide for 8. Querfurth HW, LaFerla FM. Mechanisms of
relapsing multiple sclerosis. N Engl J Med disease: Alzheimer’s disease. N Engl J Med
2011;365:1293-303. 2010;362:329-44.
10. Schapiro RT. Managing symptoms of multiple scle- 9. Reisberg B. Functional assessment staging (FAST).
rosis. Neurol Clin 2005;23:177-87. Psychopharmacol Bull 1988;24:653-9.
11. Scott LJ. Fingolimod: a review of its use in the 10. Schneider LS, Tariot PN, Dagerman KS, et al.
management of relapsing-remitting multiple scle- Effectiveness of atypical antipsychotic drugs in
rosis. CNS Drugs 2011;25:673-98. patients with Alzheimer’s disease. N Engl J Med
2006;335:1525-38.
Spinal Cord Injuries 11. Tariot PN, Farlow MR, Grossberg GT, et al.
1. Ganguly K, Abrams GM. Management of chronic Memantine treatment in patients with moderate
myelopathy symptoms and activities of daily liv- to severe Alzheimer disease already receiving
ing. Semin Neurol 2012;32:161-8. donepezil: a randomized controlled trial. JAMA
2004;291:317-24.

ACCP Updates in Therapeutics® 2016: Ambulatory Care Pharmacy Preparatory Review and Recertification Course

1-507
Neurology

Traumatic Brain Injury 6. Suchowersky O, Gronseth G, Perlmutter J, et al;


1. Diaz-Arrastia R, Kochanek PM, Bergold P, et Quality Standards Subcommittee of the American
al. Pharmacotherapy of traumatic brain injury: Academy of Neurology. Practice parameter: neu-
state of the science and the road forward report roprotective strategies and alternative therapies
of the Department of Defense Neurotrauma for Parkinson disease (an evidence-based review):
Pharmacology Workgroup. J Neurotrauma report of the Quality Standards Subcommittee of
2014;31:135-58. the American Academy of Neurology. Neurology
2. Giacino JT, Whyte J, Bagiella E, et al. Placebo- 2006;66:976-82.
controlled trial of amantadine for severe traumatic 7. Zesiewicz TA, Sullivan KL, Arnulf I, et al.;
brain injury. N Engl J Med 2012;366:819-26. Quality Standards Subcommittee of the American
3. Hoffer ME, Balaban C, Slade MD, et al. Academy of Neurology. Practice parameter: treat-
Amelioration of acute sequelae of blast induced ment of nonmotor symptoms of Parkinson disease:
mild traumatic brain injury by N-acetyl cysteine: a report of the Quality Standards Subcommittee of
double-blind, placebo controlled study. PLoS ONE the American Academy of Neurology. Neurology
2013;8:e54163. 2010;74:924-31.

Parkinson Disease Essential Tremor


1. Gold Standard, Inc. carbidopa; levodopa. Clinical 1. Sadeghi R, Ondo WG. Pharmacological manage-
Pharmacology [Internet database]. Available ment of essential tremor. Drugs 2010;70:2215-28.
at www.clinicalpharmacology.com. Accessed 2. Zesiewicz TA, Elble RJ, Louis ED, et al.
October 1, 2015. Evidence-based guideline update: treatment of
2. Miyasaki JM, Martin W, Suchowersky O, et al. essential tremor: report of the Quality Standards
Practice parameter: initiation of treatment for Subcommittee of the American Academy of
Parkinson’s disease: an evidence based review: Neurology. Neurology 2011;77:1752-5.
report of the Quality Standards Subcommittee of
the American Academy of Neurology. Neurology
2002;58:11-7.
3. Miyasaki JM, Shannon K, Voon V, et al.; Quality
Standards Subcommittee of the American Academy
of Neurology. Practice parameter: evaluation and
treatment of depression, psychosis, and dementia
in Parkinson disease (an evidence-based review):
report of the Quality Standards Subcommittee of
the American Academy of Neurology. Neurology
2006;66:996-1002.
4. Nutt JG, Wooten GF. Diagnosis and initial man-
agement of Parkinson’s disease. N Engl J Med
2005;353:1021-7.
5. Pahwa R, Factor SA, Lyons KE, et al.; Quality
Standards Subcommittee of the American
Academy of Neurology. Practice parameter: treat-
ment of Parkinson disease with motor fluctuations
and dyskinesia (an evidence-based review): report
of the Quality Standards Subcommittee of the
American Academy of Neurology. Neurology
2006;66:983-95.

ACCP Updates in Therapeutics® 2016: Ambulatory Care Pharmacy Preparatory Review and Recertification Course

1-508
Neurology

ANSWERS AND EXPLANATIONS TO PATIENT CASES


1. Answer: B Options A, B, and C all contain drug combinations with
This patient, who is older than 60 years, has just been AEDs listed that are metabolized by the liver, making
given a diagnosis of new-onset epilepsy. The patient them not the best options for this patient because of
also has diabetic peripheral neuropathy. Gabapentin is increased laboratory monitoring.
a good drug to start with because it has an FDA-labeled
indication for both complex partial seizures and diabetic 4. Answer: B
peripheral neuropathy. Veterans Administration (VA) This patient is seen in the ED for status epilepticus. The
Cooperative Study 428 supports the use of gabapentin ED physician initiates phenytoin intravenously. The
in patients older than 60 years with new-onset seizures maximal infusion rate for phenytoin is 50 mg/minute, and
with respect to efficacy and tolerability. The medications it is important to communicate to minimize complications
in Options A, C, and D all have indications for partial- such as hypotension, cardiac arrhythmias, and venous
onset seizures; however, they may not be as effective for cording. Phenytoin can be given intramuscularly (Option
diabetic peripheral neuropathy. Felbamate is not consid- A) but only when intravenous access cannot be achieved
ered a first-line choice for partial-onset epilepsy because because of the higher occurrence of tissue necrosis
of its adverse event profile. with this route of administration. Option C: Phenytoin
is safest when diluted in normal saline as opposed
2. Answer: B to dextrose, which would increase the likelihood of
This patient now has severe renal dysfunction, so microcrystal formation and tissue necrosis. Option D:
it would be appropriate to choose an AED that is Tissue necrosis can occur with intravenous extravasation
metabolized by the liver and incompletely eliminated and intramuscular injection.
by the kidneys. Lamotrigine is effective for partial-onset
seizures and has proved to be a well-tolerated drug by 5. Answer: A
the VA Cooperative Study 428. Topiramate (Option A) This patient is experiencing a cluster headache, as based
is metabolized through the liver. However, because of on several factors. One factor is his sex because cluster
the patient’s severe renal dysfunction and topiramate’s headaches are more common in male individuals. There is a
carbonic anhydrase inhibition, the risk of kidney stones genetic component with cluster headaches, and the patient’s
outweighs the potential benefit of the AED. Option C father experienced the same type of headaches. Another
and Option D represent AEDs that are either minimally factor is that cluster headaches are of shorter duration
metabolized or altogether eliminated by the kidneys. and present with more excruciating pain than migraine
One of these AEDs could be chosen; however, extensive headaches. The clinical factors that are consistent in this
monitoring of renal function and clinical toxicity would case are the patient’s symptoms of “ice pick” pain sensation
be necessary, making them less desirable options. through his eye, unilateral location, and no nausea or aura.
Vigabatrin is FDA indicated for infantile spasms and This patient is also describing his headaches as having a
refractory complex partial seizures, making it not the cluster pattern for the past 2 years in the spring and fall.
best AED for this patient currently. He is also having trouble sitting still. Option B, tension
headache, is incorrect because tension-type headaches are
3. Answer: D less severe and bilateral, generally with a longer duration and
This patient has just been given a diagnosis of severe liver a bandlike sensation. Option C and Option D are incorrect.
disease. It is important to try to discontinue the drugs Migraine headaches with and without aura usually last
that are metabolized by the liver, both phenobarbital and between 4 and 72 hours; have a pulsating characteristic;
valproate. It would be appropriate to initiate levetiracetam and are associated with nausea, vomiting, photophobia, and
and pregabalin, one at a time, because they are not phonophobia. Patients with migraine headaches generally
dependent on the liver for metabolism. Levetiracetam want to be still, not be bothered, and reside in a quiet, dark
and pregabalin are both FDA indicated for partial-onset place. Exertion aggravates the pain of migraine headache.
seizures with secondary generalization. When initiating Migraine with aura would have all the same symptoms as
drug therapy for this patient, pregabalin should not a regular migraine but with a neurologic aura sensation 60
be added if levetiracetam is controlling his seizures. minutes before the pain phase.

ACCP Updates in Therapeutics® 2016: Ambulatory Care Pharmacy Preparatory Review and Recertification Course

1-509
Neurology

6. Answer: D 11. Answer: A


The best acute therapy for this patient’s cluster headache Unlike patients with acute pain, many patients with
is oxygen therapy. Choices A, B, and C are options; chronic pain will not experience elevations in their BP
however, because they are orally administered, their and HR while experiencing moderate to severe pain.
onset of action is delayed in a patient with severe pain. These patients experience pain on a daily basis and have
become accustomed to it. Blood pressure and HR are
7. Answer: C not accurate predictors of pain severity in the population
Sumatriptan subcutaneous injection has the fastest onset with chronic pain. Although it is difficult for the health
of action of all the triptans. This is important in treating care provider to take patients’ descriptions of severe pain
a cluster headache because of its intense pain and when they can sit calmly and describe horrible pain, the
generally short duration. The medications in Options A, provider should consider this as part of the normal life
B, and D all have a longer time to onset than sumatriptan of these patients. However, pain scales and descriptors
subcutaneous injection. appropriate to their disease should be used to obtain the
best measure of their clinical status.
8. Answer: B
Topiramate (Answer B) is FDA approved as a preventive 12. Answer: B
therapy for migraine and is a good choice for this patient. The patient has FM according to the ACR 2010 criteria
However, it is a pregnancy category D drug, and she is for FM diagnostic criteria. She has widespread pain
of childbearing age; thus, counsel and recommend to that has lasted much longer than 3 months. She also
the patient the use of a barrier method of birth control. would likely have a high SS score—fatigue, cognitive
Excedrin Migraine (Option A) is not preferred because symptoms, constipation, and somatic concerns.
the patient has used this drug without success, and Fibromyalgia is a chronic pain disorder that affects
continued use could potentially cause medication- many different systems. She should be educated about
overuse headache. Sumatriptan (Option C) is not the her disease and its therapy.
best choice. Sumatriptan is an abortive agent that could
potentially cause medication-overuse headache when 13. Answer: C
used on a daily basis. Propranolol (Option D) is FDA The patient should begin a trial with duloxetine. One of
approved for migraine prophylaxis, yet this patient is an the first agents used for patients with FM is often an SNRI
athlete and has a low HR, making this option incorrect to address both the pain and the depression. In this case,
for her treatment currently. the patient’s sertraline would need to be discontinued
before starting duloxetine therapy. Although weak
9. Answer: D opioids and tramadol can be used in patients with FM,
Rizatriptan, Answer D, is correct because it is available they should be reserved until patients’ other therapies
in an orally disintegrating tablet formulation. The have failed. This patient is already classified as obese;
medications in Options A, B, and C are available only therefore, pregabalin is not a good choice for her.
as oral formulations and may not be the best options
because they may cause nausea. 14. Answer: B
This patient should be seen in an ED immediately.
10. Answer: B Numbness in an extremity can be a sign of cord
Frovatriptan is the correct answer. Given that it has the compression and is a red flag symptom of back and neck
longest half-life of all the triptans, frovatriptan would pain. If a patient reports this as a new symptom to you,
be ideal for this patient’s long flight. The half-life of he or she should be referred immediately.
frovatriptan is 26 hours. The half-life of rizatriptan is
2–3 hours. The half-life of sumatriptan is 2.5 hours. The
half-life of almotriptan is 3–4 hours.

ACCP Updates in Therapeutics® 2016: Ambulatory Care Pharmacy Preparatory Review and Recertification Course

1-510
Neurology

15. Answer: B 19. Answer: A


The test of choice in this instance would be GC-MS This patient is given a diagnosis of CIS. She has two
because of the medications being prescribed. Although lesions on MRI and is monosymptomatic (optic neuritis).
the GC-MS is a more costly test than an immunoassay, It is appropriate to initiate a first-generation DMT such
immunoassays do not as reliably pick up the synthetic as glatiramer acetate for her new diagnosis because it
opioids. Serum testing is more difficult/invasive and is effective and has less burdensome adverse effects.
is usually performed when exact concentrations of Neither Option B nor Option D is the best choice for this
medications or their metabolites are needed. Serum patient right now because each option has more severe
testing may also be needed if a CYP anomaly is adverse effects and requires increased monitoring.
suspected. Saliva testing is not used in routine practice. Option C would not be the best choice right now because
this agent is newer and generally reserved by insurance
16. Answer: B for patients whose first-generation DMT has failed or for
The DIRE tool is best to use when the health care provider patients with more severe disease at presentation.
wants to conduct the risk assessment as an interview.
The SOAPP is to be used as a patient-completed form. 20. Answer: D
The COMM is for evaluating patients who are receiving Natalizumab is the best choice currently because
chronic opioid therapy to monitor potentially aberrant use this patient’s first-generation DMT has failed, and
of their therapy. Each of these tools is used to evaluate natalizumab has increased efficacy over the first-
a patient’s risk of chronic opioid therapy. In each case, generation DMTs. Natalizumab has the potential to
patients are given a risk level of high, medium, or low, cause life-threatening adverse effects, so the patient will
which can be used to adjust their care plan. The CAGE need to be enrolled in the TOUCH REMS program.
tool is a more generic screening tool that evaluates patients Mitoxantrone (Option A) has an adverse effect profile
for inappropriate use of substances. (secondary leukemias 1:145) that makes it not the best
choice currently. Interferon β-1a (Option B) is a potent
17. Answer: D first-generation DMT; however, this patient’s disease
The patient is taking a total daily dose of 180 mg of course is progressing faster than when she originally
oxycodone. The equianalgesic ratio is 20 mg of oxycodone began therapy. Some insurance companies require that a
equal to 30 mg of morphine. The conversion is 270 mg patient’s two first-generation DMTs fail before they can
total daily dose of morphine. A slight reduction in this switch to a second-generation DMT. Option C is not the
dose should be incorporated to account for incomplete best choice currently because the efficacy of this agent
tolerance between opioids. A 25% reduction to this dose is similar to that of the first-generation DMTs, with an
would be a 67-mg reduction. A convenient dose would be increased adverse effect profile.
100 mg twice daily of morphine extended release.
21. Answer: D
18. Answer: B The MMSE is not a diagnostic test for AD or dementia; it
The adverse effects of mycophenolate mofetil reported in is merely a screening tool to identify cognitive symptoms.
studies in patients with MG includes myelosuppression Therefore, this patient has cognitive impairment, as based
(with increased risk of infection), nausea, vomiting, and on the MMSE score. Whether the cognitive impairment
diarrhea. It is not associated with thrombocytosis, renal is caused by AD, stroke, depression, or other factors is
impairment, or alopecia. unclear in this case. Until the results of other tests yet to
be performed (laboratory testing, CT) are available, the
underlying etiology of the cognitive impairment in this
patient cannot be determined with certainty.

ACCP Updates in Therapeutics® 2016: Ambulatory Care Pharmacy Preparatory Review and Recertification Course

1-511
Neurology

22. Answer: B 25. Answer: C


Pharmacologic effects of the cholinesterase inhibitors, Most over-the-counter sleep medications contain
as a class, can include bradycardia, similar to stimulation antihistamines such as diphenhydramine. The
of the vagus nerve. A recent retrospective study found anticholinergic effects of antihistamines can interact
that patients taking cholinesterase inhibitors were more with cholinesterase inhibitors, reducing the clinical
likely to experience emergency department visits and effectiveness of these medications. The sudden decline in
hospitalizations for syncope, bradycardia, and falls as cognition and function in this case is most likely because
compared with nonusers of cholinesterase inhibitors. of the use of an antihistamine. The normal decline in AD
Patients at increased risk of bradycardia, including is slow and progressive and is not typically associated
patients taking β-blockers and non-dihydropyridine with sudden or dramatic changes. Some patients may
calcium channel blockers, should be monitored closely have adverse response to cholinesterase inhibitors
if cholinesterase inhibitors are prescribed. Memantine, behaviorally or cognitively, but this likely occurs early
which has a different mechanism of action, has not been in treatment, not after several months of use. Sleep
associated with bradycardia, and it would be the safest deprivation could be associated with worsening of AD
option for this patient. symptoms, but this does not appear to be the problem in
this case. Often, daytime napping can be associated with
23. Answer: A nighttime sleeping problems in patients with AD.
The recommendations for titrating cholinesterase
inhibitors are based largely on the dose-related risk of 26. Answer: B
GI adverse effects. If treatment with a cholinesterase Hypotension and orthostatic hypotension can be an
inhibitor is interrupted for several days or longer, autonomic complication of PD. Medications used to treat
package insert recommendations are to reinitiate therapy PD, particularly dopaminergic therapies, can also be
at the lowest dose and to re-titrate at the same rate as associated with hypotension. Pramipexole was recently
if the patient were starting the drug for the first time. added, and the levodopa dose was reduced, which led
For galantamine ER, the initial dose is 8 mg/day for to an improvement in PD symptoms. Complicating the
at least 4 weeks; titrate to 16 mg/day for 4 weeks; and issue, in this case, is the fact that the patient is taking
then titrate to 24 mg/day. Published case reports show three antihypertensive agents. Discontinuing pramipexole
patients experiencing intractable vomiting related to too- would lead to a worsening of PD symptom control, which
rapid titration of cholinesterase inhibitors or reinitiating might increase the risk of falls. Adding another drug
therapy at maximal tolerated doses, including cases of to increase BP (midodrine, fludrocortisone) in a patient
esophageal tears. taking three antihypertensives would not make sense.
Reducing the dose or discontinuing at least one of the
24. Answer: C antihypertensives would be the most rational approach to
Treatment of behavioral symptoms related to AD should reducing her risk of orthostasis and future falls.
be based on the specific behaviors observed. Before
medications are recommended, the context of the 27. Answer: B
behavior should be explored to identify behaviors that may The symptoms described by this woman are consistent
respond to nonpharmacologic interventions. Patients who with excessive dopaminergic treatment, particularly in
experience hallucinations or delusions that are disturbing the context of dyskinesias. Adding rasagiline or ropinirole
or potentially harmful to the patient or others may require to her current therapy would make the problem worse.
low doses of antipsychotics such as risperidone, quetiapine, Changing the levodopa/carbidopa dose to 100/10 mg
olanzapine, or aripiprazole. The use of antipsychotics four times daily would do little, if anything, to improve
in patients with AD is considered “off-label” and has a dyskinesias and would likely make nausea symptoms
black box warning because of an increased risk of stroke worse because 75–100 mg of carbidopa per day is needed
and death in clinical trials, relative to placebo-treated to saturate peripheral dopa decarboxylase. Reducing
individuals. Although zolpidem, trazodone, or temazepam her levodopa/carbidopa dose to her previous dose of
may be considered to help with sleep, none of these would three times daily would provide the optimal balance of
be beneficial for hallucinations. symptom control while minimizing adverse effects.

ACCP Updates in Therapeutics® 2016: Ambulatory Care Pharmacy Preparatory Review and Recertification Course

1-512
Neurology

28. Answer: C 31. Answer: B


Selegiline is metabolized to an amphetamine metabolite, The recommended initial treatments for ET include
and administering the second daily dose late in the day propranolol and primidone. However, between 30% and
has been associated with insomnia. As such, the second 50% of patients will not achieve response to these initial
dose is recommended to be given midday. This would therapies. Guidelines from the American Academy
be the optimal initial approach, rather than adding of Neurology provide recommendations for other
another agent to treat sleep. Anticholinergics such as treatment options for patients whose initial treatment
diphenhydramine might provide benefit for tremor options fail. Gabapentin is recommended as a “probably
symptoms, but the possible anticholinergic adverse effective” treatment. Nimodipine is recommended as a
effects of this treatment make the risk-benefit profile “possibly effective” treatment, and levetiracetam is not
of this option unfavorable. Adding levodopa/carbidopa recommended for treating ET. Evidence is insufficient to
currently is not indicated unless functional symptoms support or refute the use of zonisamide for ET.
are poorly controlled with selegiline.

29. Answer: D
The occurrence of motor complications (wearing off,
on-off, freezing) is common with chronic levodopa therapy.
Adjusting the dose or timing of levodopa administration
might help symptoms in some instances (wearing off), but
it would not likely improve symptoms in others (on-off,
freezing). Practice guidelines of the American Academy of
Neurology recommend adding a MAO-B inhibitor, COMT
inhibitor, or dopamine agonist for treatment of patients
with motor complications, such as for this patient. Using an
anticholinergic agent or switching from immediate-release
levodopa preparations to sustained-release preparations is
not recommended in this instance. Entacapone is available
as a separate agent that can be added to levodopa/carbidopa.
The branded product containing levodopa/carbidopa/
entacapone should not be crushed or broken; therefore, using
separate products should be recommended in this instance.

30. Answer: B
Abnormal involuntary movements (e.g., dyskinesias)
suggest excessive dopaminergic treatment. When a second
agent is added to levodopa therapy, whether it is a MAO-B
inhibitor, COMT inhibitor, or dopamine agonist, most
patients require a decrease in their levodopa dose to avoid
possible dopamine-related adverse effects (dyskinesias,
nausea or vomiting, hallucinations). Decreasing the
levodopa dose would be the best initial approach to
managing this problem. Adding rasagiline would not help
these symptoms and would most likely make them worse.
Adding prochlorperazine would be counterproductive
because it possesses dopamine-blocking effects, and
adding a drug to treat the adverse effects of another drug
should be avoided if possible. Anticholinergic agents would
not be effective in this case, and they are poorly tolerated.

ACCP Updates in Therapeutics® 2016: Ambulatory Care Pharmacy Preparatory Review and Recertification Course

1-513
Neurology

ANSWERS AND EXPLANATIONS TO SELF-ASSESSMENT QUESTIONS


1. Answer: B 4. Answer: A
The best answer is lacosamide. Cyclosporine is a Benzodiazepines are considered first-line therapy
substrate of the CYP3A4 isoenzyme. Carbamazepine because they can rapidly control seizures. The three
is not the best option because it is a potent inducer of most commonly used benzodiazepines are diazepam,
the CYP3A4 isoenzyme. Oxcarbazepine is a moderate lorazepam, and midazolam. Lorazepam enters the brain
enzyme inducer/inhibitor at high doses of the drug. rapidly and does not partition out quickly, as opposed to
Oxcarbazepine is metabolized by CYP3A4, so again, the diazepam, which is more lipophilic but partitions out of
serum concentration would be lowered for cyclosporine. the brain rapidly. Phenobarbital (Option B) is an option
Phenytoin is also not preferred for the same reason (i.e., it only after a benzodiazepine has been administered.
is a potent enzyme inducer of CYP3A4). Carbamazepine, Although its efficacy is similar to that of benzodiazepines,
oxcarbazepine, and phenytoin could be given; however, it is not truly considered a first-line agent because of
extensive monitoring of the cyclosporine concentrations its adverse reactions. Phenobarbital causes prolonged
and of signs of organ rejection would be recommended. sedation because of its long half-life. Propofol (Option C)
is not the best option because it is considered a third-line
2. Answer: B therapy, given its adverse reactions. Propofol is associated
Levetiracetam is correct because it is available as an with metabolic acidosis syndrome, rhabdomyolysis, renal
injectable formulation. Levetiracetam is also available failure, and cardiac dysfunction. Levetiracetam (Option
in oral, tablet, and extended-release formulations. D) is not the best option because it is not a first-line agent
Levetiracetam is 100% bioavailable, so a one-to-one for status epilepticus.
conversion between intravenous and oral form can be
made when the patient is able to take oral medications. 5. Answer: B
You would also need to make sure that levetiracetam is Migraines can be set off by different types of triggers.
effective for the patient’s seizure type. Carbamazepine, Avoidance of these triggers is a good nonpharmacologic
topiramate, and lamotrigine are unavailable in an option for patients suffering from migraines. Migraine
injection formulation; therefore, they are not the best triggers include skipping meals, sleep deprivation,
choices currently. Lamotrigine is available as an orally excessive amounts of sleep, red wine, changes in weather,
disintegrating tablet but would be inappropriate for this fragrances, caffeine, and certain types of foods, among
patient because the tablets are still being absorbed by the other factors. In the patient case, the migraines are
GI system. Levetiracetam is FDA indicated as adjunctive possibly being set off by several known migraine triggers,
therapy, so as soon as the patient is taking oral medications, including stress, missing meals, caffeine consumption,
carbamazepine can be reinitiated, if necessary. alcohol intake, and a sporadic sleep schedule. Although
it is best to avoid these triggers, abruptly discontinuing
3. Answer: B a highly caffeinated beverage such as Mountain Dew
Answer B, dizziness from initial dose, is the best answer. may cause the patient to have rebound headaches. The
This is a dose-dependent adverse effect that can be patient should be counseled to wean herself off caffeine
mitigated by starting at a low dose. Option A is not the gradually rather than discontinue it abruptly. She must
best reason to start at a low dose because carbamazepine limit her red wine consumption and determine whether
will precipitate absence seizures at almost any dose. it is her main trigger.
Option C is not the best answer because rash is an
idiosyncratic reaction that occurs independently of
dose. Option D is not the best reason. The mechanism
of carbamazepine-induced hyponatremia is not fully
elucidated but may involve osmoreceptor perturbation
that develops over time. Starting at a low dose is not
known to reduce the risk.

ACCP Updates in Therapeutics® 2016: Ambulatory Care Pharmacy Preparatory Review and Recertification Course

1-514
Neurology

6. Answer: C occasionally seen; however, patients should be counseled


The most commonly used agents for migraine on the symptoms, and the encounter should be documented.
prevention include β-blockers, calcium channel blockers, Theoretically, long-acting triptans such as frovatriptan
antidepressants (TCAs and SSRIs), AEDs, and NSAIDs. and naratriptan have an increased likelihood of causing
Option A is incorrect because it was noted that, for serotonin syndrome. In contrast, triptans are only taken as
the past 6 months, the patient’s preventive drug had needed, not on a scheduled basis; therefore, the likelihood of
not been effective. With that information, it would be serotonin syndrome is minimal. Answer A is the best choice
inappropriate to continue giving the patient the same because Option C and Option D represent longer-acting
ineffective dose. As mentioned above, TCAs (e.g., triptans, so there is a higher risk of serotonin syndrome,
nortriptyline) are a good option for migraine prevention; and eletriptan is not metabolized by MAO. Rizatriptan
however, they are often accompanied by many adverse (Option B) is metabolized by MAO, so it represents a risk
effects (e.g., anticholinergic issues). In the patient case, of serotonin syndrome when combined with selegiline.
the patient was noted to suffer from dry mouth. Option B
is incorrect because increasing the dose of nortriptyline 9. Answer: A
could potentially worsen this adverse effect, thus making Although the current FM guidelines list all of these agents
it a less-than-ideal option. Candesartan (Option D) as acceptable for therapy, acetaminophen would be the
should only be used for patients with concomitant HTN. best choice in this situation. The EULAR (European
Propranolol (Answer C) is a good option for preventing League Against Rheumatism Guidelines for Fibromyalgia)
migraines because it is FDA label approved in adults for and the Canadian Fibromyalgia Guidelines both suggest
migraine prevention and is not associated with many acetaminophen as an initial therapy for mild pain. Although
adverse effects at low doses. the patient’s pain is moderate and her risk of using chronic
opioid therapy is high, current guidelines do not support
7. Answer: B the use of tramadol in patients with risk of misuse. Given
Triptans are good options for the abortive treatment of this assessment, tramadol would not be an appropriate
migraines. Although all are good agents, selection of a agent. Duloxetine would not be a good first-line agent for
triptan should be tailored to the individual patient. From this patient because of cost and her comorbid condition,
the patient case, we know the patient’s migraines usually bipolar disorder. The case states her bipolar disorder is
start around her menstrual cycle and generally last 24 currently under control with valproate, so therapies that
hours. In this instance, a long-acting triptan would be modify serotonin or dopamine would not be ideal in her
preferable. When migraines occur around the menstrual situation. Gabapentin, although effective for some patients,
cycle, it is appropriate to give a triptan with a long half- is often limited by adverse effects such as sedation. This
life before the menstrual cycle begins and to continue it agent may be considered if her condition does not respond
for a couple of days after the cycle is finished. The two to scheduled acetaminophen. With this patient, it will be
triptans with a long half-life are frovatriptan (half-life very important to provide education regarding FM and to
26 hours) and naratriptan (half-life 6 hours). Options A, encourage nonpharmacologic therapy such as exercise.
C, and D are not the best choices because sumatriptan,
rizatriptan, and almotriptan all have half-lives shorter 10. Answer: D
than 5 hours. This patient is now considered high risk—he has taken
another patient’s medication and has been routinely
8. Answer: A going to an ED for intramuscular injection opioids.
Triptans are serotonin-receptor agonists, so it is conceivable From the information in the case, this patient would
that any of the triptans could cause serotonin syndrome. be most appropriately managed by a specialist in pain
The patient case states that the patient is currently taking and addiction medicine. He should receive appropriate
selegiline, an MAOI, for Parkinson disease. Administration treatment for his pain as well as patient education and
of triptans that are metabolized by MAO in addition to an training in coping skills/behavioral therapy.
MAOI should be avoided. Triptans that are at least partly
metabolized by MAO include almotriptan, rizatriptan,
sumatriptan, and zolmitriptan. Serotonin syndrome is

ACCP Updates in Therapeutics® 2016: Ambulatory Care Pharmacy Preparatory Review and Recertification Course

1-515
Neurology

11. Answer: B hydromorphone. Although the threshold for detection may


The Brief Pain Inventory allows patients to indicate vary by test, the window of detection is usually 2–3 days.
on a diagram of the body where they are experiencing Patients should be educated on the importance of taking
pain and indicate their pain for that region. It also asks their medications as prescribed. If they are having less
patients to indicate the amount of relief they experience pain, tapering their opioid therapy should be considered.
from various therapies. Patients have several affected Immunoassay is a less expensive test, but it is subject to
anatomic areas and types of pain, and it is important false positives. Gas chromatography–mass spectrometry
for the clinician to differentiate the severity of pain by is a more expensive confirmatory test. A patient’s therapy
location. The McGill Pain Questionnaire may be useful should not be discontinued because of the results of a
to apply descriptors for pain; however, this will not single immunoassay urine drug screen. Serum drug
provide as much information about the severity of pain in screening can be used to obtain quantitative results for a
different anatomic locations. The VAS is not appropriate specific medication, but this type of test is costly and is not
for chronic pain in several locations. The FACES scale necessary in this situation.
is designed for pediatric patients and patients with
decreased literacy. 15. Answer: B
Most nonspecific urine drug screens, such as the
12. Answer: C immunoassay forms, will not recognize the synthetic
Current recommendations from the CDC for use of opioids. These include methadone and fentanyl. Tramadol
methadone include restricting its use to patients who are will also not be recognized as an opioid by the test.
opioid tolerant. Those with opioid tolerance are defined Verapamil will cause a false-positive result with these
as patients who are receiving the equivalent of 60 mg nonspecific tests. For patients taking these medications
of oral morphine each day. The normal starting dose known to cause false-positive results, more specific forms
for methadone is 2.5–5 mg twice daily, and all patients of testing should be used, such as GC-MS. Practitioners
beginning therapy should have a baseline ECG; however, should be familiar with tests used in their clinic settings
this patient is not a candidate for methadone initiation. and potential confounders for those tests. Patients should
Methadone and all opioids should be used with caution not have treatment decisions that are based entirely on the
in the elderly in consideration of increased risk of falls results of nonspecific urine assays without confirmation
and potential for polypharmacy. However, the elderly using more specific forms of testing.
are a population at risk of undertreatment of pain. As
with any group of patients, each elderly patient should be 16. Answer D
educated about the risks and benefits of therapy. The most appropriate tool listed for evaluating risk in
patients with a history of substance abuse during their
13. Answer: C opioid therapy is the COMM. This tool is designed to
The patient is currently prescribed a total daily dose of evaluate emotional lability, patient motivation to use
morphine of 270 mg. According to the package insert medication, and aberrant behaviors. The CAGE tool was
recommendation, this would correspond to a 75-mcg/ developed to identify potential addiction, and SOAPP
hour dose of the fentanyl patch. and ORT were developed to identify the risk level for
chronic opioid therapy in a general pain population.
14. Answer: B
The patient did not take his morning dose of hydrocodone 17. Answer A
before his urine drug test. Although urine drug screens Ibuprofen would be the best option for this patient,
can be used to help assist with adherence, these procedures whose acute pain is inflammatory in origin. Short-acting
are not fail-safe. Each test has a threshold for detection. opioids would be an option if his pain were moderate
Testing for hydrocodone (a short half-life) medication to severe or if he had not responded to therapy with a
is error prone when the medication is taken erratically. milder agent. Celecoxib, a COX-2–specific agent, is not
Currently, the history reported by the patient may be necessary in this case. The patient has no history of GI
in agreement with the current urine drug screen. Many disorders or failure of other NSAID therapies. Celecoxib
immunoassays for hydrocodone also detect its metabolite, is considerably more costly than ibuprofen.

ACCP Updates in Therapeutics® 2016: Ambulatory Care Pharmacy Preparatory Review and Recertification Course

1-516
Neurology

18. Answer C 22. Answer: C


Duloxetine would be the agent to add to this patient’s Dimethyl fumarate is the best option for this patient
regimen for chronic low back pain. The SNRIs work by right now. Its efficacy is better than the first-line DMT
modulating pain. Although duloxetine may increase BP, (annualized relapse rate 44%–53% vs. glatiramer 29%).
it has less propensity to increase BP than other SNRIs Its safety profile is also more favorable than that of the
such as venlafaxine. Pregabalin and gabapentin are both other, newer DMTs. Option A is not the best choice
associated with weight gain. currently because of the agent’s adverse effect profile
(secondary leukemias 1:145); it should be reserved for
19. Answer D patients whose condition does not respond to other
Risk Evaluation and Mitigation Strategies are designed agents. Option B is not the best choice because this
to decrease serious adverse effects and unintended agent is pregnancy category X, and a forced elimination
deaths associated with long-acting opioids. The FDA procedure with cholestyramine or activated charcoal
implemented the REMS program in July 2012. The would be necessary at least 2 years before pregnancy.
program requires manufacturers to provide education Option D is not the best option currently because this
to prescribers about the appropriate prescribing of their agent is less effective. Moreover, the patient indicates a
product, including the risks and benefits of its use. These preference for an oral agent.
concerns should be discussed with all patients before
therapy is initiated. Patients should be provided with 23. Answer: C
written information, including the medication guide, by Management of an acute attack should focus on
the pharmacist and counseled on the appropriate use of identifying and eliminating the stimulus. Bladder
the medication. distention and fecal impaction are one of the common
precipitants of autonomic dysreflexia. Because P.W. has
20. Answer: C bladder dysfunction and requires an indwelling catheter,
The most commonly reported adverse effects of obstruction in the catheter leading to a distended bladder
pyridostigmine are related to its cholinergic properties. may be the cause. If fecal impaction is the cause, P.W.
Pyridostigmine is primarily used to improve muscle would require prompt rectal evacuation. Option D is not
strength in MG but does not alter the course of the the best option because the onset of action for lactulose
disease progression. It is administered on a regular may take 24 to 48 hours. There is no basis for initiating
basis, and the development of tolerance is not known a regular antihypertensive agent (Option B). A prompt
to be associated with frequency of use. Pyridostigmine reduction in BP with a rapid-onset/short-duration agent
provides symptomatic relief, and the clinical effect such as a topical or sublingual nitrate can be considered,
occurs within 10–15 minutes. if necessary.

21. Answer: C 24. Answer: C


The best answer currently is glatiramer acetate because of This patient has experienced cognitive decline during the
this patient’s untreated depression and new diagnosis of past year, as indicated by the change in MMSE scores.
RRMS. Neither Option A nor Option B is the best choice The decline in MMSE score and the score of 23/30 are
currently because of the patient’s untreated depression, not diagnostic for AD or dementia. Hypothyroidism can
which is an adverse effect of interferons. Option D is contribute to cognitive symptoms; however, this patient
not the best choice currently because the patient has had a normal TSH reading 2 months ago. Alendronate
minimal disability, and fingolimod has a more extensive is not commonly known to cause cognitive symptoms.
adverse effect profile and requires increased monitoring. Tolterodine, however, can affect cognitive functioning
because of antimuscarinic effects, particularly in
patients who may have cognitive impairment at baseline.
Similarly, other nonspecific muscarinic antagonists
(oxybutynin, diphenhydramine) may be associated with
cognitive symptoms.

ACCP Updates in Therapeutics® 2016: Ambulatory Care Pharmacy Preparatory Review and Recertification Course

1-517
Neurology

25. Answer: A symptoms, but it will not address her worsening AD


The most common adverse effects of the cholinesterase symptoms. One of the pivotal studies with memantine
inhibitors as a class are GI in nature, including nausea, showed that adding memantine to donepezil provided
vomiting, and diarrhea. Dose titration reduces the additional clinical benefit beyond the use of either
likelihood of intolerability. This patient has been unable to donepezil or memantine alone.
tolerate galantamine 16 mg/day, the minimally effective
(target) dose from clinical studies; thus, galantamine at 28. Answer: A
this dose should be discontinued. Initiating memantine A recent study evaluated the use of amantadine in
could be considered in patients with moderate to severe patients with severe nonpenetrating TBI. The trial was
AD; however, this patient has mild AD, given the randomized and placebo controlled, enrolling patients
patient’s recent MMSE score of 22/30. Clinical studies between 4 and 16 weeks post injury who were in a
show that memantine has negligible benefit on the vegetative or minimally conscious state. Amantadine
cognitive and functional status of patients with mild AD. was initiated at 100 mg twice daily for 2 weeks and then
Some individuals tolerate one cholinesterase inhibitor titrated to 150 mg twice daily for week 3, followed by a
better than another; hence, switching this patient’s titration to 200 mg twice daily during week 4. Patients
medication to a different cholinesterase inhibitor would treated with amantadine had an accelerated rate of
be appropriate. From accumulated clinical data, fewer functional recovery, as rated by the Disability Rating
patients report GI intolerability with donepezil than with Scale, compared with placebo-treated patients.
rivastigmine. Furthermore, the rivastigmine dose of 6
mg twice daily is the target dose, not the starting dose. 29. Answer: C
Initiating rivastigmine at 6 mg twice daily could possibly Before initiating therapy for PD, patients should be
induce severe symptoms of nausea and vomiting. screened for the possibility of drug-induced symptoms
(antiemetics, antipsychotics). Because of dopamine-
26. Answer: D blocking activity, metoclopramide can induce PD-like
Tacrine, the first drug approved to treat AD, was features. In addition, because metoclopramide is renally
associated with liver toxicity and required periodic liver eliminated, it can accumulate in patients with impaired
function tests. Currently, no data suggest that concurrent renal function, increasing the likelihood of adverse effects.
use of vitamin E with cholinesterase inhibitors improves Metoclopramide should be reduced to the minimally
efficacy. The clinical effects of cholinesterase inhibitors effective dose or discontinued completely, if possible.
are very modest, so patients and families should have
reasonable expectations for what the medications are 30. Answer: D
able to achieve. Anticholinergic medications can reduce The recommendations from guidelines (American
the efficacy of cholinesterase inhibitors and worsen the Academy of Neurology, Movement Disorder Society)
cognitive symptoms of the disease, so they should be for the initial treatment of patients with PD suggest the
avoided in patients with AD. use of a dopaminergic agent (levodopa or a dopamine
agonist). Anticholinergic agents such as benztropine could
27. Answer: C be considered in patients whose predominant symptom
The donepezil 23-mg tablet dosage form was approved is tremor. However, many patients, particularly older
for use in patients who have been taking and tolerating patients, do not tolerate anticholinergic agents well (adverse
donepezil 10 mg/day for 3 months or longer. Clinical effects include confusion, constipation, dry mouth, and
studies have shown a small clinical benefit of the urinary retention), and guidelines do not support the use
23-mg dose compared with the 10-mg dose. However, of anticholinergic agents as initial therapy. Entacapone is
the incidence of GI adverse effects (nausea, vomiting, typically added to levodopa therapy when a patient begins
diarrhea, weight loss, anorexia) was much higher with to experience motor complications related to the long-term
the 23-mg dose, particularly among patients who use of levodopa, so initiating therapy with a combination of
weighed less than 55 kg. This patient currently has both drugs would not be justified. One study with coenzyme
occasional nausea with the 10-mg dose, and her body Q10 showed functional benefits when dosed at 1200 mg/
weight is quite low (51 kg). Decreasing the dose of day. However, current recommendations do not support the
donepezil may help the patient’s occasional nausea initiation of coenzyme Q10 as initial treatment of PD.

ACCP Updates in Therapeutics® 2016: Ambulatory Care Pharmacy Preparatory Review and Recertification Course

1-518
Neurology

31. Answer: D
The common adverse effects of levodopa at initiation
of therapy include nausea, vomiting, dizziness, and
hypotension. Adding a dopa decarboxylase inhibitor
such as carbidopa can considerably reduce these
peripheral adverse effects. The dose of carbidopa needed
to saturate peripheral dopa decarboxylase is between 75
and 100 mg/day. This patient is only receiving 30 mg
of carbidopa daily. Changing the levodopa/carbidopa
dosage formulation from 100/10 mg three times daily to
100/25 mg three times daily would increase the carbidopa
intake to 75 mg/day. Adding rasagiline to levodopa
therapy would not resolve the patient’s symptoms, and
it might actually make them worse. Decreasing the
levodopa/carbidopa 100/10 mg dose from three times
daily to twice daily might reduce the adverse effects, but
the carbidopa dose would still be insufficient, and the
lower daily levodopa dose might reduce the control of
her PD symptoms.

32. Answer: B
Abnormal involuntary movements such as dyskinesias
suggest excessive dopaminergic treatment. Nausea in
the context of abnormal involuntary movements can
also be a sign of excessive dopaminergic treatment.
When a second drug is added to levodopa therapy,
whether it is a MAO-B inhibitor, a COMT inhibitor, or a
dopamine agonist, most patients will require a decrease
in the levodopa dose to avoid possible dopamine-
related adverse effects (dyskinesias, nausea or vomiting,
hallucinations). Decreasing the levodopa dose would
be the best initial approach to managing this problem.
Changing from rasagiline to selegiline or ropinirole
would still be associated with a similar problem and
would likely necessitate a reduction in the levodopa dose.

ACCP Updates in Therapeutics® 2016: Ambulatory Care Pharmacy Preparatory Review and Recertification Course

1-519

You might also like